You are on page 1of 145

IAS BABA

RAPID REVISION SERIES

8
High Probable
Topics for
UPSC Prelims 2021
(Current Affairs + Static Portion)

िह ंदी
Full Compilation
POLITY
Make the best use of this document by combining free explanation
videos of these topics, Current Affairs Quiz, Static Quiz, CSAT Quiz, &
3 Full Mocks on rrs.iasbaba.Com (A Free Initiative targeting
UPSC PRELIMS 2021)
IASbaba’s Rapid Revision (RaRe) Series Polity Day 1-120 2021
HINDI

Q.1) '1773 के रेगल


ु ेट िंग एक् ' के बारे में टिम्िटलटित कथिों पर टिचार करें:
1. यह भारत में EIC के मामलों को नियंनित और नियनमत करिे के नलए निनिश सरकार द्वारा उठाया गया पहला कदम था।
2. इस अनिनियम द्वारा EIC के वानिनययक और राजिीनतक कायों को पृथक नकया गया।
3. इस अनिनियम द्वारा बॉम्बे और मद्रास प्रेसीडेंसी के गविनर बगं ाल के गविनर के अिीि हो गए।
उपरोक्त कथिों के आधार पर सही उत्तर का चयि कीटिये?
a) के वल 1 और 2
b) के वल 2 और 3
c) के वल 1 और 3
d) 1, 2 और 3
Q.1) Solution (c)

➢ 1773 का रे गल
ु ेनिगं एक्ि बहुत सवं ैिानिक महत्व का था क्योंनक यह निनिश सरकार द्वारा भारत में ईस्ि इनं डया कंपिी के
मामलों को नियंनित और नियनमत करिे के नलए उठाया गया पहला कदम था।

➢ इस अनिनियम िे पहले के नवपरीत, जब तीिों प्रेसीडेंसी एक दसू रे से स्वतंि थी, बॉम्बे और मद्रास प्रेसीडेंसी के गविनरों को
बंगाल के गविनर-जिरल के अिीिस्थ बिा नदया।

➢ नपि्स इनं डया एक्ि 1784 (Pitt's India Act 1784) द्वारा EIC के वानिनययक और राजिीनतक कायन को पृथक नकया गया।
Q.2) टिम्िटलटित में से टकस अटधटियम के पाररत होिे से भारत में किंपिी के क्षेत्रों को पहली बार 'भारत में टिट श सिंपटत्त'
कहा गया था?

a) रे गल
ु ेनिंग एक्ि 1773
b) चािनर अनिनियम 1813
c) चािनर अनिनियम 1793
d) नपि्स इनं डया अनिनियम 1784
Q.2) Solution (d)

➢ 1784 के नपि्स इनं डया अनिनियम (Pitt's India Act of 1784) के पाररत होिे के साथ, भारत में कंपिी के क्षेिों को
पहली बार 'भारत में निनिश सपं नि' कहा गया; तथा निनिश सरकार को भारत में कंपिी के मामलों और उसके प्रशासि पर
सवोच्च नियंिि प्राप्त हुआ।
Q.3) 1853 के चा टर अटधटियम द्वारा पहली बार:

a) गविनर जिरल की पररषद के नविायी और कायनकारी कायों को पृथक नकया गया।


b) एक वानिनययक निकाय के रूप में ईस्ि इनं डया कंपिी की सभी गनतनवनियों को समाप्त कर नदया गया।

www.iasbaba.com rrs.iasbaba.com Page 1


IASbaba’s Rapid Revision (RaRe) Series Polity Day 1-120 2021
HINDI

c) भारत में अंग्रेज़ों के अिीि संपिू न प्रादेनशक क्षेि पर अनिकार रखिे वाली सरकार का निमानि नकया गया।
d) उपरोक्त में से कोई िहीं।
Q.3) Solution (a)

➢ 1853 के चािनर अनिनियम (Charter Act of 1853) द्वारा पहली बार गविनर-जिरल की पररषद के नविायी और कायनकारी
कायों को पृथक नकया गया। इसिे पररषद में नविाि पाषनद कहे जािे वाले छह िए सदस्यों को जोड़िे का प्राविाि नकया। दसू रे
शब्दों में, इसिे एक अलग गविनर-जिरल की नविाि पररषद की स्थापिा की नजसे भारतीय (कें द्रीय) नविाि पररषद के रूप में
जािा जािे लगा। पररषद की इस नविायी शाखा िे निनिश ससं द के समाि प्रनियाओ ं को अपिाते हुए एक लघु ससं द के रूप
में कायन नकया। इस प्रकार, कािूि को पहली बार सरकार के एक नवशेष कायन के रूप में मािा गया, नजसके नलए नवशेष मशीिरी
और नवशेष प्रनिया की आवश्यकता होती थी।

➢ एक वानिनययक निकाय के रूप में ईस्ि इनं डया कंपिी की सभी गनतनवनियों को 1833 के चािनर अनिनियम द्वारा इस शतन के
साथ समाप्त कर नदया गया, नक भारत में कंपिी के क्षेि 'महामनहम, उिके वाररस और उिरानिकाररयों के नवश्वास में' थे।

➢ 1833 के चािनर अनिनियम िे बगं ाल के गविनर-जिरल को भारत का गविनर-जिरल बिा नदया और उिमें सभी िागररक और
सैन्य शनक्तयााँ निनहत हो गई।ं इस प्रकार, अनिनियम िे पहली बार, भारत में अंग्रेजों के कब्जे वाले परू े क्षेिीय क्षेि पर अनिकार
रखिे वाली भारत सरकार का निमानि नकया। लॉडन नवनलयम बेंनिक भारत के पहले गविनर-जिरल बिे।
Q.4) '1858 के भारत सरकार अटधटियम' के बारे में टिम्िटलटित कथिों पर टिचार करें:

1. इस अनिनियम को भारत की अच्छी सरकार के नलए अनिनियम के रूप में भी जािा जाता था।
2. इस अनिनियम िे बोडन ऑफ कंट्रोल और कोिन ऑफ डायरे क्िसन को खत्म करके दोहरी सरकार की व्यवस्था को समाप्त कर
नदया।
3. इस अनिनियम में नसनवल सेवकों के चयि और भती की खल
ु ी प्रनतस्पिान प्रिाली की शरुु आत की गयी।
उपरोक्त कथिों के आधार पर सही उत्तर का चयि कीटिये ?

a) के वल 1 और 2
b) के वल 2 और 3
c) के वल 1 और 3
d) 1, 2 और 3

Q.4) Solution (a)

➢ 1858 का भारत सरकार अनिनियम 1857 के नवद्रोह के मद्देिजर अनिनियनमत नकया गया था- नजसे स्वतंिता के प्रथम यद्ध ु
या 'नसपाही नवद्रोह' के रूप में भी जािा जाता है। भारत की अच्छी सरकार के नलए अनिनियम के रूप में नवख्यात इस अनिनियम

www.iasbaba.com rrs.iasbaba.com Page 2


IASbaba’s Rapid Revision (RaRe) Series Polity Day 1-120 2021
HINDI

िे ईस्ि इनं डया कंपिी को समाप्त कर नदया और निनिश राज (िाउि) को सरकार, क्षेिों और राजस्व की शनक्तयां हस्तातं ररत
कर दी।

➢ इसिे बोडन ऑफ कंट्रोल और कोिन ऑफ डायरे क्िसन को समाप्त कर दोहरी सरकार की व्यवस्था को समाप्त कर नदया। इसिे एक
िया कायानलय बिाया, भारत के नलए रायय सनचव, भारतीय प्रशासि पर पिू न अनिकार और नियंिि के साथ निनहत है। रायय
सनचव निनिश कै नबिेि का सदस्य होते थे जो अतं तः निनिश ससं द के प्रनत उिरदायी थे।

➢ 1853 के चािनर अनिनियम द्वारा नसनवल सेवकों के चयि और भती की खल


ु ी प्रनतयोनगता प्रिाली शरू
ु की गई थी।
Q.5) बिंगाल, उत्तर-पटिमी सीमा प्ािंत (NWFP), और पिंिाब के टलए िई टिधाि पररषदों की स्थापिा का प्ािधाि
टिम्िटलटित में से टकस अटधटियम द्वारा टकया गया?
a) भारतीय पररषद अनिनियम 1861
b) भारतीय पररषद अनिनियम 1892
c) भारतीय पररषद अनिनियम 1909
d) उपरोक्त में से कोई िहीं।
Q.5) Solution (a)
➢ 1861 के भारतीय पररषद अनिनियम में बगं ाल, उिर-पनिमी सीमांत प्रांत (NWFP) और पंजाब के नलए िई नविायी पररषदों
की स्थापिा का प्राविाि नकया गया था, जो िमशः 1862, 1886 और 1897 में स्थानपत हुई थी।

Q.6) 'भारत सरकार अटधटियम 1935' के सिंबध


िं में टिम्िटलटित कथिों पर टिचार करें:
1. इसिे लंदि में भारत के उच्चायक्त
ु का एक िया कायानलय बिाया और भारत के रायय सनचव द्वारा नकए गए कुछ कायों को उन्हें
स्थािांतररत कर नदया गया।
2. इस अनिनियम िे कें द्र और इकाइयों के बीच शनक्तयों को तीि सनू चयों - संघीय सचू ी, प्रांतीय सचू ी और समवती सचू ी के रूप
में नवभानजत नकया।
3. नबहार, असम और संयक्त ु प्रांत में एक नद्वसदिीय नविानयका की शरुु आत की गयी।
उपरोक्त कथिों के आधार पर सही उत्तर का चयि कीटिये?
a) के वल 1 और 2
b) के वल 2 और 3
c) के वल 1 और 3
d) 1, 2 और 3

Q.6) Solution (b)

www.iasbaba.com rrs.iasbaba.com Page 3


IASbaba’s Rapid Revision (RaRe) Series Polity Day 1-120 2021
HINDI

➢ भारत सरकार अनिनियम 1935 में एक अनखल भारतीय महासघं की स्थापिा का प्राविाि नकया गया था, नजसमें प्रातं ों और
ररयासतों को इकाइयों के रूप में शानमल नकया गया था। इस अनिनियम िे तीि सनू चयों के संदभन में कें द्र और प्ातां ों के बीच
शनक्तयों को नवभानजत नकया-संघीय सचू ी (कें द्र के नलए, 59 नवषयों के साथ), प्रातं ीय सचू ी (प्रातं ों के नलए, 54 नवषयों के
साथ) और समवती सचू ी (दोिों के नलए, 36 नवषयों के साथ)। वायसराय को अवनशष्ट शनक्तयां प्रदाि की गयी। हालानं क, यह
महासंघ कभी अनस्तत्व में िहीं आया क्योंनक ररयासतों िे इसमें शानमल होिे से इक
ं ार कर नदया।

➢ इसिे ग्यारह में से छह प्रांतों में नद्वसदिपद्धनत (Bicameralism) की शरुु आत की। इस प्रकार, बंगाल, बॉम्बे, मद्रास, नबहार,
असम और संयक्त ु प्रातं की नविानयकाओ ं को एक नविाि पररषद (उच्च सदि) और एक नविाि सभा (निम्ि सदि) से नमलकर
नद्वसदिीय बिाया गया था। हालांनक उि पर कई तरह के प्रनतबंि लगाये गए थे।

➢ 1919 के भारत सरकार अनिनियम िे लंदि में भारत के नलए उच्चायक्त ु का एक िया कायानलय बिाया और अब तक भारत
के रायय सनचव द्वारा नकये जािे वाले कुछ कायों को उन्हें स्थािांतररत कर नदया गया।
Q.7) टिम्िटलटित में से टकस अटधटियम िे भारतीय ररयासतों पर टिट श सिोच्चता की समाटि की घोषणा की?

a) भारतीय पररषद अनिनियम 1909


b) भारत सरकार अनिनियम 1919
c) भारत सरकार अनिनियम 1935
d) भारतीय स्वतिं ता अनिनियम 1947
Q.7) Solution (d)

➢ 1947 के भारतीय स्वतिं ता अनिनियम द्वारा भारत में निनिश शासि को समाप्त कर नदया गया और 15 अगस्त, 1947 से
भारत को एक स्वतंि और संप्रभु रायय घोनषत कर नदया गया। इस अनिनियम िे 15 अगस्त 1947 से भारतीय ररयासतों पर
निनिश सवोच्चता और आनदवासी क्षेिों के साथ सनं ि सबं िं ों के समानप्त की घोषिा की।

➢ इसिे भारतीय ररयासतों को या तो भारत या पानकस्ताि के प्रभत्ु व में शानमल होिे या स्वतिं रहिे की स्वतंिता प्रदाि की। इसिे
1935 के भारत सरकार अनिनियम द्वारा िए संनविािों के निमानि तक प्रत्येक उपनिवेश और प्रांतों के शासि प्राविाि नकया।
हालांनक ररयासतों को इस अनिनियम में संशोिि करिे के नलए अनिकृ त नकया गया था।
Q.8) सिंटिधाि सभा में प.िं ििाहरलाल िेहरू द्वारा पेश टकए गए ''उद्देश्य प्स्ताि'' के बारे में टिम्िटलटित कथिों पर टिचार
करें:

1. इसिे भारत के सभी लोगों को सामानजक, आनथनक और राजिीनतक न्याय की गारंिी और सरु क्षा प्रदाि की गयी।
2. इसिे अल्पसंख्यकों, नपछड़े और आनदवासी क्षेिों, और दनलतों और अन्य नपछड़े वगों के नलए पयानप्त सरु क्षा उपाय प्रदाि नकए।
3. उद्देश्य प्रस्ताव का संशोनित संस्करि वतनमाि संनविाि में 'रायय िीनत के निदेशक नसद्धातं ों' को प्रस्तुत करता है ।
उपरोक्त कथिों के आधार पर सही उत्तर का चयि कीटिये?

www.iasbaba.com rrs.iasbaba.com Page 4


IASbaba’s Rapid Revision (RaRe) Series Polity Day 1-120 2021
HINDI

a) के वल 1 और 2
b) के वल 2 और 3
c) के वल 1 और 3
d) 1, 2 और 3
Q.8) Solution (a)

➢ उद्देश्य प्रस्ताव को 1946 में पं. जे. िेहरू द्वारा प्रस्तुत नकया था इसिे सवं ैिानिक ढांचे के मल
ू नसद्धातं ों और दशनि को नििानररत
नकया। इसिे भारत के सभी लोगों को सामानजक, आनथनक और राजिीनतक न्याय की गारंिी तथा सरु क्षा एवं अवसर और कािूि
के समक्ष समािता; कािूि और सावनजनिक िैनतकता के अिीि नवचार, अनभव्यनक्त, आस्था, नवश्वास, पजू ा, व्यवसाय, संघ
और कारन वाई की स्वतंिता प्रदाि की।

➢ इसमें अल्पसंख्यकों, नपछड़े और जिजातीय क्षेिों और अवसादग्रस्त और अन्य नपछड़े वगों के नलए पयानप्त सरु क्षा उपायों का
प्राविाि नकया गया था।

➢ उद्देश्य प्रस्ताव का सश
ं ोनित सस्ं करि वतनमाि सनं विाि की प्रस्ताविा बिा।
Q.9) भारतीय सिंटिधाि की टिटभन्ि टिशेषताएिं टिश्व सिंटिधाि के कई दस्तािेिों से उधार ली गई हैं। इस सबिं िंध में
टिम्िटलटित कथिों पर टिचार करें:

1. संनविाि में संशोिि की प्रनिया जमनिी के वाईमर संनविाि से ली गई है।


2. व्यापार, वानियय और समागम की स्वतंिता की नवशेषता किाडा के संनविाि से ली गई है।
उपरोक्त कथिों के आधार पर सही उत्तर का चयि कीटिये?

a) के वल 1
b) के वल 2
c) 1 और 2 दोिों
d) ि तो 1 और ि ही 2
Q.9) Solution (d)

➢ संनविाि में संशोिि और राययसभा के सदस्यों के चिु ाव प्रनिया दनक्षि अफ्रीका के संनविाि से ली गई है।

➢ व्यापार, वानियय और समागम की स्वतिं ता और ससं द के दोिों सदिों की सयं क्त


ु बैठक की नवशेषताएं ऑस्ट्रेनलयाई सनं विाि
से उिार ली गई हैं।
Q.10) भारत के सटिं िधाि की अिुसूटचयों के सिंबध
िं में टिम्िटलटित कथिों पर टिचार करें:
1. संनविाि की तीसरी अिुसचू ी में उच्च न्यायालय के मख्ु य न्यायािीश के नलए शपथ या प्रनतज्ञाि के रूपों का प्राविाि है।

www.iasbaba.com rrs.iasbaba.com Page 5


IASbaba’s Rapid Revision (RaRe) Series Polity Day 1-120 2021
HINDI

2. अिुसनू चत क्षेिों और अिुसनू चत जिजानतयों के प्रशासि और नियंिि से संबंनित प्राविाि संनविाि की छठी अिुसचू ी में
नदए गए हैं।
उपरोक्त कथिों के आधार पर सही उत्तर का चयि कीटिये?

a) के वल 1
b) के वल 2
c) 1 और 2 दोिों
d) ि तो 1 और ि ही 2
Q.10) Solution (a)

सिंटिधाि की तीसरी अिुसूची में टिम्िटलटित के टलए शपथ या प्टतज्ञाि के रूपों का प्ािधाि है:

● कें द्रीय मिं ी


● ससं द चिु ाव के नलए उम्मीदवार
● संसद सदस्य
● सवोच्च न्यायालय के न्यायाधीश
● भारत के नियिं क और महालेखा परीक्षक
● रायय के मंिी
● रायय नविािमंडल चिु ाव उम्मीदवार
● रायय नविािमंडल सदस्य
● उच्च न्यायालयों के न्यायािीश
अिसु नू चत क्षेिों और अिसु नू चत जिजानतयों के प्रशासि और नियिं ि से सबं नं ित प्राविाि सनं विाि की पाचं वीं अिसु चू ी में नदए गए
हैं।

Q.11) टिम्िटलटित में से कौि सा उद्देश्य भारत के सिंटिधाि की प्स्ताििा में सटन्िटहत है?

1. राजिीनतक न्याय
2. आस्था की स्वतिं ता
3. वैधानिक न्याय
4. राजिीनतक स्वतिं ता
5. आनथनक समािता
उपरोक्त कथिों के आधार पर सही उत्तर का चयि कीटिये:

a) के वल 1 और 2

www.iasbaba.com rrs.iasbaba.com Page 6


IASbaba’s Rapid Revision (RaRe) Series Polity Day 1-120 2021
HINDI

b) के वल 1, 2, और 4
c) के वल 2, 4 और 5
d) 1, 2, 3, 4 और 5
Q. 11) Solution (a)

भारतीय सनं विाि की प्रस्ताविा एक सनं क्षप्त पररचयात्मक वक्तव्य है जो राष्ट्ट्र के लोगों का मागनदशनि करिे के नलए, सनं विाि के नसद्धातं ों
को प्रस्तुत करिे हेतु, उस स्रोत को इनं गत करिे के नलए नदशा-निदेश नििानररत करता है नजससे दस्तावेज़ अपिा अनिकार और अथन प्राप्त
करता है। यह लोगों की आशाओ ं और आकांक्षाओ ं को दशानता है। प्रस्ताविा को उस पररचय अथवा भनू मका के रूप में संदनभनत नकया
जा सकता है जो सपं िू न सनं विाि पर प्रकाश डालती है।

अपिे वतनमाि स्वरुप में प्रस्ताविा है:

"हम, भारत के लोग, भारत को एक संपर्ू ण प्रभत्ु व-संपन्न समाजवादी पंथननरपेक्ष लोकतत्रं ात्मक

गर्राज्य बनाने के नलए, तथा उसके समस्त नागररकों को:

सामानजक, आनथणक और राजनीनतक न्याय,

नवचार, अनभव्यनि, नवश्वास, धमण और उपासना की स्वतंत्रता,

प्रनतष्ठा और अवसर की समता,

प्राप्त कराने के नलए, तथा उन सब में,

व्यनि की गररमा और राष्ट्र की एकता और

अखण्डता सनु ननित करने वाली बंधतु ा बढाने के नलए दृढ संकनपपत होकर

अपनी इस संनवधान सभा में आज तारीख 26 नवम्बर 1949 ईस्वी (नमनत मागणशीर्ण शक्ु ल सप्तमी, संवत दो हजार छह नवक्रमी) को एतद्
द्वारा इस सनं वधान को अगं ीकृ त, अनधननयनमत और आत्मानपणत करते हैं।

इस प्रकार यह स्पष्ट है नक कािूिी/ वैधानिक न्याय, राजिीनतक स्वतंिता और आनथनक समािता भारतीय संनविाि की प्रस्ताविा में
शानमल िहीं हैं ।

Q. 12) भारतीय सटिं िधाि की "प्स्ताििा" के सबिं ध


िं में टिम्िटलटित कथिों पर टिचार करें:
1. यह नविानयका के नलए शनक्त का स्रोत है।
2. यह न्यायोनचत िहीं (non-justiciable) है।
उपरोक्त कथिों के आधार पर सही उत्तर का चयि कीटिये?

www.iasbaba.com rrs.iasbaba.com Page 7


IASbaba’s Rapid Revision (RaRe) Series Polity Day 1-120 2021
HINDI

a) के वल 1
b) के वल 2
c) 1 और 2 दोिों
d) ि तो 1 और ि ही 2
Q.12) Solution (b)

प्रस्ताविा मल
ू दशनि और मौनलक मल्ू यों-राजिीनतक, िैनतक और िानमनक- का प्रतीक है नजस पर संनविाि आिाररत है। इसमें संनविाि
सभा की भव्य और महाि दृनष्ट शानमल है और यह सनं विाि के सस्ं थापकों के सपिों और आकाक्ष ं ाओ ं को दशानता है। सनं विाि के नकसी
अन्य भाग की तरह प्रस्ताविा को भी संनविाि सभा द्वारा अनिनियनमत नकया गया था; लेनकि, बाकी संनविाि के लागू होिे के बाद ऐसा
नकया गया था। प्रस्ताविा को अंत में सनम्मनलत करिे का कारि यह सनु िनित करिा था नक यह संनविाि सभा द्वारा अपिाए गए संनविाि
के अिरू ु प हो। प्रस्ताविा को वोि के नलए अग्रेनषत करते हुए सनं विाि सभा के अध्यक्ष िे कहा, प्रश्न यह है नक क्या प्रस्ताविा सनं विाि
का नहस्सा है। प्रस्ताव को तब अपिाया गया था। इसनलए, सवोच्च न्यायालय द्वारा वतनमाि राय है नक प्रस्ताविा संनविाि का एक नहस्सा
है। संनविाि के संस्थापकों की राय के अिुरूप है।
हालााँनक, दो बातों पर ध्याि नदया जािा चानहए:

1. प्रस्ताविा ि तो नविानयका की शनक्त का स्रोत है और ि ही नविानयका की शनक्तयों पर रोक है।


2. यह गैर-न्यायसंगत है, अथानत इसके प्राविाि कािूि की अदालतों में लागू करिे योग्य िहीं हैं।
Q.13) भारत के सटिं िधाि में टिटहत राज्य िीटत के टिदेशक टसद्ािंतों के तहत टिम्िटलटित प्ािधािों पर टिचार करें:

1. मातृत्व राहत (maternity relief) का प्ावधाि करिा।


2. गरीबों को मफ्ु त काििू ी सहायता प्रदाि करिा।
3. कृ नष एवं पशपु ालि को आिनु िक एवं वैज्ञानिक आिार पर संगनठत करिा।
4. िि और उत्पादि के साििों की एकाग्रता की रोकथाम सनु िनित करिा।
उपरोक्त में से कौि से समाििादी टसद्ािंत हैं िो राज्य के िीटत टिदेशक टसद्ािंतों में पररलटक्षत होते हैं?

a) के वल 1, 2, और 3
b) के वल 2, 3, और 4
c) के वल 1, 2, और 4
d) 1, 2, 3 और 4
Q.13) Solution (c)

सनं विाि में रायय के िीनत निदेशक तत्वों का कोई वगीकरि िहीं है। हालांनक, उिकी नवषय-सामग्री और नदशा के आिार पर उन्हें तीि
व्यापक श्रेनियों में वगीकृ त नकया जा सकता है, जैसे समाजवादी, गािं ीवादी और उदारवादी-बौनद्धक।

www.iasbaba.com rrs.iasbaba.com Page 8


IASbaba’s Rapid Revision (RaRe) Series Polity Day 1-120 2021
HINDI

समाजवादी नसद्धातं समाजवाद की नवचारिारा को दशानते हैं। वे एक लोकतांनिक समाजवादी रायय के ढाचं े को नििानररत करते हैं, नजसका
उद्देश्य सामानजक और आनथनक न्याय प्रदाि करिा और कल्यािकारी रायय की नदशा में रास्ता तय करिा है। वे रायय को निदेनशत करते
हैं:

1. सामानजक, आनथनक और राजिीनतक न्याय द्वारा व्याप्त सामानजक व्यवस्था को सरु नक्षत करके आय, नस्थनत, सनु विाओ ं और
अवसरों में असमािताओ ं को कम करिा (अिुच्छे द 38) और लोगों के कल्याि को बढावा देिा।
2. (क) सभी िागररकों के नलए आजीनवका के पयानप्त सािि का अनिकार सरु नक्षत करिे के नलए; (ख) सामान्य नहत के नलए
समदु ाय के भौनतक संसाििों का समाि नवतरि; (ग) िि और उत्पादि के साििों की एकाग्रता की रोकथाम; (घ) परुु ष और
मनहलाओ ं के समाि कायन के नलए समाि वेति; (ङ) बलात दव्ु यनवहार के नवरुद्ध श्रनमकों और बच्चों के स्वास््य और शनक्त
का सरं क्षि; और (च) बच्चों के स्वस्थ नवकास के अवसर (अिच्ु छे द 39)।
3. समाि न्याय को बढावा देिा और गरीबों को मफ्ु त काििू ी सहायता प्रदाि करिा (अिुच्छे द 39 ए)।
4. बेरोजगारी, वृद्धावस्था, बीमारी और अक्षमता के मामलों में काम, नशक्षा और सावनजनिक सहायता का अनिकार सरु नक्षत करिा
(अिच्ु छे द 41)।
5. काम की न्यायसंगत और मािवीय पररनस्थनतयों और मातृत्व राहत का प्राविाि करिा (अिुच्छे द 42)।
6. सभी श्रनमकों के नलए एक जीनवत मजदरू ी, एक सभ्य जीवि स्तर और सामानजक और सांस्कृ नतक अवसरों को सरु नक्षत करिे
के नलए (अिुच्छे द 43)।
7. उद्योगों के प्रबंिि में श्रनमकों की भागीदारी सनु िनित करिे के नलए कदम उठािा (अिुच्छे द 43 ए)।
8. लोगों के पोषि स्तर और जीवि स्तर को ऊपर उठािा और सावनजनिक स्वास््य में सिु ार करिा (अिुच्छे द 47)।
Q.14) भारतीय सिंटिधाि में, न्यायपाटलका को कायटपाटलका से पृथक करिा शाटमल है:

a) सनं विाि की प्रस्ताविा


b) रायय के िीनत निदेशक नसद्धांत
c) मौनलक कतनव्य
d) िौवीं अिुसचू ी
Q.14) Solution (b)

रायय के िीनत निदेशक तत्व सनं विाि के भाग IV में अिुच्छे द 36 से 51 तक वनिनत हैं। सनं विाि के अिच्ु छे द 50 में रायय की
सावनजनिक सेवाओ ं में न्यायपानलका को कायनपानलका से पृथक करिे का प्राविाि है।

Q.15) टिम्िटलटित में से कौि भारतीय सिंटिधाि में टिधाटररत िागररकों के मौटलक कतटव्यों में से हैं?

1. प्राकृ नतक पयानवरि की रक्षा और सिु ार करिा।


2. भारत के सभी लोगों के बीच सद्भाव और समाि भाईचारे की भाविा को बढावा देिा।
3. बलात दव्ु यनवहार के नखलाफ श्रनमकों और बच्चों के स्वास््य और शनक्त की रक्षा करिा।
4. सावनजनिक सपं नि की रक्षा करिा और नहसं ा त्यागिा।
उपरोक्त कथिों के आधार पर सही उत्तर का चयि कीटिये:

www.iasbaba.com rrs.iasbaba.com Page 9


IASbaba’s Rapid Revision (RaRe) Series Polity Day 1-120 2021
HINDI

a) के वल 1 और 2
b) के वल 2, 3 और 4
c) के वल 1, 2 और 4
d) 1, 2, 3 और 4
Q.15) Solution (c)

मौनलक कतनव्यों को देशभनक्त की भाविा को बढावा देिे और भारत की एकता को बिाए रखिे में मदद करिे के नलए सभी िागररकों के
िैनतक दानयत्वों के रूप में पररभानषत नकया गया है। ये कतनव्य सनं विाि के भाग IV-A में नििानररत नकये गए हैं। अिच्ु छे द 51ए के अिसु ार,
यह भारत के प्रत्येक िागररक का कतनव्य होगा:

1. संनविाि का पालि करिा और उसके आदशों और संस्थािों, राष्ट्ट्रीय ध्वज और राष्ट्ट्रगाि का सम्माि करिा।
2. स्वतंिता के नलए राष्ट्ट्रीय संघषन को प्रेररत करिे वाले महाि आदशों को संजोिा और उिका अिुसरि करिा।
3. भारत की संप्रभतु ा, एकता और अखंडता को बिाए रखिा और उसकी रक्षा करिा।
4. देश की रक्षा करिा और ऐसा करिे के नलए बल
ु ाए जािे पर राष्ट्ट्रीय सेवा प्रदाि करिा।
5. भारत के सभी लोगों के बीच िानमनक, भाषाई और क्षेिीय या अिभु ागीय नवनविताओ ं से परे सद्भाव और समाि भाईचारे की
भाविा को बढावा देिा और मनहलाओ ं की गररमा के नलए अपमािजिक प्रथाओ ं का त्याग करिा।
6. देश की समग्र संस्कृ नत की समृद्ध नवरासत को महत्व देिा और संरनक्षत करिा।
7. विों, झीलों, िनदयों और वन्यजीवों सनहत प्राकृ नतक पयानवरि की रक्षा और सिु ार करिा और जीनवत प्रानियों के प्रनत करुिा
भाव रखिा।
8. वैज्ञानिक सोच, मािवतावाद और जांच और सिु ार की भाविा नवकनसत करिा।
9. सावनजनिक संपनि की रक्षा करिा और नहसं ा को त्यागिा।
10. व्यनक्तगत और सामनू हक गनतनवनि के सभी क्षेिों में उत्कृ ष्टता की नदशा में प्रयास करिा तानक राष्ट्ट्र निरंतर उद्यम और उपलनब्ि
के उच्च स्तर तक बढ सके ।
11. छह से चौदह वषन तक की आयु के बच्चे को नशक्षा के अवसर प्रदाि करिा। यह कतनव्य 86वें संनविाि संशोिि अनिनियम,
2002 द्वारा जोड़ा गया था।
रायय के िीनत निदेशक नसद्धांतों में बलात दव्ु यनवहार के नखलाफ श्रनमकों और बच्चों के स्वास््य और साम्यन के संरक्षि का प्राविाि
है।
Q.16) राज्य के िीटत टिदेशक तत्िों के सबिं ध िं में टिम्िटलटित में से कौि सा कथि सही है/हैं?

www.iasbaba.com rrs.iasbaba.com Page 10


IASbaba’s Rapid Revision (RaRe) Series Polity Day 1-120 2021
HINDI

1. वे सिा में पािी के पररवतनि के बावजदू घरेलू और नवदेशी िीनतयों में नस्थरता और निरंतरता को सगु म बिाते हैं।
2. वे िागररकों द्वारा मौनलक अनिकारों के पिू न और उनचत आिंद के नलए एक अिक ु ू ल वातावरि का निमानि करते हैं।
उपरोक्त कथिों के आधार पर सही उत्तर का चयि कीटिये:

a) के वल 1
b) के वल 2
c) 1 और 2 दोिों
d) ि तो 1 और ि ही 2
Q.16) Solution (c)

राज्य के िीटत टिदेशक तत्ि टिम्िटलटित महत्िपूणट भटू मकाएिं टिभाते हैं:

1. वे सिा में पािी के पररवतनि के बावजदू राजिीनतक, आनथनक और सामानजक क्षेिों में घरेलू और नवदेशी िीनतयों में नस्थरता
और निरंतरता को सगु म बिाते हैं।
2. वे िागररकों के मौनलक अनिकारों के परू क हैं। उिका उद्देश्य सामानजक और आनथनक अनिकार प्रदाि करके भाग III में ररक्त
स्थाि को भरिा है।
3. उिका कायानन्वयि िागररकों द्वारा मौनलक अनिकारों के पिू न और उनचत आिंद के नलए अिुकूल वातावरि बिाता है।
आनथनक लोकतिं के नबिा राजिीनतक लोकतंि का कोई अथन िहीं है।
4. वे नवपक्ष को सरकार के सचं ालि पर प्रभाव और नियिं ि करिे में सक्षम बिाते हैं। नवपक्ष सिािारी दल को इस आिार पर
दोष दे सकता है नक उिकी गनतनवनियां निदेशों के नखलाफ हैं।
5. वे सरकार के प्रदशनि के नलए एक महत्वपिू न परीक्षा के रूप में कायन करते हैं। लोग इि संवैिानिक घोषिाओ ं के आलोक में
सरकार की िीनतयों और कायनिमों की जाचं कर सकते हैं।
6. वे एक साझा राजिीनतक घोषिापि के रूप में कायन करते हैं। 'एक सिारूढ दल, चाहे वह अपिी राजिीनतक नवचारिारा का
हो, को इस त्य को पहचाििा होगा नक इि नसद्धातं ों का उद्देश्य उसके नविायी और कायनकारी कृ त्यों में उसका मागनदशनक,
दाशननिक और नमि होिा है।
Q.17) वाक एवं अटभव्यटक्त की स्ितत्रिं ता (Freedom of Expression) के अटधकार में शाटमल हैं:

1. वानिनययक नवज्ञापिों की स्वतिं ता।


2. न्यायािीश और वकीलों के बीच अदालतों में मौनखक वाद की ररपोिन की स्वतंिता।
3. मौि रहिे की स्वतिं ता।
4. सरकारी गनतनवनियों के बारे में जाििे का अनिकार।
उपरोक्त कथिों के आधार पर सही उत्तर का चयि कीटिये:

www.iasbaba.com rrs.iasbaba.com Page 11


IASbaba’s Rapid Revision (RaRe) Series Polity Day 1-120 2021
HINDI

a) के वल 1
b) के वल 1, 2 और 3
c) के वल 1, 2 और 4
d) 1, 2, 3 और 4
Q.17) Solution (d)

वाक एवां अनभव्यनक्त की स्वतिं ता (Freedom Of Expression) का तात्पयन है नक प्रत्येक िागररक को अपिे नवचार, राय, नवश्वास
और आस्था को मौनखक रूप से, लेखि, मद्रु ि, नचि या नकसी अन्य तरीके से स्वतिं रूप से व्यक्त करिे का अनिकार है। सप्रु ीम कोिन िे
मािा नक भाषि और अनभव्यनक्त की स्वतंिता में निम्िनलनखत शानमल हैं:

● अपिे नवचारों के साथ-साथ दसू रों के नवचारों को प्रचाररत करिे का अनिकार।


● पिकाररता की स्वतिं ता।
● वानिनययक नवज्ञापिों की स्वतिं ता।
● िेलीफोि पर बातचीत के िैनपंग के नखलाफ अनिकार।
● प्रसारि का अनिकार अथानत इलेक्ट्रॉनिक मीनडया पर सरकार का कोई एकानिकार िहीं है।
● नकसी राजिीनतक दल या संगठि द्वारा बल ु ाए गए बांद (bundh) के नखलाफ अनिकार।
● सरकारी गनतनवनियों के बारे में जाििे का अनिकार।
● मौि रहिे की स्वतिं ता।
● नकसी समाचार पि पर पवू न सेंसरनशप लगािे के नवरुद्ध अनिकार।
● प्रदशनि या िरिा देिे का अनिकार लेनकि हड़ताल करिे का अनिकार िहीं।
उच्चतम न्यायालय िे हाल ही में एक नििनय में कहा, नक मीनडया को न्यायािीशों और वकीलों के बीच अदालतों में मौनखक वाद सनहत
सिु वाई के दौराि की गई निप्पनियों की ररपोिन करिे का अनिकार है। यह अनभव्यनक्त की स्वतिं ता के अनिकार का नहस्सा है।

Q.18) टििता का अटधकार सिंटिधाि के टिम्िटलटित में से टकस अिुच्छे द के तहत एक मौटलक अटधकार है:

1. अिुच्छे द 19
2. अिुच्छे द 21
3. अिच्ु छे द 23
4. अिुच्छे द 24
उपरोक्त कथिों के आधार पर सही उत्तर का चयि कीटिये:

a) के वल 1 और 2
b) के वल 2 और 3
c) के वल 3 और 4

www.iasbaba.com rrs.iasbaba.com Page 12


IASbaba’s Rapid Revision (RaRe) Series Polity Day 1-120 2021
HINDI

d) के वल 2 और 4
Q.18) Solution (a)

न्यायमनू तन के .एस. पट्टु स्वामी (सेवानिवृि) बिाम भारत संघ में उच्चतम न्यायालय िे मािा नक निजता का अनिकार अिुच्छे द 21 के तहत
जीवि के अनिकार और व्यनक्तगत स्वतिं ता के आतं ररक नहस्से के रूप में और सनं विाि के भाग III द्वारा गारंिीकृ त स्वतिं ता के एक
नहस्से के रूप में संरनक्षत है।

सप्रु ीम कोिन िे यह भी मािा नक निजता का अनिकार एक पिू न अनिकार िहीं है और रायय या गैर-रायय अनभिेताओ ं द्वारा निजता के
नकसी भी अनतिमि को निम्िनलनखत तीि परीक्षिों को परू ा करिा चानहए:

1. वैि उद्देश्य
2. समािता
3. वैिता
Q.19) टिम्िटलटित में से कौि सा मौटलक अटधकार भारत के िागररकों और टिदेटशयों दोिों के टलए उपलब्ध है?

1. प्रारंनभक नशक्षा का अनिकार


2. भेदभाव का निषेि
3. जीवि और व्यनक्तगत स्वतिं ता की सरु क्षा
उपरोक्त कथिों के आधार पर सही उत्तर का चयि कीटिये:

a) के वल 1
b) के वल 3
c) के वल 1 और 2
d) के वल 2 और 3
Q.19) Solution (a)

टिम्िटलटित मौटलक अटधकार भारत के िागररकों और टिदेटशयों दोिों के टलए उपलब्ध हैं:

मौनलक अनिकारों का पालि भारत के िागररकों और नवदेनशयों दोिों के नलए उपलब्ि हैं:
1. नवनध के समक्ष समािता और नवनध का समाि संरक्षि (अिुच्छे द 14) ।
2. अपरािों के नलए सजा के संबंि में संरक्षि (अिुच्छे द 20)
3. जीवि और व्यनक्तगत स्वतिं ता की सरु क्षा (अिच्ु छे द 21)
4. प्रारंनभक नशक्षा का अनिकार (अिच्ु छे द 21A) ।
5. कुछ मामलों (अिुच्छे द 22) में नगरफ्तारी और िजरबंदी के नवरुद्ध सरु क्षा।
6. मािव दव्ु यानपार, बेगार तथा बिं आ
ु मज़दरू ी की प्रथा का उन्मल
ू ि (अिच्ु छे द 23)।
7. कारखािों आनद में बच्चों के रोजगार पर रोक ( अिुच्छे द 24)।

www.iasbaba.com rrs.iasbaba.com Page 13


IASbaba’s Rapid Revision (RaRe) Series Polity Day 1-120 2021
HINDI

8. अतं ःकरि की और िमन की अबाि रूप से माििे, आचरि और प्रचार करिे की स्वतंिता (अिुच्छे द 25)।
9. िानमनक कायों के प्रबंिि की स्वतिं ता (अिुच्छे द 26)।
10. नकसी भी िमन के प्रचार के नलए करों के भगु ताि से मनु क्त (अिुच्छे द 27)।
11. कुछ शैक्षनिक संस्थािों में िानमनक नशक्षा या पजू ा में भाग लेिे से स्वतिं ता (अिुच्छे द 28)
टिम्िटलटित मौटलक अटधकार के िल िागररकों के टलए उपलब्ध हैं, टिदेटशयों के टलए िहीं:

1. िमन, िस्ल, जानत, नलंग या जन्म स्थाि के आिार पर भेदभाव का निषेि (अिुच्छे द15)।
2. सावनजनिक रोजगार के मामलों में अवसर की समािता (अिुच्छे द 16)
3. स्वतिं ता के सबं िं में छह अनिकारों का सरं क्षि: (i) भाषि और अनभव्यनक्त, (ii) सभा, (iii) सघं , (iv) आदं ोलि, (v)
निवास, और (vi) व्यवसाय (अिुच्छे द 19)।
4. जीवि और व्यनक्तगत स्वतिं ता का संरक्षि (अिुच्छे द 21)।
5. अल्पसंख्यकों को शैनक्षक संस्थािों की स्थापिा और प्रशासि का अनिकार (अिुच्छे द 30)
Q.20) टिम्िटलटित में से कौि सा देश में लोकतािंटत्रक प्णाली का आधार है और साथ ही सरकार के अटधकार की पूणटता
की िािंच करता है?

a) प्रस्ताविा
b) रायय के िीनत निदेशक तत्व
c) मौनलक कतनव्य
d) मौनलक अनिकार
Q.20) Solution (d)

मौनलक अनिकार निम्िनलनखत मामलों में महत्वपिू न हैं:

● वे देश में लोकतानं िक व्यवस्था का आिार हैं।


● वे सरकार के अनिकार की पिू तन ा की जाचं करते हैं।
● वे मिुष्ट्य की भौनतक और िैनतक सरु क्षा के नलए आवश्यक शतें प्रदाि करते हैं।
● वे व्यनक्तगत स्वतिं ता के एक दजु ेय कवच के रूप में कायन करते हैं।
● वे देश में काििू के शासि की स्थापिा की सनु विा प्रदाि करते हैं ।
● वे अल्पसख्ं यकों और समाज के कमजोर वगों के नहतों की रक्षा करते हैं।
● वे भारतीय रायय के िमननिरपेक्ष तािे-बािे को मजबतू करते हैं।
● वे सामानजक समािता और सामानजक न्याय की आिारनशला रखते हैं।
● वे व्यनक्तयों की गररमा और सम्माि सनु िनित करते हैं ।
● वे राजिीनतक और प्रशासनिक प्रनिया में लोगों की भागीदारी को सगु म बिाते हैं।

www.iasbaba.com rrs.iasbaba.com Page 14


IASbaba’s Rapid Revision (RaRe) Series Polity Day 1-120 2021
HINDI

Q.21) प्त्यक्ष लोकतिंत्र के उपकरणों के सिंबिंध में टिम्िटलटित में से कौि सा कथि सही है:

1. ररफ़रें डम/ मत-सग्रां ह (Referendum) सावनजनिक महत्व के नकसी भी मद्दु े पर लोगों की राय प्राप्त करिे की एक नवनि है।
इसका उपयोग आमतौर पर क्षेिीय/ प्ादेनशक नववादों को हल करिे के नलए नकया जाता है।
2. पहल (Initiative) एक ऐसा तरीका है नजसके द्वारा लोग नविानयका को अनिनियनमत करिे के नलए एक नविेयक का
प्रस्ताव दे सकते हैं।
3. ररकॉल (Recall) एक ऐसी नवनि है नजसके माध्यम से मतदाता नकसी प्रनतनिनि या अनिकारी को उसके कायनकाल की
समानप्त से पहले हिा सकते हैं, जब वह अपिे कतनव्यों का ठीक से निवनहि करिे में नवफल रहता है।
4. जिमत-सग्रां ह (Plebiscite) एक ऐसी प्रनिया है नजसके तहत एक प्रस्तानवत काििू को मतदाताओ ं को उिके प्रत्यक्ष मतों
द्वारा निपिाि के नलए भेजा जाता है।
उपरोक्त कथिों के आधार पर सही उत्तर का चयि कीटिये:

a) के वल 1 और 2
b) के वल 2 और 3
c) के वल 3 और 4
d) के वल 1 और 4
Q. 21) Solution (b)

लोकतंि दो प्रकार का होता है-प्रत्यक्ष और अप्रत्यक्ष। प्रत्यक्ष लोकतिं में, लोग अपिी सवोच्च शनक्त का सीिे प्रयोग करते हैं जैसा नक
नस्वि्जरलैंड में होता है। प्रत्यक्ष लोकतंि के चार उपकरि हैं, अथानत् ररफ़रें डम (Referendum), पहल (Initiative), ररकॉल (Recall)
और जिमत-सांग्रह (Plebiscite)।

● ररफ़रें डम/ मत-सांग्रह (Referendum) एक ऐसी प्रनिया है नजसके तहत एक प्रस्तानवत कािूि को मतदाताओ ं को उिके प्रत्यक्ष
वोिों द्वारा निपिाि के नलए भेजा जाता है।
● पहल (Initiative) एक ऐसा तरीका है नजसके द्वारा लोग नविानयका को अनिनियनमत करिे के नलए एक नविेयक का प्रस्ताव
दे सकते हैं।
● ररकॉल (Recall) एक ऐसा नवनि है नजसके माध्यम से मतदाता नकसी प्रनतनिनि या अनिकारी को उसके कायनकाल की समानप्त
से पहले हिा सकते है, जब वह अपिे कतनव्यों का ठीक से निवनहि करिे में नवफल रहता है।
● जिमत-सांग्रह (Plebiscite) सावनजनिक महत्व के नकसी भी मद्दु े पर भारत के लोगों की राय प्राप्त करिे का एक तरीका है। यह
आमतौर पर क्षेिीय/ प्ादेनशक नववादों को हल करिे के नलए प्रयोग नकया जाता है।
Q. 22) भारतीय सटिं िधाि में टिम्िटलटित प्ािधािों में से कौि सा भारतीय राज्य की प्कृटत को पररभाटषत करता है?

1. मौनलक कतनव्य
2. मौनलक अनिकार
3. प्रस्ताविा

www.iasbaba.com rrs.iasbaba.com Page 15


IASbaba’s Rapid Revision (RaRe) Series Polity Day 1-120 2021
HINDI

4. रायय िीनत के निदेशक नसद्धांत


Q. 22) Solution (c)

प्रस्ताविा चार अवयवों या घिकों को प्रकि करती है:

1. संनविाि के अनिकार का स्रोत: प्रस्ताविा में कहा गया है नक संनविाि भारत के लोगों से अपिा अनिकार प्राप्त करता है।

2. भारतीय रायय की प्रकृ नत: यह भारत को एक संप्रभ,ु समाजवादी, िमननिरपेक्ष लोकतानं िक और गितिं ात्मक रायय घोनषत करता है।

3. संनविाि के उद्देश्य: यह न्याय, स्वतंिता, समािता और बंित्ु व को उद्देश्यों के रूप में निनदनष्ट करता है।

4. संनविाि को अपिािे की नतनथ: यह तारीख के रूप में 26 िवंबर, 1949 को नििानररत करती है।

Q. 23) सटिं िधाि की प्स्ताििा भारत को सप्िं भु गणराज्य घोटषत करती है। भारतीय सदिं भट में "गणराज्य" शब्द क्या है?

1. भारत में एक निनित अवनि के नलए एक निवाननचत प्रमख ु होता है।


2. निवाननचत प्रमख
ु में राजिीनतक संप्रभतु ा का निनहत होिा।
3. नकसी भी नवशेषानिकार प्राप्त वगन की अिुपनस्थनत।
उपरोक्त कथिों के आधार पर सही उत्तर का चयि कीटिये?

a) के वल 1 और 2
b) के वल 1 और 3
c) के वल 2 और 3
d) 1, 2 और 3
Q. 23) Solution (b)

एक लोकतानं िक व्यवस्था को दो श्रेनियों में वगीकृ त नकया जा सकता है- राजशाही और गितिं । एक राजशाही में, रायय के प्रमख ु
(आमतौर पर राजा या रािी) को वश ं ािुगत नस्थनत प्राप्त होती है, अथानत वह उिरानिकार के माध्यम से कायानलय में आता है, उदाहरि के
नलए, नििेि। एक गितंि में, दसू री ओर, रायय के प्रमख ु हमेशा प्रत्यक्ष या अप्रत्यक्ष रूप से एक निनित अवनि के नलए चिु े जाते हैं,
उदाहरि के नलए, संयक्त
ु रायय अमेररका।
इसनलए, हमारी प्रस्ताविा में 'गितिं ' शब्द इनं गत करता है नक भारत में एक निवाननचत प्रमख
ु है नजसे राष्ट्ट्रपनत कहा जाता है। वह अप्रत्यक्ष
रूप से पाचं साल की तय अवनि के नलए चिु े जाते हैं।

एक 'गितिं ' के अथन में दो और चीजें भी हैं: एक, लोगों में राजिीनतक संप्रभतु ा का निनहत होिा और राजा जैसे नकसी एक व्यनक्त में िहीं;
दसू रा, नकसी नवशेषानिकार प्राप्त वगन का अभाव और इसनलए सभी सावनजनिक कायानलयों को नबिा नकसी भेदभाव के प्रत्येक िागररक
के नलए खल ु े रहते हैं।

www.iasbaba.com rrs.iasbaba.com Page 16


IASbaba’s Rapid Revision (RaRe) Series Polity Day 1-120 2021
HINDI

Q. 24) मौटलक अटधकारों की टिशेषताओ िं के सबिं ध


िं में टिम्िटलटित कथिों पर टिचार करें:
1. वे न्यायोनचत (justiciable) हैं।
2. वे निरपेक्ष (absolute) और अहनताप्राप्त (qualified) हैं।
3. उिमें से कुछ चररि में िकारात्मक हैं।
4. वे अिुल्लंघिीय और स्थायी हैं।
उपरोक्त कथिों के आधार पर सही उत्तर का चयि कीटिये?

a) के वल 1 और 4
b) के वल 2 और 3
c) के वल 2 और 4
d) के वल 1 और 3
Q. 24) Solution (d)

संनविाि द्वारा गारंिीकृ त मौनलक अनिकारों की नवशेषता निम्िनलनखत है:

● वे न्यायोनचत हैं, यनद उिका उल्लघं ि नकया जाता है, तो व्यनक्तयों को उिके प्रवतनि के नलए अदालतों में जािे की अिमु नत दी
गई है।

● वे निरपेक्ष िहीं बनल्क अहनताप्राप्त (qualified) हैं। रायय उि पर उनचत प्रनतबिं लगा सकता है। हालााँनक, इस तरह के प्रनतबिं
उनचत हैं या िहीं, यह अदालतों द्वारा तय नकया जािा है। इस प्रकार, वे व्यनक्तगत स्वतिं ता और सामानजक नियंिि के बीच
व्यनक्त और परू े समाज के अनिकारों के बीच संतल ु ि बिाते हैं।

● उिमें से कुछ चररि में िकारात्मक हैं, अथानत,् रायय के अनिकार पर सीमाएं लगाते हैं, जबनक अन्य प्रकृ नत में सकारात्मक हैं,
व्यनक्तयों को कुछ नवशेषानिकार प्रदाि करते हैं।

● वे अिुल्लंघिीय या स्थायी िहीं हैं। संसद उन्हें कम या निरस्त कर सकती है लेनकि के वल एक संवैिानिक संशोिि अनिनियम
द्वारा, ि की एक सामान्य अनिनियम द्वारा। इसके अलावा, यह संनविाि के 'बुनियादी ढांचे' को प्रभानवत नकए नबिा नकया जा
सकता है।
Q. 25) सिंटिधाि के भाग-3 के टिटभन्ि प्ािधािों में उटललटित "राज्य" शब्द में क्या शाटमल है?

1. भारत की ससं द
2. राययों के नविािमंडल
3. स्थािीय अनिकारी
4. सानं वनिक और गैर-सानं वनिक प्रानिकरि

www.iasbaba.com rrs.iasbaba.com Page 17


IASbaba’s Rapid Revision (RaRe) Series Polity Day 1-120 2021
HINDI

िीचे टदए गए कू ों में से सही उत्तर का चयि कीटिये:

a) के वल 1
b) के वल 1 और 2
c) के वल 1, 2 और 3
d) 1, 2, 3 और 4
Q. 25) Solution (d)

मौनलक अनिकारों से संबंनित नवनभन्ि प्राविािों में 'रायय' शब्द का प्रयोग नकया गया है। इसनलए, अिच्ु छे द 12 िे भाग-3 के प्रयोजिों
के नलए शब्द को पररभानषत नकया है। इसके अिुसार, रायय में निम्िनलनखत शानमल हैं:

1. भारत की सरकार और संसद, अथानत कें द्र सरकार की कायनपानलका और नविायी अगं ।
2. राययों की सरकार और नविानयका अथानत रायय सरकार की कायनपानलका और नविायी अंग।
3. सभी स्थािीय प्रानिकरि, जो िगर पानलकाओ,ं पचं ायतों, नजला बोडों, सिु ार न्यास आनद हैं।
4. अन्य सभी प्रानिकरि, अथानत एलआईसी, ओएिजीसी, सेल आनद जैसे सांनवनिक या गैर-सांनवनिक प्रानिकरि।
इस प्रकार, रायय को व्यापक अथों में पररभानषत नकया गया है तानक इसकी सभी एजेंनसयों को शानमल नकया जा सके । इि एजेंनसयों के
कायों को मौनलक अनिकारों के उल्लंघि के रूप में अदालतों में चिु ौती दी जा सकती है।

उच्चतम न्यायालय के अिुसार, यहां तक नक रायय के सािि के रूप में काम करिे वाली एक निजी संस्था या एजेंसी भी अिुच्छे द 12 के
अंतगनत 'रायय' के अथन में आती है।

Q. 26) टिम्िटलटित कथिों पर टिचार करें:

1. छह से चौदह वषन की आयु के सभी बच्चों को मफ्ु त और अनिवायन नशक्षा को वाक एवां अनभव्यनक्त की स्वतंिता के मौनलक
अनिकार के रूप में मान्यता दी गई है।
2. नशक्षा के अनिकार के अिसु रि में ससं द िे निशल्ु क और अनिवायन नशक्षा का अनिकार अनिनियम, 2009 लागू नकया।
उपरोक्त कथिों के आधार पर सही उत्तर का चयि कीटिये?

a) के वल 1
b) के वल 2
c) 1 और 2 दोिों
d) ि तो 1 और ि ही 2
Q. 26) Solution (b)

www.iasbaba.com rrs.iasbaba.com Page 18


IASbaba’s Rapid Revision (RaRe) Series Polity Day 1-120 2021
HINDI

अिुच्छे द 21 ए में घोषिा की गई है नक रायय छह से चौदह वषन की आयु के सभी बच्चों को रायय द्वारा नििानररत तरीके से मफ्ु त और
अनिवायन नशक्षा प्रदाि करेगा। इस प्रकार, यह प्राविाि के वल प्रारंनभक नशक्षा को मौनलक अनिकार बिाता है, ि नक उच्च या व्यावसानयक
नशक्षा को।

यह प्राविाि 2002 के 86वें संनविाि संशोिि अनिनियम द्वारा जोड़ा गया था। यह संशोिि 'सभी के नलए नशक्षा' प्राप्त करिे के देश के
उद्देश्य में एक प्रमख
ु मील का पत्थर है। सरकार िे इस कदम को 'िागररकों के अनिकारों के अध्याय में दसू री िांनत की सबु ह' बताया।
उच्चतम न्यायालय िे अिच्ु छे द 21 के तहत जीवि के अनिकार में प्राथनमक नशक्षा के मौनलक अनिकार को मान्यता दी।

अिुच्छे द 21ए के अिुसरि में संसद िे निशल्ु क एवं अनिवायन नशक्षा का अनिकार अनिनियम, 2009 लागू नकया। इस अनिनियम में यह
प्राविाि करिे का प्राविाि नकया गया है नक प्रत्येक बच्चे को औपचाररक स्कूल में संतोषजिक और समाि गिु विा की पिू नकानलक
प्रारंनभक नशक्षा प्रदाि करिे का अनिकार है जो कुछ आवश्यक मािदडं ों और मािकों को परू ा करता है।

Q. 27) टिम्िटलटित कथिों पर टिचार करें:

1. मौनलक अनिकारों के प्रवतनि के मामले में सवोच्च न्यायालय के पास मल


ू और अिन्य क्षेिानिकार है।
2. सप्रु ीम कोिन उच्च न्यायालय के अलावा नकसी अन्य अदालत को मौनलक अनिकारों को लागू करिे के नलए ररि जारी करिे
का अनिकार दे सकता है।
उपरोक्त कथिों के आधार पर सही उत्तर का चयि कीटिये?

a) के वल 1
b) के वल 2
c) 1 और 2 दोिों
d) ि तो 1 और ि ही 2
Q. 27) Solution (d)

मौनलक अनिकारों के प्रवतनि के मामले में, उच्चतम न्यायालय का क्षेिानिकार मलू है लेनकि अिन्य िहीं है। यह अिुच्छे द 226 के तहत
उच्च न्यायालय के अनिकार क्षेि के साथ समवती है। यह मौनलक अनिकारों को लागू करिे के नलए सभी प्रकार के निदेश, आदेश और
ररि जारी करिे के नलए उच्च न्यायालय में मलू शनक्तयां निनहत करता है। इसका मतलब है नक जब नकसी िागररक के मौनलक अनिकारों
का हिि होता है तो पीनड़त पक्ष के पास सीिे उच्च न्यायालय या उच्चतम न्यायालय जािे का नवकल्प होता है।

अिुच्छे द 32 एक पीनड़त िागररक के मौनलक अनिकारों के प्रवतनि के नलए उपचार का अनिकार प्रदाि करता है। उच्चतम न्यायालय िे
फै सला सिु ाया है नक अिुच्छे द 32 संनविाि की एक मलू नवशेषता है। इसनलए, इसे संनविाि में संशोिि के माध्यम से भी संनक्षप्त या
हिाया िहीं जा सकता है। इसमें निम्िनलनखत चार प्राविाि शानमल हैं:

● मौनलक अनिकारों के प्रवतनि के नलए उनचत कायनवाही द्वारा उच्चतम न्यायालय जािे के अनिकार की गारंिी है।

www.iasbaba.com rrs.iasbaba.com Page 19


IASbaba’s Rapid Revision (RaRe) Series Polity Day 1-120 2021
HINDI

● उच्चतम न्यायालय के पास नकसी भी मौनलक अनिकार के प्रवतनि के नलए निदेश या आदेश या ररि जारी करिे का अनिकार
होगा। जारी नकए गए ररि में बंदी प्रत्यक्षीकरि, परमादेश, निषेि, उत्प्रेषि और अनधकार-पृच्छा शानमल हो सकते हैं।
● ससं द नकसी भी अन्य न्यायालय को सभी प्रकार के निदेश, आदेश और ररि जारी करिे का अनिकार दे सकती है। हालााँनक,
यह सवोच्च न्यायालय को प्रदि उपरोक्त शनक्तयों पर प्रनतकूल प्रभाव डाले नबिा नकया जा सकता है। यहां नकसी अन्य न्यायालय
में उच्च न्यायालय शानमल िहीं हैं क्योंनक अिुच्छे द 226 उच्च न्यायालयों को पहले ही ये शनक्तयां प्रदाि कर चक ु ा है।
● उच्चतम न्यायालय में जािे का अनिकार संनविाि द्वारा प्रदाि नकए गए अन्यथा के अलावा निलंनबत िहीं नकया जाएगा। इस
प्रकार सनं विाि में प्राविाि है नक राष्ट्ट्रपनत राष्ट्ट्रीय आपातकाल (अिच्ु छे द 359) के दौराि मौनलक अनिकारों के प्रवतनि के
नलए नकसी भी अदालत में जािे के अनिकार को निलंनबत कर सकते हैं।
Q. 28) सिंपटत्त के अटधकार को मौटलक अटधकारों की सूची से ह ा टदया गया और 1978 के 44 िें सिंटिधाि सिंशोधि
अटधटियम द्वारा एक कािूिी अटधकार बिाया गया। इसके टिटहताथट क्या है/हैं?

1. निजी सपं नि कायनकारी कारन वाई के नखलाफ सरु नक्षत है लेनकि नविायी कारन वाई के नखलाफ िहीं।
2. जब रायय अल्पसख्ं यक शैक्षनिक सस्ं थाि की सपं नि का अनिग्रहि करता है तो मआ ु वजे का कोई गारंिीकृ त अनिकार
प्रदाि िहीं नकया जाता है।
3. जब रायय व्यनक्तगत कृ नष के अांतगगत नकसी व्यनक्त द्वारा िाररत भनू म का अनिग्रहि करता है तो मआ
ु वजे का कोई गारंिीशदु ा
अनिकार प्रदाि िहीं नकया जाता है।
उपरोक्त कथिों के आधार पर सही उत्तर का चयि कीटिये?

a) के वल 1
b) के वल 1 और 2
c) के वल 3
d) 1, 2 और 3
Q. 28) Solution (a)

मल ू रूप से, सपं नि का अनिकार सनं विाि के भाग III के तहत सात मौनलक अनिकारों में से एक था। 1978 के 44 वें सश
ं ोिि अनिनियम
िे भाग III से अिुच्छे द 19 (1) (F) और अिुच्छे द 31 को निरस्त करके संपनि के अनिकार को मौनलक अनिकार के रूप में समाप्त कर
नदया। इसके बजाय, अनिनियम िे 'संपनि का अनिकार' शीषनक के तहत भाग XII में एक िया अिुच्छे द 300A सनम्मनलत नकया। इसमें
प्राविाि है नक कोई भी व्यनक्त काििू के अनिकार को छोड़कर उसकी सपं नि से वनं चत िहीं रहेगा।

इस प्रकार, सपं नि का अनिकार अभी भी एक काििू ी अनिकार या एक सवं ैिानिक अनिकार बिा हुआ है, हालानं क अब एक मौनलक
अनिकार िहीं है। यह संनविाि के मल
ू ढाचं े का नहस्सा िहीं है।
कािूिी अनिकार के रूप में संपनि के अनिकार (मौनलक अनिकारों से अलग) के निम्िनलनखत निनहताथन हैं:

www.iasbaba.com rrs.iasbaba.com Page 20


IASbaba’s Rapid Revision (RaRe) Series Polity Day 1-120 2021
HINDI

● इसे संसद के एक सामान्य कािूि द्वारा संवैिानिक संशोिि के नबिा नवनियनमत नकया जा सकता है, अथानत, घिाया, संनक्षप्त
या सशं ोनित नकया जा सकता है।
● यह कायनकारी कारन वाई के नखलाफ निजी सपं नि की रक्षा करता है लेनकि नविायी कारन वाई के नखलाफ िहीं।
● उल्लंघि के मामले में, पीनड़त व्यनक्त इसके प्रवतनि के नलए अिुच्छे द 32 (ररि सनहत संवैिानिक उपचार का अनिकार) के
तहत सीिे सवोच्च न्यायालय िहीं जा सकता है। वह अिुच्छे द 226 के तहत उच्च न्यायालय जा सकता है।
● रायय द्वारा निजी संपनि के अनिग्रहि या मांग के मामले में मआ
ु वजे का कोई गारंिी अनिकार िहीं है।

हालांनक भाग III के तहत संपनि के मौनलक अनिकार को समाप्त कर नदया गया है,लेनकि भाग III में अभी भी दो प्राविाि हैं जो रायय
द्वारा निजी सपं नि के अनिग्रहि या मांग के मामले में मआ
ु वजे के गारंिीकृ त अनिकार का प्राविाि करते हैं। ये दो मामले जहां मआ
ु वजे
का भगु ताि नकया जािा है:
● जब रायय अल्पसंख्यक शैक्षनिक संस्थाि की संपनि का अनिग्रहि करता है (अिुच्छे द 30); तथा
● जब रायय व्यनक्तगत कृ नष के अांतगगत नकसी व्यनक्त द्वारा िाररत भनू म का अनिग्रहि करता है और भनू म वैिानिक सीमा के भीतर
है (अिुच्छे द 31 ए)।

पहला प्राविाि 44वें संशोिि अनिनियम (1978) द्वारा जोड़ा गया था, जबनक दसू रा प्राविाि 17वें संशोिि अनिनियम (1964) द्वारा
जोड़ा गया।

Q. 29) टिम्िटलटित टिदेशक टसद्ािंतों में से कौि सा समाििाद की टिचारधारा को दशाटता है?

1. कायन हेतु न्यायसंगत और मािवीय पररनस्थनतयों और मातृत्व राहत का प्राविाि करिा।


2. बच्चों के स्वस्थ नवकास के अवसरों को सरु नक्षत करिा।
3. परू े देश में सभी िागररकों के नलए एक समाि िागररक सनं हता को सरु नक्षत करिा।
उपरोक्त कथिों के आधार पर सही उत्तर का चयि कीटिये?

a) के वल 1 और 2
b) के वल 2 और 3
c) के वल 1 और 3
d) 1, 2 और 3
Q. 29) Solution (a)

सनं विाि में निदेशक नसद्धातं ों का कोई वगीकरि िहीं है। हालानं क, उिकी सामग्री और नदशा के आिार पर उन्हें तीि व्यापक श्रेनियों में
वगीकृ त नकया जा सकता है, जैसे समाजवादी, गांिीवादी और उदारवादी-बौनद्धक।

www.iasbaba.com rrs.iasbaba.com Page 21


IASbaba’s Rapid Revision (RaRe) Series Polity Day 1-120 2021
HINDI

समाजवादी नसद्धातं समाजवाद की नवचारिारा को दशानते हैं। उन्होंिे एक लोकतांनिक समाजवादी रायय की रूपरेखा तैयार की, नजसका
उद्देश्य सामानजक और आनथनक न्याय प्रदाि करिा और कल्यािकारी रायय की ओर मागन प्रशस्त करिा था। वे रायय को निदेनशत करते
हैं:

1. सामानजक, आनथनक और राजिीनतक न्याय द्वारा व्याप्त सामानजक व्यवस्था को सरु नक्षत करके लोगों के कल्याि को बढावा
देिा और आय, नस्थनत, सनु विाओ ं और अवसरों में असमािताओ ं को कम करिा (अिुच्छे द 38)।
2. सभी िागररकों के नलए (क) आजीनवका के पयानप्त साििों के अनिकार सरु नक्षत करिा; (ख) आम भलाई के नलए समदु ाय के
भौनतक संसाििों का समाि नवतरि; (ग) िि की एकाग्रता और उत्पादि के साििों की रोकथाम; (घ) परुु षों और मनहलाओ ं
के नलए समाि कायन के नलए समाि वेति; (ङ) बलात दव्ु यनवहार के नवरुद्ध कामगारों और बच्चों के स्वास््य और शनक्त का
सरं क्षि; और (च) बच्चों के स्वास््य नवकास के नलए अवसर (अिच्ु छे द 39)।
3. समाि न्याय को बढावा देिा और गरीबों को मफ्ु त काििू ी सहायता प्रदाि करिा (अिुच्छे द 39 A)।
4. बेरोजगारी, वृद्धावस्था, बीमारी और नवकलांगता के मामलों में काम, नशक्षा और सावनजनिक सहायता के अनिकार को सरु नक्षत
करिा। (अिच्ु छे द 41)
5. काम हेतु न्यायसंगत और मािवीय पररनस्थनतयों और मातृत्व राहत का प्राविाि करिा (अिुच्छे द 42)।
6. सभी श्रनमकों के नलए एक जीवियापि वेति, एक सभ्य जीवि स्तर और सामानजक और सांस्कृ नतक अवसरों को सरु नक्षत
करिा (अिुच्छे द 43)।
7. उद्योगों के प्रबंिि में श्रनमकों की भागीदारी सनु िनित करिे के नलए कदम उठािा (अिुच्छे द 43 A)।
8. लोगों के पोषि स्तर और जीवि स्तर को ऊपर उठािा और सावनजनिक स्वास््य में सिु ार करिा (अिुच्छे द 47)।

Q.30) राज्य के िीटत टिदेशक टसद्ािंतों में से कौि से 1976 के 42 िें सिंशोधि अटधटियम द्वारा िोडे गए थे?

1. बच्चों के स्वस्थ नवकास के अवसरों को सरु नक्षत करिा।


2. आय, नस्थनत, सनु विाओ ं और अवसरों में असमािताओ ं को कम करिा
3. उद्योगों के प्रबंिि में श्रनमकों की भागीदारी सनु िनित करिे के नलए कदम उठािा।
4. सहकारी सनमनतयों के स्वैनच्छक गठि, स्वायि कामकाज, लोकतांनिक नियंिि और पेशेवर प्रबंिि को बढावा देिा।
उपरोक्त कथिों के आधार पर सही उत्तर का चयि कीटिये?

a) के वल 1 और 2
b) के वल 2 और 4
c) के वल 3 और 4
d) के वल 1 और 3
Q. 30) Solution (d)

1976 के 42वें संशोिि अनिनियम िे मल


ू सचू ी में चार िए निदेशक नसद्धांतों को जोड़ा। वे रायय के नलए आवश्यक है:

www.iasbaba.com rrs.iasbaba.com Page 22


IASbaba’s Rapid Revision (RaRe) Series Polity Day 1-120 2021
HINDI

1. बच्चों के स्वस्थ नवकास के नलए अवसरों को सरु नक्षत करिा (अिुच्छे द 39)।
2. समाि न्याय को बढावा देिा और गरीबों को मफ्ु त काििू ी सहायता प्रदाि करिा (अिुच्छे द 39 A)।
3. उद्योगों के प्रबंिि में श्रनमकों की भागीदारी सनु िनित करिे के नलए कदम उठािा (अिुच्छे द 43 A)।
4. पयानवरि की रक्षा और सिु ार करिा तथा विों और वन्य जीवों की रक्षा करिा (अिुच्छे द 48 A)।
1978 के 44वें संशोिि अनिनियम िे एक और निदेशक नसद्धातं जोड़ा, नजसके नलए रायय को आय, नस्थनत, सनु विाओ ं और अवसरों
में असमािताओ ं को कम करिे की आवश्यकता है (अिुच्छे द 38)।

2002 के 86 वें सश
ं ोिि अनिनियम िे अिच्ु छे द 45 की नवषय-वस्तु को बदल नदया और प्रारंनभक नशक्षा को अिच्ु छे द 21 A के तहत
एक मौनलक अनिकार बिा नदया। संशोनित निदेश में रायय को छह वषन की आयु परू ी होिे तक सभी बच्चों के नलए प्रारंनभक बाल्यावस्था
देखभाल और नशक्षा प्रदाि करिे की आवश्यकता है।

2011 के 97 वें संशोिि अनिनियम में सहकारी सनमनतयों से संबंनित एक िया निदेशक नसद्धांत जोड़ा गया। इसके नलए रायय को
सहकारी सनमनतयों के स्वैनच्छक गठि, स्वायि कामकाज, लोकतानं िक नियिं ि और पेशेवर प्रबिं ि को बढावा देिे की आवश्यकता है
(अिुच्छे द 43 B)

Q. 31) टिम्िटलटित कथिों पर टिचार करें:


1. िए राययों के गठि से संबंनित काििू को संसद में नवशेष बहुमत से पाररत नकया जाएगा।
2. अिच्ु छे द 368 के तहत संनविाि में संशोिि करके भारतीय क्षेि को नकसी नवदेशी रायय को सौंपा जा सकता है।
3. भारत और दसू रे देश के बीच सीमा नववाद को अिुच्छे द 368 के तहत संनविाि में संशोिि करके सल ु झाया जा सकता है।
उपरोक्त कथिों के आधार पर सही उत्तर का चयि कीटिये ?
a) के वल 1 और 2
b) के वल 2
c) के वल 2 और 3
d) 1, 2 और 3

Q. 31) Solution (b)

सनं विाि (अिुच्छे द 4) में ही यह घोषिा की गई है नक िए राययों के प्रवेश या स्थापिा के नलए बिाए गए कािूि (अिुच्छे द 2 के तहत)
और िए राययों के गठि और मौजदू ा राययों के क्षेिों, सीमाओ ं या िामों में पररवतनि (अिच्ु छे द 3 के तहत) को अिच्ु छे द 368 के तहत
संनविाि के संशोिि के रूप में िहीं मािा जायेगा। इसका मतलब यह है नक इस तरह के कािूिों को सािारि बहुमत से और सािारि
नविायी प्रनिया से पाररत नकया जा सकता है।

क्या नकसी रायय के क्षेिों को कम करिे की संसद की शनक्त (अिुच्छे द 3 के तहत) में भारतीय क्षेि को नकसी नवदेशी देश को सौंपिे की
शनक्त भी शानमल है?

www.iasbaba.com rrs.iasbaba.com Page 23


IASbaba’s Rapid Revision (RaRe) Series Polity Day 1-120 2021
HINDI

1960 में राष्ट्ट्रपनत द्वारा नदए गए एक संदभन में यह प्रश्न सवोच्च न्यायालय के समक्ष परीक्षि के नलए आया। कें द्र सरकार के बेरूबारी संघ
(पनिम बंगाल) के रूप में जािा जािे वाला एक क्षेि पानकस्ताि को सौंपिे के नििनय िे राजिीनतक आंदोलि और नववाद को जन्म नदया
और इस प्रकार राष्ट्ट्रपनत के सदं भन की आवश्यकता पड़ी। सप्रु ीम कोिन िे मािा नक नकसी रायय के क्षेि को कम करिे की ससं द की शनक्त
(अिुच्छे द 3 के तहत) में नकसी नवदेशी देश के नलए भारतीय क्षेि के अनिग्रहि को शानमल िहीं करती है। इसनलए, भारतीय क्षेि को
के वल अिुच्छे द 368 के तहत संनविाि में संशोिि करके एक नवदेशी रायय को सौंपा जा सकता है। ितीजति, 9वां संनविाि संशोिि
अनिनियम (1960) उक्त क्षेि को पानकस्ताि में स्थािातं ररत करिे के नलए अनिनियनमत नकया गया था।

दसू री ओर, 1969 में सप्रु ीम कोिन िे फै सला सिु ाया नक, भारत और दसू रे देश के बीच सीमा नववाद के निपिारे के नलए संवैिानिक
संशोिि की आवश्यकता िहीं है। यह कायनकारी कारन वाई द्वारा नकया जा सकता है क्योंनक इसमें नकसी नवदेशी देश के नलए भारतीय क्षेि
का अनिग्रहि शानमल िहीं है।

Q. 32) 1956 के बाद भारत के िए राज्यों और कें द्र शाटसत प्देशों के टिमाटण का सही कालक्रम क्या है:

a) दादरा और िगर हवेली, पडु ु चेरी, िागालैंड, निपरु ा, नमजोरम


b) पडु ु चरे ी, दादरा और िगर हवेली, नमजोरम, िागालैंड, निपरु ा
c) नमजोरम, निपरु ा, पडु ु चरे ी, दादरा और िगर हवेली, िागालैंड
d) िागालैंड, दादरा और िगर हवेली, नमजोरम, पडु ु चरे ी, निपरु ा
Q. 32) Solution (a)

दादरा और िगर हिेली: पतु गन ानलयों िे 1954 तक इसकी स्वतांत्रता तक इस क्षेि पर शासि नकया। इसके बाद, प्रशासि को 1961 तक
लोगों द्वारा चिु े गए प्रशासक द्वारा चलाया जाता रहा। इसे 10वें सनं विाि सश
ं ोिि अनिनियम, 1961 द्वारा भारत के एक कें द्र शानसत
प्रदेश में पररवनतनत कर नदया गया।

पुडुचेरी: पडु ु चरे ी के क्षेि में भारत में पवू न फ्रांसीसी प्रनतष्ठाि शानमल हैं नजन्हें पडु ु चरे ी, कराईकल, माहे और यिम के िाम से जािा जाता
है। 1954 में फ्रांसीनसयों िे इस क्षेि को भारत को सौंप नदया। इसके बाद, 1962 तक इसको 'अनिग्रनहत क्षेि' के रूप में प्रशानसत नकया
गया, बाद में इसे 14 वें सवं ैिानिक सश ं ोिि अनिनियम द्वारा कें द्र शानसत प्रदेश बिा नदया गया।
िागालैंड: 1963 में, िागा नहल्स और त्यएु िसागं क्षेि को असम रायय से बाहर निकालकर िागालैंड रायय का गठि नकया गया था। ऐसा
नवरोिी िागों के आंदोलि को संतष्टु करिे के नलए नकया गया था। हालााँनक, िागालैंड को भारतीय संघ के 16वें रायय का दजान देिे से
पहले इसे, 1961 तक असम के राययपाल के नियिं ि में रखा गया था।

टत्रपुरा: 1972 में, पवू ोिर भारत के राजिीनतक मािनचि में एक बड़ा बदलाव आया। इस प्रकार, मनिपरु और निपरु ा के दो कें द्र शानसत
प्रदेशों और मेघालय के उप-रायय को रायय का दजान नमला और दो कें द्र शानसत प्रदेश नमजोरम और अरुिाचल प्रदेश (मलू रूप से पवू ोिर
सीमांत एजेंसी-िेफा के रूप में जािा जाता है) अनस्तत्व में आए। इसके साथ ही भारतीय संघ के राययों की संख्या बढकर 21 (मनिपरु
19वें, निपरु ा 20वें और मेघालय 21वें) हो गई।

www.iasbaba.com rrs.iasbaba.com Page 24


IASbaba’s Rapid Revision (RaRe) Series Polity Day 1-120 2021
HINDI

Q. 33) सिंटिधाि के लागू होिे के समय टिम्िटलटित में से कौि से भाग बी (Part B) के राज्य थे?

1. जम्मू और कश्मीर
2. निपरु ा
3. मैसरू
4. मनिपरु
5. राजस्थाि
उपरोक्त कथिों के आधार पर सही उत्तर का चयि कीटिये ?

a) के वल 1, 3 और 4
b) के वल 2 और 4
c) के वल 1, 3 और 5
d) के वल 2, 4 और 5
Q. 33) Solution (c)

1950 में, संनविाि में भारतीय संघ के राययों और क्षेि चार भागों - भाग A, भाग B और भाग C राययों और भाग D क्षेिों में वगीकृ त
थे। कुल नमलाकर, उिकी सख्ं या 29 थी।

भाग A राययों में निनिश भारत के िौ तत्कालीि गविनर प्रातं शानमल थे। भाग B राययों में नविानयकाओ ं के साथ िौ पवू नवती ररयासतें
शानमल थीं। भाग C राययों में निनिश भारत के तत्कालीि मख्ु य आयक्त ु के प्रातं और कुछ पवू नवती ररयासतें शानमल थीं। ये भाग C रायय
(संख्या में सभी 10 में) कें द्रीय रूप से प्रशानसत थे। अंडमाि और निकोबार द्वीप समहू को एकान्त भाग D प्रदेशों के रूप में रखा गया था।

भाग B रायय थे: हैदराबाद, जम्मू और कश्मीर, मध्य भारत, मैसरू , पनियाला और पवू ी पंजाब, राजस्थाि, सौराष्ट्ट्र, िाविकोर- कोचीि,
नवंध्य प्रदेश।

निपरु ा और मनिपरु भाग C के रायय थे।

Q. 34) सिंटिधाि के टिम्िटलटित प्ािधािों में से कौि से सस


िं द के टिशेष बहुमत और आधे राज्य टिधािसभाओ िं की सहमटत
से सिंशोटधत टकये िाते हैं;

1. सघं और राययों की कायनकारी शनक्त का नवस्तार


2. राययों में नविाि पररषदों का उन्मल
ू ि या निमानि
3. िागररकता-अनिग्रहि और समानप्त
4. राष्ट्ट्रपनत का चिु ाव और उसका तरीका
उपरोक्त कथिों के आधार पर सही उत्तर का चयि कीटिये?

a) के वल 1 और 2

www.iasbaba.com rrs.iasbaba.com Page 25


IASbaba’s Rapid Revision (RaRe) Series Polity Day 1-120 2021
HINDI

b) के वल 2 और 3
c) के वल 3 और 4
d) के वल 1 और 4
Q. 34) Solution (d)

सनं विाि के वे प्राविाि जो रायय व्यवस्था के सघं ीय ढाचं े से सबं नं ित हैं, ससं द के नवशेष बहुमत द्वारा तथा एक सािारि बहुमत द्वारा
रायय नविािसभाओ ं के आिे नहस्से की सहमनत से संशोनित नकए जा सकते हैं। यनद एक या कुछ या सभी शेष रायय नविेयक पर कोई
कारन वाई िहीं करते हैं, तो कोई बात िहीं; जैसे ही आिे रायय अपिी सहमनत देते हैं, औपचाररकता परू ी हो जाती है। ऐसी कोई समय सीमा
िहीं है नजसके भीतर राययों को नविेयक पर अपिी सहमनत देिी चानहए।

निम्िनलनखत प्राविािों को इस प्रकार सश


ं ोनित नकया जा सकता है:

● राष्ट्ट्रपनत का चिु ाव और उसका तरीका।


● सघं और राययों की कायनकारी शनक्त की सीमा।
● उच्चतम न्यायालय और उच्च न्यायालय।
● सघं और राययों के बीच नविायी शनक्तयों का नवतरि।
● गडु ् स एंड सनवनसेज िैक्स काउंनसल (वस्तु एवां सेवा कर पररषद)
● सातवीं अिुसचू ी में से कोई भी सचू ी।
● संसद में राययों का प्रनतनिनित्व।
● संनविाि और उसकी प्रनिया में संशोिि करिे की संसद की शनक्त (अिुच्छे द 368 ही)
राययों में नविाि पररषदों के उन्मल
ू ि या निमानि और िागररकता-अनिग्रहि और समानप्त से संबंनित प्राविािों में के वल सािारि बहुमत
के माध्यम से संशोिि नकया जा सकता है।

Q. 35) अिुच्छे द 368 में टिधाटररत सटिं िधाि के सिंशोधि से सिंबटिं धत प्ािधािों के सिंबिंध में टिम्िटलटित कथिों पर टिचार
करें:

1. इस नविेयक को राष्ट्ट्रपनत की अिमु नत से संसद में पेश नकया जाता है।


2. इस नविेयक को प्रत्येक सदि में संसद के नवशेष बहुमत से ही अलग-अलग पाररत नकया जािा चानहए।
3. राष्ट्ट्रपनत को नविेयक पर अपिी सहमनत देिी चानहए। वह ि तो नविेयक पर अपिी सहमनत रोक सकता है और ि ही ससं द के
पिु नवनचार के नलए नविेयक को वापस कर सकता है।
उपरोक्त कथिों के आधार पर सही उत्तर का चयि कीटिये?

a) के वल 1
b) के वल 2 और 3
c) के वल 3
www.iasbaba.com rrs.iasbaba.com Page 26
IASbaba’s Rapid Revision (RaRe) Series Polity Day 1-120 2021
HINDI

d) 1, 2 और 3
Q. 35) Solution (c)

अिुच्छे द 368 दो प्रकार के संशोििों का प्राविाि करता है, अथानत् ससं द के नवशेष बहुमत से और आिे राययों के सािारि बहुमत द्वारा
अिसु मथनि के माध्यम से। लेनकि, कुछ अन्य अिच्ु छे द ससं द के सािारि बहुमत से सनं विाि के कुछ प्राविािों में सश
ं ोिि का प्राविाि
करते हैं, यािी प्रत्येक सदि के उपनस्थत और मतदाि करिे वाले सदस्यों के बहुमत से (सामान्य नविायी प्रनिया के समाि)। उल्लेखिीय
है नक, इि संशोििों को अिुच्छे द 368 के प्रयोजिों के नलए संनविाि का संशोिि िहीं मािा जाता है।

अिुच्छे द 368 में नििानररत संनविाि के संशोिि की प्रनिया इस प्रकार है:

1. संनविाि में संशोिि के वल संसद के नकसी भी सदि में इस उद्देश्य के नलए एक नविेयक पेश करके शरू
ु नकया जा सकता है, ि नक
रायय नविािसभाओ ं में।

2. नविेयक को या तो एक मिं ी या एक निजी सदस्य द्वारा पेश नकया जा सकता है और इसके नलए राष्ट्ट्रपनत की पवू न अिुमनत की
आवश्यकता िहीं होती है।

3. नविेयक को प्रत्येक सदि में नवशेष बहुमत से पाररत नकया जािा चानहए, अथानत सदि की कुल सदस्यता का बहुमत तथा सदि के
उपनस्थत और मतदाि करिे वाले सदस्यों के दो-नतहाई बहुमत से।

4. प्रत्येक सदि को अलग से नविेयक पाररत करिा होगा। दोिों सदिों के बीच असहमनत की नस्थनत में, नविेयक पर नवचार-नवमशन और
पाररत होिे के उद्देश्य से दोिों सदिों की संयक्त
ु बैठक आयोनजत करिे का कोई प्राविाि िहीं है।
5. यनद नविेयक संनविाि के संघीय प्राविािों में संशोिि करिा चाहता है, तो इसे आिे राययों के नविािमंडलों द्वारा सािारि बहुमत
से, यािी सदि के उपनस्थत और मतदाि करिे वाले सदस्यों के बहुमत से भी अिुमोनदत नकया जािा चानहए।

6. संसद के दोिों सदिों द्वारा नवनिवत पाररत होिे और रायय नविािसभाओ ं द्वारा अिमु ोनदत होिे के बाद, जहां आवश्यक हो, नविेयक
को राष्ट्ट्रपनत की सहमनत के नलए प्रस्ततु नकया जाता है।

7. राष्ट्ट्रपनत को नविेयक पर अपिी सहमनत देिी चानहए। वह ि तो नविेयक पर अपिी सहमनत रोक सकता है और ि ही संसद के
पिु नवनचार के नलए नविेयक को वापस कर सकता है।

8. राष्ट्ट्रपनत की सहमनत के बाद, नविेयक एक अनिनियम (यािी, एक सवं ैिानिक सश


ं ोिि अनिनियम) बि जाता है और सनं विाि
अनिनियम की शतों के अिुसार सश ं ोनित होता है।
Q. 36) टिम्िटलटित में से कौि सिंटिधाि की मूल सिंरचिा के तत्ि हैं/हैं?

1. न्यानयक समीक्षा
2. कािूि का शासि
3. स्वतिं और निष्ट्पक्ष चिु ाव
www.iasbaba.com rrs.iasbaba.com Page 27
IASbaba’s Rapid Revision (RaRe) Series Polity Day 1-120 2021
HINDI

उपरोक्त कथिों के आधार पर सही उत्तर का चयि कीटिये ?

a) के वल 1 और 2
b) के वल 2 और 3
c) के वल 1 और 3
d) 1, 2 और 3
Q. 36) Solution (d)

नवनभन्ि नििनयों से, निम्िनलनखत सनं विाि की' बुनियादी नवशेषताओ ं 'या संनविाि की' मल
ू संरचिा' के तत्वों के रूप में उभरे हैं:

● सनं विाि की सवोच्चता


● भारतीय रायय व्यवस्था की संप्रभ,ु लोकतांनिक और गितांनिक प्रकृ नत
● संनविाि का िमननिरपेक्ष चररि
● नविानयका, कायनपानलका और न्यायपानलका के बीच शनक्तयों का पृथक्करि
● संनविाि का संघीय चररि
● राष्ट्ट्र की एकता और अखडं ता
● कल्यािकारी रायय (सामानजक-आनथनक न्याय)
● न्यानयक समीक्षा
● व्यनक्त की स्वतंिता और गररमा
● संसदीय प्रिाली
● कािूि का शासि
● मौनलक अनिकारों और निदेशक नसद्धांतों के बीच सामंजस्य और संतल ु ि
● समािता का नसद्धातं
● स्वतिं और निष्ट्पक्ष चिु ाव
● न्यायपानलका की स्वतिं ता
● संनविाि में संशोिि के नलए संसद की सीनमत शनक्त
● न्याय तक प्रभावी पहुचं
● मौनलक अनिकारों के अंतनिननहत नसद्धांत (या सार)
● अिुच्छे द 32, 136, 141 और 142 के तहत सप्रु ीम कोिन की शनक्तयां
● अिुच्छे द 226 और 227 के तहत उच्च न्यायालयों की शनक्तयां।

Q. 37) एक राष्ट्रीय पा ी के रूप में मान्यता प्ाि करिे के टलए एक पा ी को टकि शतों को पूरा करिा चाटहए:

www.iasbaba.com rrs.iasbaba.com Page 28


IASbaba’s Rapid Revision (RaRe) Series Polity Day 1-120 2021
HINDI

1. इसे लोकसभा के आम चिु ाव में नकन्हीं छह या अनिक राययों में मतदाि नकये गए वैि मतों का चार प्रनतशत सरु नक्षत करिा
चानहए।
2. इसे कम से कम तीि राययों में एक रायय पािी के रूप में मान्यता प्ाप्त होिी चानहए।
उपरोक्त कथिों के आधार पर सही उत्तर का चयि कीटिये ?

a) के वल 1
b) के वल 1 और 2
c) दोिों 1 और 2
d) ि तो 1 और ि ही 2
Q. 37) Solution (d)

एक पािी को राष्ट्ट्रीय पािी के रूप में मान्यता दी जाती है यनद निम्िनलनखत में से कोई भी शतन परू ी होती है:

1. यनद यह लोकसभा या नविाि सभा के आम चिु ाव में नकन्हीं चार या अनिक राययों में डाले गए वैि मतों का छह प्रनतशत हानसल
करता है; और, इसके अलावा, यह नकसी भी रायय या राययों से लोकसभा में चार सीिें जीतती है; या

2. यनद वह आम चिु ाव में लोकसभा में दो प्रनतशत सीिें जीतती है; और ये उम्मीदवार तीि राययों से चिु े जाते हैं; या

3. यनद इसे चार राययों में एक रायय पािी के रूप में मान्यता प्राप्त है।

Q. 38) टिम्िटलटित कथिों पर टिचार करें:

1. नकसी रायय में मनं िपररषद में मख्ु यमिं ी सनहत मनं ियों की कुल सख्ं या 15 से कम िहीं होगी।
2. एक सदि का मिोिीत सदस्य सदि के सदस्य होिे के नलए अयोग्य हो जाता है यनद वह सदि में अपिा स्थाि ग्रहि करिे की
तारीख से छह महीिे के भीतर नकसी राजिीनतक दल में शानमल हो जाता है।
3. दलबदल से उत्पन्ि होिे वाली अयोग्यता के सबं िं में सदि के पीठासीि अनिकारी का नििनय अनं तम होता है और इसे नकसी
भी अदालत में चिु ौती िहीं दी जा सकती है।
उपरोक्त कथिों के आधार पर सही उत्तर का चयि कीटिये ?

a) के वल 1 और 2
b) के वल 2 और 3
c) 1, 2 और 3
d) उपरोक्त से कोई िहीं
Q. 38) Solution (d)

नकसी रायय में मंनिपररषद में मख्ु यमिं ी सनहत मंनियों की कुल संख्या उस रायय की नविाि सभा की कुल संख्या के 15 प्रनतशत से अनिक
िहीं होिी चानहए। लेनकि, नकसी रायय में मख्ु यमिं ी सनहत मनं ियों की सख्ं या 12 से कम िहीं होिी चानहए।

www.iasbaba.com rrs.iasbaba.com Page 29


IASbaba’s Rapid Revision (RaRe) Series Polity Day 1-120 2021
HINDI

एक सदि का मिोिीत सदस्य सदि का सदस्य होिे के नलए अयोग्य हो जाता है यनद वह सदि में अपिा स्थाि ग्रहि करिे की तारीख से
छह महीिे की समानप्त के बाद नकसी राजिीनतक दल में शानमल हो जाता है। इसका मतलब है नक वह इस अयोग्यता को आमनं ित नकए
नबिा सदि में अपिा स्थाि ग्रहि करिे के छह महीिे के भीतर नकसी भी राजिीनतक दल में शानमल हो सकता है।

दलबदल से उत्पन्ि अयोग्यता के सबं ंि में कोई प्रश्न सदि के पीठासीि अनिकारी द्वारा तय नकया जाएगा। मल
ू रूप से, अनिनियम में यह
प्राविाि है नक पीठासीि अनिकारी का नििनय अंनतम है और नकसी भी अदालत में इसपर प्रश्न िहीं नकया जा सकता है। हालााँनक, नकहोतो
होलोहि मामले (1993) में, सवोच्च न्यायालय िे इस प्राविाि को इस आिार पर असंवैिानिक घोनषत नकया नक वह सवोच्च न्यायालय
और उच्च न्यायालयों के अनिकार क्षेि को छीििा चाहता है। यह मािा गया नक पीठासीि अनिकारी दसवीं अिुसचू ी के तहत एक प्रश्न
का नििनय करते समय एक न्यायानिकरि के रूप में कायन करता है। इसनलए, नकसी भी अन्य न्यायानिकरि की तरह उसका नििनय
दभु ानविा, नवकृ नत आनद के आिार पर न्यानयक समीक्षा के अिीि है। लेनकि, अदालत िे इस तकन को खाररज कर नदया नक पीठासीि
अनिकारी में नििनयात्मक शनक्तयों का निनहत होिा राजिीनतक पवू ानग्रह के आिार पर अपिे आप में अमान्य है ।

Q. 39) निम्िनिनित कथिों पर नवचार करें:

1. प्त्येक कें द्र शानसत प्देश का प्शासि राष्ट्रपनत, स्वयां द्वारा नियक्त
ु प्शासक के माध्यम से करता है।
2. कें द्र शानसत प्देशों में नवधािसभाओ ां की स्थापिा उि पर राष्ट्रपनत और सांसद के सवोच्च नियांत्रण पर कुछ प्नतबांध लगाती है।
उपरोक्त में से कौि सा/से कथि सही है/हैं?

a) के वल 1
b) के वल 2
c) दोिों 1 और 2
d) ि तो 1 और ि ही 2
Q. 39) Solution (a)

Basic Information:
• प्त्येक कें द्र शानसत प्देश का प्शासि राष्ट्रपनत स्वयां द्वारा नियक्त
ु प्शासक के माध्यम से करता है। एक कें द्र शानसत प्देश का
प्शासक राष्ट्रपनत का एजेंट होता है, ि नक राज्यपाल की तरह राज्य का मनु खया।
• राष्ट्रपनत एक प्शासक के पद को निनदगष्ट कर सकता है; यह उपराज्यपाल या मख्ु य आयक्त
ु या प्शासक हो सकता है।
• राष्ट्रपनत नकसी राज्य के राज्यपाल को निकटवती कें द्र शानसत प्देश का प्शासक भी नियक्त
ु कर सकता है। उस क्षमता में,
राज्यपाल को अपिे मनां त्रपररषद से स्वतत्रां रूप से कायग करिा होता है।
• कें द्र शानसत प्देशों के नलए सांसद तीि सनू चयों (राज्य सचू ी सनहत) के नकसी भी नवषय पर काििू बिा सकती है।
• राष्ट्रपनत द्वारा बिाए गए नवनियम में सांसद के अनधनियम के समाि बल और प्भाव होता है तथा इि कें द्र शानसत प्देशों के
सबां धां में ससां द के नकसी भी अनधनियम को निरस्त या सश
ां ोनधत भी कर सकता है।

www.iasbaba.com rrs.iasbaba.com Page 30


IASbaba’s Rapid Revision (RaRe) Series Polity Day 1-120 2021
HINDI

िोट: कें द्र शानसत प्देशों में नवधािसभाओ ां और मनां त्रपररषद जैसी प्नतनिनध सांस्थाओ ां की स्थापिा से उि पर राष्ट्रपनत और सांसद का
सवोच्च नियांत्रण कम िहीं होता है।

Q.40) िागररकता (सिंशोधि) अटधटियम, 2019 के सिंबिंध में टिम्िटलटित कथिों पर टिचार करें:

1. एक व्यनक्त पंजीकरि द्वारा िागररकता के नलए आवेदि कर सकता है यनद वह एक वषन के नलए भारत में रहता है और यनद
उसके माता-नपता में से कोई एक पवू न भारतीय िागररक है।
2. अफगानिस्ताि, बांग्लादेश और पानकस्ताि के नहदं ,ू नसख, बौद्ध, जैि, पारसी और ईसाई िागररकता के नलए आवेदि करिे से
पहले कम से कम 5 साल के नलए भारत में रहिे पर देशीयकरि/ प्ाकृ नतकरण (naturalisation) द्वारा िागररकता के नलए
आवेदि कर सकते हैं।
उपरोक्त कथिों के आधार पर सही उत्तर का चयि कीटिये ?

a) के वल 1
b) के वल 2
c) 1 और 2 दोिों
d) ि तो 1 और ि ही 2
Q. 40) Solution (c)

िागररकता (संशोिि) अनिनियम, 2019 िागररकता अनिनियम, 1955 में संशोिि करिा चाहता है ।

िागररकता अनिनियम, 1955 नवनभन्ि तरीके प्रदाि करता है नजसमें िागररकता प्राप्त की जा सकती है। यह जन्म, वंश, पंजीकरि,
प्राकृ नतककरि और भारत में क्षेि को शानमल करके िागररकता प्रदाि करता है।

संशोिि अनिनियम के प्रमख


ु प्राविाि:

● नविेयक में अनिनियम में सश ं ोिि करते हुए यह प्राविाि नकया गया है नक अफगानिस्ताि, बाग्ं लादेश और पानकस्ताि के नहदं ,ू
नसख, बौद्ध, जैि, पारसी और ईसाई, नजन्होंिे 31 नदसंबर, 2014 को या उससे पहले भारत में प्रवेश नकया था, उन्हें अवैि
प्रवानसयों के रूप में िहीं मािा जाएगा।
● प्राकृ नतककरि द्वारा िागररकता प्राप्त करिे के नलए, योग्यताओ ं में से एक यह है नक िागररकता के नलए आवेदि करिे से पहले
व्यनक्त को भारत में निवास करिा चानहए या कम से कम 11 वषों तक कें द्र सरकार की सेवा में होिा चानहए। यह नविेयक इस
योग्यता के संबंि में अफगानिस्ताि, बांग्लादेश और पानकस्ताि के नहदं ओ ु ,ं नसखों, बौद्धों, जैनियों, पारनसयों और ईसाइयों के
नलए एक अपवाद बिाता है। व्यनक्तयों के इि समहू ों के नलए, 11 वषन की आवश्यकता को घिाकर पाचं वषन कर नदया जाएगा।
● अवैि प्रवानसयों के नलए िागररकता पर ये प्राविाि संनविाि की छठी अिुसचू ी में शानमल असम, मेघालय, नमजोरम और
निपरु ा के आनदवासी क्षेिों पर लागू िहीं होंगे। इि आनदवासी क्षेिों में काबी आगं लोंग (असम में), गारो नहल्स (मेघालय में),
चकमा नजला (नमजोरम में) और निपरु ा जिजातीय क्षेि नजला शानमल हैं।
● इसके अलावा, यह बंगाल ईस्ििन फ्रंनियर रेगल ु ेशि, 1873 के तहत अनिसनू चत "इिर लाइि" क्षेिों पर लागू िहीं होगा। इि
क्षेिों में, भारतीयों द्वारा यािा 'इिर लाइि परनमि' के माध्यम से नियनं ित होते हैं।

www.iasbaba.com rrs.iasbaba.com Page 31


IASbaba’s Rapid Revision (RaRe) Series Polity Day 1-120 2021
HINDI

Q. 41) निम्िनिनित को ध्याि में रिते हुए:


1. बहुमत दल का शासि
2. दोहरी कायगकारी
3. शनक्तयों का पणू ग पृथक्करण
4. निचले सदि का नवघटि
उपरोक्त में से कौि सी सरकार के संसदीय स्वरूप की नवशेषताएं हैं?
a) के वल 1, 2 और 3 3
b) के वल 2, 3 और 4
c) के वल 1, 2 और 4
d) 1, 2, 3 और 4
Q. 41) Solution (c)
भारत का सांनवधाि कें द्र और राज्यों दोिों में सरकार के सांसदीय स्वरूप का प्ावधाि करता है। अिुच्छे द 74 और 75 कें द्र में सांसदीय
प्णाली से सांबांनधत है और अिुच्छे द 163 और 164 राज्यों में सांसदीय प्णाली से सांबांनधत है।
भारत में सांसदीय सरकार की नवशेषताएां या नसद्धाांत हैं:
• बहुमत दल का शासि: वह राजिीनतक दल जो लोकसभा में बहुमत प्ाप्त करता है, सरकार बिाता है। उस पाटी के िेता को
राष्ट्रपनत द्वारा प्धाि मांत्री के रूप में नियक्त
ु नकया जाता है; अन्य मांनत्रयों की नियनु क्त राष्ट्रपनत द्वारा प्धािमांत्री की सलाह पर
की जाती है। हालााँनक, जब नकसी एक दल को बहुमत िहीं नमलता है, तो राष्ट्रपनत द्वारा सरकार बिािे के नलए पानटगयों के
गठबधां ि को आमनां त्रत नकया जा सकता है।
• दोहरी सदस्यता: मत्रां ी नवधानयका और कायगपानलका दोिों के सदस्य होते हैं। इसका अथग यह हुआ नक कोई व्यनक्त ससां द का
सदस्य हुए नबिा मत्रां ी िहीं हो सकता। सनां वधाि में कहा गया है नक एक मत्रां ी जो लगातार छह महीिे तक ससां द का सदस्य िहीं
होता है, वह मत्रां ी िहीं रहता है।
• निचले सदि का नवघटि: ससां द के निचले सदि (लोकसभा) को राष्ट्रपनत द्वारा प्धािमत्रां ी की नसफाररश पर भगां नकया जा
सकता है। दसू रे शब्दों में, प्धाि मत्रां ी राष्ट्रपनत को अपिे कायगकाल की समानप्त से पहले लोकसभा को भगां करिे और िए चिु ाव
करािे की सलाह दे सकता है। इसका अथग है नक कायगपानलका को ससां दीय प्णाली में नवधानयका को भगां करिे का अनधकार
प्ाप्त है।
शनक्तयों का पणू ग पृथक्करण सरकार के राष्ट्रपनत स्वरूप की नवशेषता है। शनक्तयों के पृथक्करण का नसद्धाांत अमेररकी राष्ट्रपनत प्णाली
का आधार है। सरकार की नवधायी, कायगकारी और न्यानयक शनक्तयाां सरकार के तीिों अांग में स्वतांत्र रूप से पृथक और निनहत होती हैं।

Q. 42) निम्िनिनित में से कौि सरकार के संसदीय स्वरूप के गुण हैं/हैं?

www.iasbaba.com rrs.iasbaba.com Page 32


IASbaba’s Rapid Revision (RaRe) Series Polity Day 1-120 2021
HINDI

1. नवधानयका और कायगपानलका के बीच सामांजस्य


2. निरांकुशता पर रोक (Prevents despotism)
3. िीनतयों में निनितता (Definiteness in policies)
िीचे नदए गए कूटों में से सही उत्तर चुनिए:
a) के वल 1 और 2
b) के वल 2 और 3
c) के वल 1 और 3
d) 1, 2 और 3
Q. 42) Solution (a)
सरकार की सांसदीय प्णाली के निम्िनलनखत गणु हैं:
• नवधानयका और कायगपानलका के बीच सामांजस्य: सांसदीय प्णाली का सबसे बडा लाभ यह है नक यह सरकार के नवधायी और
कायगकारी अगां ों के बीच सामांजस्यपणू ग सांबांध और सहयोग सनु िनित करती है। कायगपानलका नवधानयका का एक नहस्सा है और
दोिों काम पर परस्पर-निभगर हैं। ितीजति, दोिों अांगों के बीच नववाद और सांघषग की गांजु ाइश कम होती है।
• निरांकुशता पर रोक: इस प्णाली के तहत, कायगकारी अनधकार व्यनक्तयों के एक समहू (मांनत्रपररषद) में निनहत होता है, ि नक
नकसी एक व्यनक्त में। सत्ता का यह फै लाव कायगपानलका की तािाशाही प्वृनत्तयों को रोकता है। इसके अलावा, कायगपानलका
सांसद के प्नत उत्तरदायी होती है और उसे अनवश्वास प्स्ताव द्वारा हटाया जा सकता है।
ससां दीय सरकार के दोषों में से एक यह है नक िीनतयों की निरांतरता िहीं होती है। सांसदीय प्णाली दीघगकानलक िीनतयों के निमागण और
कायागन्वयि के नलए मागगदशगक िहीं है। इसका कारण है सरकार के कायगकाल की अनिनितता। सत्तारूढ़ दल में बदलाव के बाद आमतौर
पर सरकार की िीनतयों में बदलाव होता है।

Q. 43) निम्िनिनित में से कौि संसदीय सरकार का आधारभूत नसद्ांत है?


a) नस्थर सरकार
b) नवशेषज्ञों द्वारा सरकार
c) सामनू हक उत्तरदानयत्व
d) इच्छुक या उत्सक
ु वैकनपपक सरकार (Ready alternative government)
Q. 43) Solution (c)
सामनू हक उत्तरदानयत्व सांसदीय सरकार का मल ू नसद्धाांत है। मांत्री सामनू हक रूप से सांसद के प्नत और नवशेष रूप से लोकसभा के प्नत
उत्तरदायी होते हैं (अिुच्छे द 75)। वे एक टीम के रूप में कायग करते हैं, और एक साथ तैरते और डूबते हैं। सामनू हक उत्तरदानयत्व के

www.iasbaba.com rrs.iasbaba.com Page 33


IASbaba’s Rapid Revision (RaRe) Series Polity Day 1-120 2021
HINDI

नसद्धाांत का तात्पयग है नक लोकसभा अनवश्वास प्स्ताव पाररत करके मत्रां ालय (यािी, प्धाि मांत्री की अध्यक्षता में मांनत्रपररषद) को पद से
हटा सकती है।

Q. 44) निम्िनिनित में से कौि-सी संनवधाि की संघीय नवशेषताएँ (federal features) हैं?
1. लचीला सनां वधाि
2. नलनखत सनां वधाि
3. सनां वधाि की सवोच्चता
4. राज्य अनविाशी िहीं (States not indestructible)
िीचे नदए गए कूटों में से सही उत्तर चुनिए:
a) के वल 1 और 2
b) के वल 2 और 3
c) के वल 3 और 4
d) के वल 1 और 4
Q. 44) Solution (b)
नलनखत सांनवधाि और सांनवधाि की सवोच्चता सांनवधाि की सांघीय नवशेषताएां हैं जबनक सांनवधाि का लचीलापि और राज्यों की
अनविाशीता (Non indestructibility) सांनवधाि की एकात्मक नवशेषताएां हैं।
सांनवधाि की सांघीय नवशेषताएां:
• नलनखत सांनवधाि: भारतीय सांनवधाि ि के वल एक नलनखत दस्तावेज है बनपक दनु िया का सबसे लांबा सनां वधाि भी है। मल ू
रूप से, इसमें एक प्स्ताविा, 395 अिुच्छे द (22 भागों में नवभानजत) और 8 अिुसनू चयााँ शानमल थीं। वतगमाि में (2019),
इसमें एक प्स्ताविा, लगभग 470 अिुच्छे द (25 भागों में नवभानजत) और 12 अिुसनू चयााँ शानमल हैं। यह सांरचिा को निनदगष्ट
करता है। कें द्र और राज्य दोिों सरकारों के सांगठि, शनक्तयाां और कायग और उि सीमाओ ां को निधागररत करता है नजिके भीतर
उन्हें काम करिा चानहए। इस प्कार, यहााँ दोिों के बीच गलतफहमी और असहमनत से बचा जाता है।
• सनां वधाि की सवोच्चता: भारतीय सांनवधाि देश का सवोच्च (या सवोच्च) कािूि है। कें द्र और राज्यों द्वारा बिाए गए कािूिों
को इसके प्ावधािों के अिुरूप होिा चानहए। अन्यथा, उन्हें सवोच्च न्यायालय या उच्च न्यायालयों द्वारा न्यानयक समीक्षा की
अपिी शनक्त के माध्यम से अमान्य घोनषत नकया जा सकता है। इस प्कार, दोिों स्तरों पर सरकार के अांगों (नवधायी, कायगकारी
और न्यानयक) को सांनवधाि द्वारा निधागररत क्षेत्रानधकार के भीतर काम करिा चानहए।
सांनवधाि की एकात्मक नवशेषताएां:
• सनां वधाि का लचीलापि: सांनवधाि सांशोधि की प्निया अन्य सांघों की तल
ु िा में कम कठोर है। सांनवधाि के अनधकाांश भाग
को सांसद की एकतरफा कारग वाई द्वारा या तो साधारण बहुमत से या नवशेष बहुमत से सांशोनधत नकया जा सकता है। इसके

www.iasbaba.com rrs.iasbaba.com Page 34


IASbaba’s Rapid Revision (RaRe) Series Polity Day 1-120 2021
HINDI

अलावा, सांनवधाि में सांशोधि शरू


ु करिे की शनक्त के वल कें द्र के पास है। अमेररका में राज्य सांनवधाि में सांशोधि का प्स्ताव
भी दे सकते हैं।
• राज्य अनविाशी िहीं: अन्य सांघों के नवपरीत, भारत के राज्यों को क्षेत्रीय अखांडता का कोई अनधकार िहीं है। सांसद एकतरफा
कारग वाई से नकसी भी राज्य के क्षेत्र, सीमाओ ां या िाम को बदल सकती है। इसके अलावा, इसके नलए के वल एक साधारण
बहुमत की आवश्यकता होती है ि नक नवशेष बहुमत की। इसनलए, भारतीय सघां "नविाशकारी राज्यों का एक अनविाशी सघां "
है। दसू री ओर, अमेररकी सघां को "अनविाशी राज्यों का एक अनविाशी सघां " के रूप में वनणगत नकया गया है।

Q. 45) निम्िनिनित को ध्याि में रिते हुए:


1. आपातकालीि प्ावधाि
2. एकीकृ त न्यायपानलका
3. स्वतांत्र न्यायपानलका
4. अनखल भारतीय सेवाएां
5. नद्वसदिीय नवधािमांडल
उपरोक्त में से कौि सनं वधाि की एकात्मक नवशेषताएं (unitary features) हैं?
a) के वल 1, 2 और 3 3
b) के वल 2, 3 और 5
c) के वल 1, 2 और 4 4
d) के वल 2, 3, 4 और 5
Q. 45) Solution (c)
सनां वधाि की एकात्मक नवशेषताए:ां
• आपातकालीि प्ावधाि: सांनवधाि तीि प्कार की आपात नस्थनतयों को निधागररत करता है-राष्ट्रीय, राज्य और नवत्तीय। एक
आपात नस्थनत के दौराि, कें द्र सरकार परू ी तरह से शनक्तशाली हो जाती है और राज्य कें द्र के पणू ग नियत्रां ण में आ जाते हैं। यह
सांनवधाि के औपचाररक सांशोधि के नबिा सांघीय ढाचां े को एकात्मक में बदल देता है। इस तरह का पररवतगि नकसी अन्य सांघ
में िहीं नमलता है।
• एकीकृ त न्यायपानलका: भारतीय सांनवधाि िे एक एकीकृ त न्यानयक प्णाली की स्थापिा की है नजसके शीषग पर सवोच्च
न्यायालय और उसके िीचे राज्य उच्च न्यायालय हैं। अदालतों की यह एकल प्णाली कें द्रीय काििू ों के साथ-साथ राज्य के
कािूिों को भी लागू करती है। दसू री ओर, अमेररका में, अदालतों की दोहरी प्णाली है नजसके तहत सांघीय कािूि सांघीय
न्यायपानलका द्वारा और राज्य के कािूिों को राज्य न्यायपानलका द्वारा लागू नकया जाता है।
• अनखल भारतीय सेवाएां: अमेररका में, सांघीय सरकार और राज्य सरकारों की अपिी अलग सावगजनिक सेवाएां हैं। भारत में भी
कें द्र और राज्यों की अलग-अलग सावगजनिक सेवाएां हैं। लेनकि, इसके अलावा, अनखल भारतीय सेवाएां (आईएएस,
आईपीएस और आईएफएस) हैं जो कें द्र और राज्यों दोिों के नलए समाि हैं। इि सेवाओ ां के सदस्यों की भती और प्नशक्षण

www.iasbaba.com rrs.iasbaba.com Page 35


IASbaba’s Rapid Revision (RaRe) Series Polity Day 1-120 2021
HINDI

कें द्र द्वारा नकया जाता है नजसका उि पर अांनतम नियांत्रण भी होता है। इस प्कार, ये सेवाएां सांनवधाि के तहत सांघवाद के नसद्धाांत
का उपलांघि करती हैं।
सांनवधाि की सांघीय नवशेषताएां:
• स्वतांत्र न्यायपानलका: सांनवधाि दो उद्देश्यों के नलए सवोच्च न्यायालय की अध्यक्षता में एक स्वतांत्र न्यायपानलका की स्थापिा
करता है: एक, न्यानयक समीक्षा की शनक्त का प्योग करके सांनवधाि की सवोच्चता की रक्षा करिा; और दो, कें द्र और राज्यों
के बीच या राज्यों के बीच नववादों को निपटािे के नलए। सांनवधाि में न्यायपानलका को सरकार से स्वतांत्र बिािे के नलए
न्यायाधीशों के कायगकाल की सरु क्षा, निनित सेवा शतों आनद जैसे नवनभन्ि उपाय शानमल हैं।
• नद्वसदिीयवाद: सांनवधाि एक नद्वसदिीय नवधानयका का प्ावधाि करता है नजसमें एक उच्च सदि (राज्य सभा) और एक
निचला सदि (लोकसभा) शानमल है। राज्यसभा भारतीय सांघ के राज्यों का प्नतनिनधत्व करती है, जबनक लोकसभा समग्र
रूप से भारत के लोगों का प्नतनिनधत्व करती है। कें द्र के अिुनचत हस्तक्षेप के नखलाफ राज्यों के नहतों की रक्षा करके सांघीय
सांतुलि बिाए रखिे के नलए राज्य सभा (भले ही कम शनक्तशाली सदि) की आवश्यकता होती है।

Q. 46) भारतीय राजिीनतक व्यवस्था के कामकाज में निम्िनिनित में से कौि सी प्रवृनत्त संघवाद की भाविा को दशााती है:
1. राज्यों के बीच क्षेत्रीय नववाद
2. िदी जल बांटवारे को लेकर राज्यों के बीच नववाद
3. क्षेत्रीय दलों का उदय और राज्यों में उिका सत्ता में आिा
िीचे नदए गए कूटों में से सही उत्तर चुनिए:
a) के वल 1 और 2
b) के वल 2 और 3
c) के वल 1 और 3
d) 1, 2 और 3
Q. 46) Solution (d)
भारतीय राजिीनतक व्यवस्था के कामकाज में निम्िनलनखत रुझाि इसकी सांघीय भाविा को दशागते हैं::
i.राज्यों के बीच क्षेत्रीय नववाद, उदाहरण के नलए, बेलगाम को लेकर महाराष्ट्र और किागटक के बीच।
ii. िदी के पािी के बांटवारे को लेकर राज्यों के बीच नववाद, उदाहरण के नलए, कावेरी जल को लेकर किागटक और तनमलिाडु के बीच।
iii. आांध्र प्देश, तनमलिाडु आनद राज्यों में क्षेत्रीय दलों का उदय और उिका सत्ता में आिा।
iv. क्षेत्रीय आकाांक्षाओ ां को परू ा करिे के नलए िए राज्यों का निमागण, उदाहरण के नलए नमजोरम या झारखांड।
v. राज्यों की नवकास सांबांधी जरूरतों को परू ा करिे के नलए कें द्र से अनधक नवत्तीय अिदु ाि की माांग।

www.iasbaba.com rrs.iasbaba.com Page 36


IASbaba’s Rapid Revision (RaRe) Series Polity Day 1-120 2021
HINDI

vi. राज्यों द्वारा स्वायत्तता का दावा और कें द्र के हस्तक्षेप का उिका प्नतरोध।
vii. कें द्र द्वारा अिुच्छे द 356 (राज्यों में राष्ट्रपनत शासि) के उपयोग पर सप्ु ीम कोटग द्वारा कई प्नियात्मक सीमाएां लागू करिा।

Q. 47) राष्ट्रीय आपातकाि के सबं ध


ं में निम्िनिनित कथिों पर नवचार करें:
1. राष्ट्रपनत के वल प्धािमत्रां ी की सलाह पर ही राष्ट्रीय आपातकाल की घोषणा कर सकता है।
2. राष्ट्रीय आपातकाल की घोषणा को जारी होिे की तारीख से दो महीिे के भीतर सांसद के दोिों सदिों द्वारा अिुमोनदत नकया
जािा चानहए।
3. सांसद के दोिों सदिों द्वारा अिमु ोनदत होिे पर, राष्ट्रीय आपातकाल छह महीिे तक जारी रहता है।
उपरोक्त में से कौि सा कथि सही हैं?
a) के वल 1
b) के वल 1 और 2
c) के वल 3
d) 1, 2 और 3
Q. 47) Solution (c)
अिुच्छे द 352 के तहत, राष्ट्रपनत राष्ट्रीय आपातकाल की घोषणा कर सकते हैं जब भारत या उसके एक नहस्से की सरु क्षा को यद्ध
ु या
बाहरी आिमण या सशस्त्र नवद्रोह से खतरा हो।
राष्ट्रपनत, हालानां क, कै नबिेट से नलनखत नसफाररश प्ाप्त करिे के बाद ही राष्ट्रीय आपातकाल की घोषणा कर सकते हैं। इसका अथग यह
है नक के वल प्धािमांत्री की सलाह पर ही िहीं, बनपक कै नबिेट की सहमनत से ही आपातकाल की घोषणा की जा सकती है। 1975 में,
तत्कालीि प्धाि मांत्री, इनां दरा गाांधी िे राष्ट्रपनत को अपिे मांनत्रमांडल से परामशग नकए नबिा आपातकाल की घोषणा करिे की सलाह दी
थी। घोषणा के बाद कै नबिेट को सनू चत नकया गया था, क्योंनक यह एक नवश्वास के रूप में नकया गया था। 1978 के 44वें सांशोधि
अनधनियम िे इस सांबांध में के वल प्धािमत्रां ी द्वारा निणगय लेिे की नकसी भी सांभाविा को समाप्त करिे के नलए इस सरु क्षा उपाय की
शरुु आत की।
आपातकाल की उद्घोषणा को ससां द के दोिों सदिों द्वारा इसके जारी होिे की तारीख से एक महीिे के भीतर अिमु ोनदत नकया जािा
चानहए। मल
ू रूप से, ससां द द्वारा अिमु ोदि के नलए अिमु त अवनध दो महीिे थी, लेनकि 1978 के 44वें सश
ां ोधि अनधनियम द्वारा इसे
कम कर नदया गया था।
यनद सांसद के दोिों सदिों द्वारा अिमु ोनदत नकया जाता है, तो आपातकाल छह महीिे तक जारी रहता है, और इसे हर छह महीिे के नलए
सांसद की मांजरू ी के साथ अनिनित काल तक बढ़ाया जा सकता है। समय-समय पर सांसदीय अिमु ोदि के नलए यह प्ावधाि भी 44वें
सांशोधि अनधनियम 1978 द्वारा जोडा गया था।

www.iasbaba.com rrs.iasbaba.com Page 37


IASbaba’s Rapid Revision (RaRe) Series Polity Day 1-120 2021
HINDI

Q. 48) राष्ट्रीय आपातकाि का भारतीय राजिीनतक व्यवस्था पर क्या प्रभाव है/हैं?


1. राज्य सरकारों को निलांनबत कर नदया जाता है और उन्हें कें द्र के पणू ग नियांत्रण में लाया जाता है।
2. सांनवधाि सांघीय के बजाय एकात्मक हो जाता है।
3. राष्ट्रपनत कें द्र से राज्यों को नवत्त के हस्तातां रण को या तो कम या रद्द कर सकता है।
िीचे नदए गए कूटों में से सही उत्तर चुनिए:
a) के वल 1 और 2
b) के वल 2 और 3
c) के वल 1 और 3
d) 1, 2 और 3
Q. 48) Solution (b)
जबनक आपातकाल की घोषणा लागू है, कें द्र-राज्य सांबांधों के सामान्य तािे-बािे में एक बुनियादी बदलाव आता है:
एक राष्ट्रीय आपातकाल के दौराि, कें द्र की कायगकारी शनक्त नकसी भी राज्य को उस तरीके के बारे में निदेश देिे तक नवस्ताररत है नजसमें
उसकी कायगकारी शनक्त का प्योग नकया जािा है। सामान्य समय में, कें द्र नकसी राज्य को कुछ नवनशष्ट मामलों पर ही कायगकारी निदेश दे
सकता है। हालानां क, एक राष्ट्रीय आपातकाल के दौराि, कें द्र नकसी राज्य को 'नकसी भी' मामले पर कायगकारी निदेश देिे का हकदार हो
जाता है। इस प्कार, राज्य सरकारों को कें द्र के पणू ग नियांत्रण में लाया जाता है, हालाांनक उन्हें निलांनबत िहीं नकया जाता है।
राष्ट्रीय आपातकाल के दौराि, सांसद को राज्य सचू ी में उनपलनखत नकसी भी नवषय पर काििू बिािे का अनधकार नमल जाता है। यद्यनप
राज्य नवधािमांडल की नवधायी शनक्त निलांनबत िहीं होती है, यह सांसद की अनधभावी शनक्त के अधीि हो जाती है। इस प्कार, कें द्र और
राज्यों के बीच नवधायी शनक्तयों का सामान्य नवतरण निलांनबत होता है, हालाांनक राज्य नवधािमांडल निलांनबत िहीं होता हैं। सांक्षपे में,
सनां वधाि सघां ीय के बजाय एकात्मक हो जाता है।
जब राष्ट्रीय आपातकाल की उद्घोषणा चल रही होती है, राष्ट्रपनत कें द्र और राज्यों के बीच राजस्व के सवां ैधानिक नवतरण को सांशोनधत
कर सकता है। इसका मतलब है नक राष्ट्रपनत कें द्र से राज्यों को नवत्त के हस्तातां रण को या तो कम या रद्द कर सकता है। इस तरह का
सांशोधि नवत्तीय वषग के अांत तक जारी रहता है नजसमें आपातकाल समाप्त हो जाता है।

Q. 49) अिुच्छे द 356 के तहत आपातकाि की उद्घोषणा के सबं ंध में निम्िनिनित कथिों पर नवचार करें:
1. राष्ट्रपनत के वल तभी उद्घोषणा जारी करता है जब राज्यपाल ररपोटग प्स्तुत करता है नक ऐसी नस्थनत उत्पन्ि हो गई है नजसमें
नकसी राज्य की सरकार सांनवधाि के प्ावधािों के अिुसार िहीं चल सकती है।
2. अिुच्छे द 356 के तहत आपातकाल की उद्घोषणा को सांसद के दोिों सदिों द्वारा इसके जारी होिे की तारीख से एक महीिे के
भीतर अिमु ोनदत नकया जािा चानहए।
ऊपर नदए गए कथिों में से कौि-सा सही हैं?

www.iasbaba.com rrs.iasbaba.com Page 38


IASbaba’s Rapid Revision (RaRe) Series Polity Day 1-120 2021
HINDI

a) के वल 1
b) के वल 2
c) दोिों 1 और 2
d) ि तो 1 और ि ही 2
Q. 49) Solution (d)
अिुच्छे द 355 यह सनु िनित करिे के नलए कें द्र पर एक कतगव्य लगाता है नक प्त्येक राज्य की सरकार सांनवधाि के प्ावधािों के अिुसार
चलती है। यह वह कतगव्य है नजसके प्दशगि में कें द्र राज्य में सांवैधानिक तांत्र के नवफल होिे की नस्थनत में अिुच्छे द 356 के तहत नकसी
राज्य की सरकार को अपिे हाथ में ले लेता है। इसे लोकनप्य रूप से 'राष्ट्रपनत शासि' के रूप में जािा जाता है। इसे 'राज्य आपातकाल'
या 'सांवैधानिक आपातकाल' के रूप में भी जािा जाता है।
अिच्ु छे द 356 राष्ट्रपनत को एक उद्घोषणा जारी करिे का अनधकार देता है, यनद वह सतां ष्टु है नक ऐसी नस्थनत उत्पन्ि हो गई है नजसमें
नकसी राज्य की सरकार सांनवधाि के प्ावधािों के अिुसार िहीं चल सकती है। नवशेष रूप से, राष्ट्रपनत या तो राज्य के राज्यपाल की
ररपोटग पर कायग कर सकता है या अन्यथा भी (अथागत राज्यपाल की ररपोटग के नबिा भी)।
राष्ट्रपनत शासि लगािे की घोषणा को सांसद के दोिों सदिों द्वारा इसके जारी होिे की तारीख से दो महीिे के भीतर अिमु ोनदत नकया
जािा चानहए। तथानप, यनद राष्ट्रपनत शासि की उद्घोषणा ऐसे समय में जारी की जाती है जब लोक सभा भगां हो गई हो या लोक सभा
का नवघटि उद्घोषणा के अिमु ोदि के नबिा दो महीिे की अवनध के दौराि हो, तो उद्घोषणा पहली बैठक से 30 नदिों तक बिी रहती है।
लोकसभा की बैठक के पिु गगठि के बाद भी, बशते नक राज्य सभा िे इसे इस बीच में मांजरू ी दे दी हो।

Q.50) निम्िनिनित कथिों पर नवचार करें:


1. राज्यों में राष्ट्रपनत शासि का िागररकों के मौनलक अनधकारों पर कोई प्भाव िहीं पडता है।
2. राज्यों में राष्ट्रपनत शासि हर छह महीिे के नलए ससां द की मजां रू ी से अनिनित काल तक जारी रखा जा सकता है।
3. राष्ट्रीय आपातकाल के दौराि ससां द राज्य के नलए काििू बिािे की शनक्त राष्ट्रपनत या उिके द्वारा निनदगष्ट नकसी अन्य प्ानधकरण
को सौंप सकती है।
उपरोक्त में से कौि सा कथि सही हैं?
a) के वल 1
b) के वल 1 और 2
c) के वल 3
d) 1, 2 और 3
Q. 50) Solution (a)

www.iasbaba.com rrs.iasbaba.com Page 39


IASbaba’s Rapid Revision (RaRe) Series Polity Day 1-120 2021
HINDI

राज्यों में राष्ट्रपनत शासि का िागररकों के मौनलक अनधकारों पर कोई प्भाव िहीं पडता है जबनक राष्ट्रीय आपातकाल के दौराि िागररकों
के मौनलक अनधकारों को प्भानवत करता है।
राष्ट्रपनत शासि के सांचालि के नलए अनधकतम अवनध निधागररत है, अथागत तीि वषग। इसके बाद, इसे समाप्त नकया जािा चानहए और
राज्य में सामान्य सांवैधानिक तत्रां को बहाल नकया जािा चानहए। दसू री ओर राष्ट्रीय आपातकाल के सांचालि के नलए कोई अनधकतम
अवनध निधागररत िहीं है। इसे हर छह महीिे में सांसद की मांजरू ी से अनिनित काल तक जारी रखा जा सकता है।
राष्ट्रीय आपातकाल के दौराि, ससां द राज्य सचू ी में उनपलनखत नवषयों पर के वल स्वयां ही काििू बिा सकती है, अथागत वह नकसी अन्य
निकाय या प्ानधकरण को उसे प्त्यायोनजत िहीं कर सकती है। दसू री ओर राष्ट्रपनत शासि के दौराि ससां द राज्य के नलए काििू बिािे
की शनक्त राष्ट्रपनत या उिके द्वारा निनदगष्ट नकसी अन्य प्ानधकरण को सौंप सकती है। अब तक यह प्था राष्ट्रपनत द्वारा उस राज्य से ससां द
सदस्यों के परामशग से राज्य के नलए काििू बिािे की रही है। ऐसे काििू ों को राष्ट्रपनत के कृ त्यों के रूप में जािा जाता है।
Q. 51) निम्िनिनित में से नकि नवषयों को संसद िे 42वें संशोधि अनधनियम, 1976 द्वारा राज्य सूची से समवती सूची में
स्थािांतररत कर नदया था?
1. सावगजनिक स्वास््य
2. कृ नष
3. नशक्षा
4. वि
िीचे नदए गए कूटों में से सही उत्तर चुनिए:
a) के वल 1 और 2
b) के वल 2 और 3
c) के वल 3 और 4
d) के वल 1 और 4
Q. 51) Solution (c)
सनां वधाि कें द्र और राज्यों के बीच नवधायी नवषयों के नत्र-स्तरीय नवतरण का प्ावधाि करता है, अथागत सातवीं अिसु चू ी में सचू ी- I (सघां
सचू ी), सचू ी- II (राज्य सचू ी) और सचू ी- III (समवती सचू ी) ।
समवती सचू ी में उनपलनखत नकसी भी मामले के सांबांध में सांसद और राज्य नवधानयका दोिों कािूि बिा सकते हैं। इस सचू ी में वतगमाि
में आपरानधक कािूि और प्निया, िागररक प्निया, नववाह और तलाक, जिसांख्या नियत्रां ण और पररवार नियोजि, नबजली, श्रम
कपयाण, आनथगक और सामानजक नियोजि, ड्रग्स, समाचार पत्र, नकताबें और नप्नां टांग प्ेस जैसे 52 नवषय (मल ू रूप से 47 नवषय) और
अन्य हैं। 1976 के 42वें सांशोधि अनधनियम िे पाचां नवषयों को राज्य सचू ी से समवती सचू ी में स्थािाांतररत कर नदया, अथागत, (ए)
नशक्षा, (बी) वि, (सी) वजि और माप, (डी) वन्य जीवों और पनक्षयों की सरु क्षा, और (ई) न्याय का प्शासि; सवोच्च न्यायालय और
उच्च न्यायालयों को छोडकर सभी न्यायालयों का गठि और सांगठि।

www.iasbaba.com rrs.iasbaba.com Page 40


IASbaba’s Rapid Revision (RaRe) Series Polity Day 1-120 2021
HINDI

Q. 52) निम्िनिनित कथिों पर नवचार करें:


1. राष्ट्रपनत को राज्य नवधानयका द्वारा पाररत नवधेयकों पर पणू ग वीटो प्ाप्त होता है जो राज्यपाल द्वारा राष्ट्रपनत के नवचार के नलए
आरनक्षत होते हैं।
2. व्यापार और वानणज्य की स्वतत्रां ता पर प्नतबांध लगािे वाले नवधेयकों को राष्ट्रपनत की पवू ग स्वीकृ नत से ही राज्य नवधािमांडल
में प्स्ततु नकया जा सकता है।
उपरोक्त में से कौि सा कथि सही हैं?
a) के वल 1
b) के वल 2
c) दोिों 1 और 2
d) ि तो 1 और ि ही 2
Q. 52) Solution (c)
सांनवधाि कें द्र को राज्य के नवधायी मामलों पर निम्िनलनखत तरीकों से नियांत्रण करिे का अनधकार देता है:
• राज्यपाल राज्य नवधािमांडल द्वारा पाररत कुछ प्कार के नवधेयकों को राष्ट्रपनत के नवचार के नलए सरु नक्षत रख सकता है।
राष्ट्रपनत को उि पर पणू ग वीटो प्ाप्त होता है।
• राज्य सचू ी में उनपलनखत कुछ मामलों पर नवधेयक राष्ट्रपनत की पवू ग स्वीकृ नत से ही राज्य नवधािमांडल में प्स्तुत नकए जा
सकते हैं। (उदाहरण के नलए, व्यापार और वानणज्य की स्वतत्रां ता पर प्नतबांध लगािे वाले नवधेयक)।
• कें द्र राज्यों को नवत्तीय आपातकाल के दौराि राष्ट्रपनत के नवचार के नलए राज्य नवधािमांडल द्वारा पाररत धि नवधेयकों और
अन्य नवत्तीय नवधेयकों को आरनक्षत करिे का निदेश दे सकता है।

Q. 53) कें द्र-राज्य संबध


ं ों के सदं भा में, निम्िनिनित कथिों पर नवचार करें:
1. समवती नवषय पर एक काििू , हालानां क ससां द द्वारा अनधनियनमत नकया जाता है, राज्यों द्वारा निष्ट्पानदत नकया जाता है।
2. राज्य के उच्च न्यायालय के न्यायाधीशों को राज्यपाल द्वारा नियक्त
ु नकया जाता है, लेनकि राष्ट्रपनत द्वारा उन्हें हटाया जाता है।
3. ससां द कें द्र के कायगकारी कायों को उक्त राज्य की सहमनत के नबिा नकसी राज्य को सौंप सकती है।
उपरोक्त में से कौि सा कथि सही हैं?
a) के वल 1 और 2
b) के वल 2 और 3
c) के वल 3

www.iasbaba.com rrs.iasbaba.com Page 41


IASbaba’s Rapid Revision (RaRe) Series Polity Day 1-120 2021
HINDI

d) के वल 1 और 3
Q. 53) Solution (d)
उि मामलों के सांबांध में नजि पर ससां द और राज्य नवधािसभाओ ां दोिों के पास कािूि बिािे की शनक्त है (अथागत, समवती सचू ी में वनणगत
नवषय), जब तक सांवैधानिक उपबधां या सांसदीय नवनध इसे कें द्र में नवनशष्ट रूप से प्दाि िहीं कर देती, तब तक कायगपानलका-शनक्त राज्यों
के पास रहती है। इसनलए, समवती नवषय पर एक कािूि, हालाांनक सांनवधाि या सांसद द्वारा अन्यथा निदेनशत नकए जािे के नसवाय, नकसी
समवती नवषय पर लागू नवनध का निष्ट्पादि राज्यों द्वारा नकया जािा है।
यद्यनप भारत की राज्य व्यवस्था दोहरी है, लेनकि न्याय प्शासि की कोई दोहरी व्यवस्था िहीं है। दसू री ओर, सनां वधाि िे एक एकीकृ त
न्यानयक प्णाली की स्थापिा की, नजसके शीषग पर सवोच्च न्यायालय और उसके िीचे राज्य उच्च न्यायालय थे। अदालतों की यह
एकल प्णाली कें द्रीय काििू ों के साथ-साथ राज्य के काििू ों को भी लागू करती है। यह उपचारात्मक प्निया में नवनवधता को खत्म करिे
के नलए नकया गया है। एक राज्य उच्च न्यायालय के न्यायाधीशों की नियनु क्त राष्ट्रपनत द्वारा भारत के मख्ु य न्यायाधीश और राज्य के
राज्यपाल के परामशग से की जाती है। उन्हें राष्ट्रपनत द्वारा स्थािातां ररत और हटाया भी जा सकता है।
सनां वधाि कें द्र के कायगकारी कायों को उक्त राज्य की सहमनत के नबिा नकसी राज्य को सौंपिे का प्ावधाि करता है। लेनकि, इस मामले
में, सांसद द्वारा निष्ट्पादि होता है, ि नक राष्ट्रपनत द्वारा। इस प्कार, सांघ सचू ी के नवषय पर सांसद द्वारा बिाया गया कािूि नकसी राज्य को
शनक्तयाां प्दाि कर सकता है और कतगव्यों को लागू कर सकता है, या कें द्र द्वारा नकसी राज्य को शनक्तयाां प्दाि करिे और कतगव्यों को
लागू करिे के नलए अनधकृ त कर सकता है (सांबांनधत राज्य की सहमनत के साथ या सहमनत के नबिा भी) । परांतु, राज्य नवधानयका द्वारा
समाि कायग िहीं नकया जा सकता है।

Q. 54) निम्िनिनित में से कौि से कर राज्यों द्वारा िगाए, एकत्र और बिाए रिे जाते हैं?
1. कृ नष आय पर कर।
2. नबजली के उपभोग या नबिी पर कर
3. कुछ करों पर अनधभार
िीचे नदए गए कूटों में से सही उत्तर चुनिए:
a) के वल 1 और 2
b) के वल 2 और 3
c) के वल 1 और 3
d) 1, 2 और 3
Q. 54) Solution (a)
18 कर हैं जो नवशेष रूप से राज्यों से सांबांनधत हैं। उन्हें राज्य सचू ी में शानमल नकया जाता है और राज्यों द्वारा लगाया जाता है, एकत्र नकया
जाता है और बिाए रखा जाता है। वो हैं:

www.iasbaba.com rrs.iasbaba.com Page 42


IASbaba’s Rapid Revision (RaRe) Series Polity Day 1-120 2021
HINDI

1. भू राजस्व
2. कृ नष आय पर कर
3. कृ नष भनू म के उत्तरानधकार के सबां धां में शपु क
4. कृ नष भनू म के सबां धां में सपां नत्त शपु क
5. भनू म और भविों पर कर
6. खनिज अनधकारों पर कर
7. मािव उपभोग के नलए मादक शराब पर उत्पाद शपु क; अफीम, भारतीय भागां और अन्य िशीले पदाथग और िशीले पदाथग,
लेनकि इसमें अपकोहल या िशीले पदाथों से यक्त
ु औषधीय और प्साधि की चीजें शानमल िहीं है
8. नबजली की खपत या नबिी पर कर
9. पेरोनलयम िूड, हाई स्पीड डीजल, मोटर नस्पररट (आमतौर पर पेरोल के रूप में जािा जाता है), प्ाकृ नतक गैस, नवमािि
टबागइि ईधि
ां और मािव उपभोग के नलए मादक शराब की नबिी पर कर, लेनकि अांतरागज्यीय व्यापार या वानणज्य के दौराि
नबिी या ऐसे माल के अांतरराष्ट्रीय व्यापार या वानणज्य के दौराि नबिी शानमल िहीं है
10. सडक या अतां देशीय जलमागग द्वारा ले जाए जािे वाले माल और यानत्रयों पर कर
11. वाहिों पर कर
12. जािवरों और िावों पर कर
13. टोल
14. व्यवसायों, व्यापारों, कॉनलगां और रोजगार पर कर
15. कै नपटेशि टैक्स (capitation taxes)
16. पचां ायत या िगर पानलका या क्षेत्रीय पररषद या नजला पररषद द्वारा लगाए और एकत्र की गई सीमा तक मिोरांजि और मिोरांजि
पर कर
17. दस्तावेजों पर स्टाम्प शपु क (सांघ सचू ी में निनदगष्ट को छोडकर)
18. राज्य सचू ी में उनपलनखत मामलों पर शपु क (अदालत शपु क को छोडकर)
ससां द नकसी भी समय अिुच्छे द 269 और 270 (ऊपर उनपलनखत) में निनदगष्ट करों और शपु कों पर अनधभार लगा सकती है। इस तरह के
अनधभार की आय नवशेष रूप से कें द्र के पास जाती है। दसू रे शब्दों में, इि अनधभारों में राज्यों का कोई नहस्सा िहीं होता है। हालाांनक,
वस्तु एवां सेवा कर (जीएसटी) को इस अनधभार से छूट दी गई है। दसू रे शब्दों में, यह अनधभार GST पर िहीं लगाया जा सकता है।

Q. 55) अंतरााज्यीय जि नववाद अनधनियम के संबध


ं में निम्िनिनित कथिों पर नवचार करें:

www.iasbaba.com rrs.iasbaba.com Page 43


IASbaba’s Rapid Revision (RaRe) Series Polity Day 1-120 2021
HINDI

1. यह कें द्र सरकार को एक अतां र-राज्यीय िदी या िदी घाटी के पािी के सांबांध में दो या दो से अनधक राज्यों के बीच नववाद के
निणगय के नलए एक तदथग न्यायानधकरण स्थानपत करिे का अनधकार देता है।
2. न्यायानधकरण के निणगय स्वभाव में अिुशांसात्मक होते हैं और नववाद के पक्षकारों पर बाध्यकारी िहीं हैं।
3. इस अनधनियम के तहत पीनडत पक्ष न्यायानधकरण के निदेशों के नखलाफ सीधे सवोच्च न्यायालय का दरवाजा खटखटा
सकता है।
उपरोक्त में से कौि सा कथि सही हैं?
a) के वल 1
b) के वल 1 और 2
c) के वल 3
d) 1, 2 और 3
Q. 55) Solution (a)
सांनवधाि का अिुच्छे द 262 अांतरागज्यीय जल नववादों के न्यायनिणगयि का प्ावधाि करता है। यह दो प्ावधाि करता है:
(i) सांसद कािूि द्वारा नकसी भी अतां रागज्यीय िदी और िदी घाटी के जल के उपयोग, नवतरण और नियत्रां ण के सांबांध में नकसी भी नववाद
या नशकायत के न्यायनिणगयि का प्ावधाि कर सकती है।
(ii) सांसद यह भी प्ावधाि कर सकती है नक इस तरह के नकसी भी नववाद या नशकायत के सांबांध में ि तो सवोच्च न्यायालय और ि ही
कोई अन्य अदालत अपिे अनधकार क्षेत्र का प्योग करे ।
इस प्ावधाि के तहत, ससां द िे दो कािूि बिाए हैं:
• िदी बोडग अनधनियम (1956)
• अांतरागज्यीय जल नववाद अनधनियम (1956)
िदी बोडग अनधनियम अांतरागज्यीय िदी और िदी घानटयों के नियमि और नवकास के नलए िदी बोडों की स्थापिा का प्ावधाि करता
है। सांबांनधत राज्य सरकारों के अिुरोध पर उन्हें सलाह देिे के नलए कें द्र सरकार द्वारा एक िदी बोडग की स्थापिा की जाती है।
अतां र-राज्यीय जल नववाद अनधनियम कें द्र सरकार को एक अतां र-राज्यीय िदी या िदी घाटी के पािी के सबां धां में दो या दो से अनधक
राज्यों के बीच नववाद के निणगय के नलए एक तदथग न्यायानधकरण स्थानपत करिे का अनधकार देता है। न्यायानधकरण का निणगय अनां तम
होगा और नववाद के पक्षकारों पर बाध्यकारी होगा। नकसी भी जल नववाद के सबां धां में ि तो सवोच्च न्यायालय और ि ही नकसी अन्य
न्यायालय के पास अनधकार क्षेत्र है, नजसे इस अनधनियम के तहत ऐसे न्यायानधकरण को सांदनभगत नकया जा सकता है।

Q. 56) अंतरााज्यीय जि नववाद न्यायानधकरणों में शानमि राज्यों के संबंध में निम्िनिनित में से कौि सा यग्ु म सही सुमेनित
है?

www.iasbaba.com rrs.iasbaba.com Page 44


IASbaba’s Rapid Revision (RaRe) Series Polity Day 1-120 2021
HINDI

(अांतरागज्यीय जल नववाद न्यायानधकरण) (शानमल राज्य))


1. महादयी जल नववाद न्यायानधकरण गोवा, किागटक और महाराष्ट्र
2. वांशधारा जल नववाद न्यायानधकरण तेलगां ािा, तनमलिाडु और आध्रां प्देश
3. महािदी जल नववाद न्यायानधकरण ओनडशा और छत्तीसगढ़
िीचे नदए गए कूटों में से सही उत्तर चुनिए:
a) के वल 1 और 2
b) के वल 1 और 3
c) के वल 2 और 3
d) 1, 2 और 3
Q. 56) Solution (b)
अब तक स्थानपत अांतरागज्यीय जल नववाद न्यायानधकरण हैं:
SI. जल नववाद न्यायानधकरण स्थानपत शानमल राज्य
No.
1. कृ ष्ट्णा जल नववाद न्यायानधकरण-I 1969 महाराष्ट्र, किागटक और आध्रां प्देश
2. गोदावरी जल नववाद न्यायानधकरण 1969 महाराष्ट्र, किागटक, आांध्र
प्देश, मध्य प्देश और उडीसा
3. िमगदा जल नववाद न्यायानधकरण 1969 राजस्थाि, गजु रात, मध्य प्देश
और महाराष्ट्र
4. रावी और ब्यास जल नववाद न्यायानधकरण 1986 पजां ाब, हररयाणा और राजस्थाि
5. कावेरी जल नववाद न्यायानधकरण 1990 किागटक, के रल, तनमलिाडु और
पदु चु ेरी
6. कृ ष्ट्णा जल नववाद न्यायानधकरण-II 2004 महाराष्ट्र, किागटक और आध्रां प्देश
7. वांशधारा जल नववाद न्यायानधकरण 2010 ओनडशा और आांध्र प्देश
8. महादयी जल नववाद न्यायानधकरण 2010 गोवा, किागटक और महाराष्ट्र
9. महािदी जल नववाद न्यायानधकरण 2018 उडीसा और छत्तीसगढ़

• 1969 में गनठत कृ ष्ट्णा जल नववाद न्यायानधकरण-I िे 1976 में अपिा अांनतम निणगय नदया।
• 1969 में गनठत गोदावरी जल नववाद न्यायानधकरण िे 1979 में अपिा अांनतम निणगय नदया।
• 1990 में गनठत कावेरी जल नववाद न्यायानधकरण िे 2007 में अपिा अांनतम निणगय नदया।

www.iasbaba.com rrs.iasbaba.com Page 45


IASbaba’s Rapid Revision (RaRe) Series Polity Day 1-120 2021
HINDI

• 1969 में बिाए गए िमगदा जल नववाद न्यायानधकरण िे 1979 में अपिा अांनतम निणगय नदया।
• बाकी न्यायानधकरण अभी भी सनिय हैं।

Q. 57) अतं रराज्यीय पररषद (Inter-State Council) के सदं भा में निम्िनिनित कथिों पर नवचार कीनजएः:
1. अांतरराज्यीय पररषद का उद्देश्य राज्यों को यथासांभव शनक्तयों का नवकें द्रीकरण करिा है।
2. यह एक अस्थाई सांवैधानिक निकाय है।
3. इसके निणगय प्कृ नत में बाध्यकारी होते हैं।
उपरोक्त में से कौि सा कथि सही हैं?
a) के वल 1
b) के वल 1 और 2
c) के वल 3
d) 1, 2 और 3
Q. 57) Solution (b)
सघां ीय मद्दु ों से निपटिे के नलए भारत के सनां वधाि के अिुच्छे द 263 द्वारा प्दाि की गई अांतरराज्यीय पररषद (आईएससी) एक सांवैधानिक
निकाय है। इसका अनधदेश अांतरराज्यीय नववादों की जाांच और सलाह देिा और बेहतर िीनत समन्वय के नलए नसफाररशें प्दाि करिा
है।
अांतरराज्यीय पररषद के उद्देश्य इस प्कार हैं::
• यथासम्भव राज्यों को शनक्तयों का नवके न्द्रीकरण।
• राज्यों को नवत्तीय ससां ाधिों का अनधक हस्तातां रण।
• हस्तातां रण की व्यवस्था इस प्कार की जाए नक राज्य अपिे दानयत्वों को परू ा कर सकें ।
• राज्यों को नदए जािे वाले ऋणों का सबां धां 'उत्पादक नसद्धातां ' से होिा चानहए।
• राज्यों में कें द्रीय सशस्त्र पनु लस बलों की तैिाती या तो उिके अिरु ोध पर या नभन्ि प्कार से।
अतां रराज्यीय पररषद में निम्िनलनखत सदस्य होते हैं:
• प्धाि मांत्री, अध्यक्ष।
• सभी राज्यों के मख्ु यमांत्री।
• नवधािसभाओ ां वाले कें द्र शानसत प्देशों के मख्ु यमांत्री।

www.iasbaba.com rrs.iasbaba.com Page 46


IASbaba’s Rapid Revision (RaRe) Series Polity Day 1-120 2021
HINDI

• कें द्र शानसत प्देशों के प्शासक नजिकी नवधािसभाएां िहीं हैं।


• गृह मत्रां ी सनहत 6 कें द्रीय कै नबिेट मांनत्रयों को प्धािमत्रां ी द्वारा िानमत नकया जाएगा।
• राज्यों के राज्यपालों को राष्ट्रपनत शासि के अधीि प्शानसत नकया जाता है
• स्थाई सनमनत
• गृह मत्रां ी, अध्यक्ष
• 5 कें द्रीय कै नबिेट मत्रां ी
• 9 मख्ु यमत्रां ी
अतां रराज्यीय पररषद के वल एक सलाहकार निकाय है नजसका कें द्र या राज्य पर कोई बधां ि िहीं है। इस प्कार, अक्सर कें द्र और राज्यों
में सरकार द्वारा इसकी नसफाररशों की अिदेखी की जाती है।

Q. 58) क्षेत्रीय पररषदों (Zonal Councils) के संदभा में निम्िनिनित कथिों पर नवचार कीनजएः
1. ये सांवैधानिक निकाय हैं।
2. इिकी अध्यक्षता प्धािमांत्री करते हैं।
3. महाराष्ट्र और गोवा राज्य पनिमी क्षेत्रीय पररषद का नहस्सा हैं।
उपरोक्त में से कौि सा कथि सही हैं?
a) के वल 1
b) के वल 2 और 3
c) के वल 3
d) 1, 2 और 3
Q. 58) Solution (c)
क्षेत्रीय पररषदें साांनवनधक (सांवैधानिक िहीं) निकाय हैं। वे सांसद के एक अनधनियम, यािी राज्य पिु गगठि अनधनियम 1956 द्वारा स्थानपत
नकए गए हैं। इस अनधनियम िे देश को पाांच क्षेत्रों (उत्तरी, मध्य, पवू ी, पनिमी और दनक्षणी) में नवभानजत नकया और प्त्येक क्षेत्र के नलए
एक क्षेत्रीय पररषद प्दाि की।
प्त्येक क्षेत्रीय पररषद में निम्िनलनखत सदस्य होते हैं: (ए) कें द्र सरकार के गृह मत्रां ी। (बी) क्षेत्र के सभी राज्यों के मख्ु यमत्रां ी। (सी) क्षेत्र में
प्त्येक राज्य से दो अन्य मत्रां ी। (डी) क्षेत्र में प्त्येक कें द्र शानसत प्देश का प्शासक।
कें द्र सरकार के गृह मांत्री पाचां क्षेत्रीय पररषदों के सामान्य अध्यक्ष होते हैं। प्त्येक मख्ु यमांत्री बारी-बारी से पररषद के उपाध्यक्ष के रूप में
कायग करता है, एक समय में एक वषग की अवनध के नलए पद धारण करते है

www.iasbaba.com rrs.iasbaba.com Page 47


IASbaba’s Rapid Revision (RaRe) Series Polity Day 1-120 2021
HINDI

गजु रात, महाराष्ट्र, गोवा, दादरा और िगर हवेली और दमि और दीव के राज्य और कें द्र शानसत प्देश पनिमी क्षेत्रीय पररषद का नहस्सा
हैं।

Q. 59) कें द्र-राज्य संबध


ं ों के सबं ंध में, निम्िनिनित पर नवचार करें:
1. रेड यनू ियि
2. अतां र-राज्यीय व्यापार और वानणज्य
3. बैंनकांग
4. श्रम कपयाण
उपरोक्त में से कौि-से सघं सच
ू ी के नवषय हैं?
a) के वल 1 और 2
b) के वल 2 और 3
c) के वल 3 और 4
d) के वल 1 और 4
Q. 59) Solution (b)
सनां वधाि कें द्र और राज्यों के बीच नवधायी नवषयों के नत्र-स्तरीय नवतरण का प्ावधाि करता है, अथागत सातवीं अिुसचू ी में सचू ी- I (सांघ
सचू ी), सचू ी- II (राज्य सचू ी) और सचू ी- III (समवती सचू ी)
सघां सचू ी में उनपलनखत नकसी भी मामले के सांबांध में काििू बिािे के नलए सांसद के पास नवशेष शनक्तयाां हैं। इस सचू ी में वतगमाि में
रक्षा, बैंनकांग, नवदेशी मामलों, मद्रु ा, परमाणु ऊजाग, बीमा, डाक और तार, सचां ार, अतां र-राज्य व्यापार और वानणज्य, जिगणिा, लेखा
परीक्षा आनद जैसे 98 नवषय (मल ू रूप से 971 नवषय) हैं।
रेड यनू ियि और श्रम कपयाण समवती सचू ी में शानमल नवषय हैं। समवती सचू ी में उनपलनखत नकसी भी मामले के सांबांध में सांसद और
राज्य नवधािमांडल दोिों कािूि बिा सकते हैं।

Q.60) जब दो या दो से अनधक राज्यों की नवधानयकाएं राज्य सचू ी में नकसी मामिे पर काििू बिािे के निए सस
ं द से अिरु ोध
करिे वािे प्रस्ताव पाररत करती हैं, तो सस
ं द उस मामिे को नवनियनमत करिे के निए काििू बिा सकती है। इस सदं भा में
उपयाक्त
ु प्रावधाि के अंतगात निम्िनिनित में से कौि-सा/से कािूि पाररत नकया गया है/हैं?
1. वन्य जीव (सांरक्षण) अनधनियम, 1972
2. जल (प्दषू ण की रोकथाम और नियांत्रण) अनधनियम, 1974
3. शहरी भनू म (सीमा और नवनियमि) अनधनियम, 1976

www.iasbaba.com rrs.iasbaba.com Page 48


IASbaba’s Rapid Revision (RaRe) Series Polity Day 1-120 2021
HINDI

िीचे नदए गए कूटों में से सही उत्तर चुनिए:


a) के वल 1 और 2
b) के वल 2 और 3
c) के वल 1 और 3
d) 1, 2 और 3
Q. 60) Solution (d)
जब दो या दो से अनधक राज्यों की नवधानयकाएां राज्य सचू ी में नकसी मामले पर काििू बिािे के नलए ससां द से अिरु ोध करिे वाले प्स्ताव
पाररत करती हैं, तो ससां द उस मामले को नवनियनमत करिे के नलए काििू बिा सकती है। इस प्कार अनधनियनमत काििू के वल उि
राज्यों पर लागू होता है नजन्होंिे प्स्ताव पाररत नकए हैं। हालााँनक, कोई भी अन्य राज्य अपिी नवधानयका में इस आशय का प्स्ताव पाररत
करके बाद में इसे अपिा सकता है। इस तरह के काििू को के वल ससां द द्वारा सश ां ोनधत या निरस्त नकया जा सकता है, ि नक सबां नां धत
राज्यों के नवधािमांडलों द्वारा।
उपरोक्त प्ावधाि के तहत एक प्स्ताव पाररत करिे का प्भाव यह है नक सांसद उस मामले के सांबांध में कािूि बिािे की हकदार हो जाती
है नजसके नलए उसे कािूि बिािे की कोई शनक्त िहीं है। दसू री ओर, राज्य नवधानयका की उस मामले के सांबांध में काििू बिािे की शनक्त
समाप्त हो जाती है। सांकपप उस मामले के सांबांध में राज्य नवधानयका की शनक्त के त्याग या समपगण के रूप में कायग करता है और इसे परू ी
तरह से सांसद के हाथों में रखा जाता है, जो अके ले ही इसके सांबांध में कािूि बिा सकता है।
उपरोक्त प्ावधाि के तहत पाररत कािूिों के कुछ उदाहरण परु स्कार प्नतस्पधाग अनधनियम, 1955 ; वन्य जीव (सांरक्षण) अनधनियम,
1972; जल (प्दषू ण की रोकथाम और नियांत्रण) अनधनियम, 1974; शहरी भनू म (सीमा और नवनियमि) अनधनियम, 1976; और
मािव अांग प्त्यारोपण अनधनियम, 1994 हैं।
Q. 61) राष्ट्रपनत के चुिाव के निए निवााचक मडं ि में शानमि हैं:
1. सांसद के दोिों सदिों के सभी सदस्य।
2. कें द्र शानसत प्देश नदपली और पडु ु चेरी की नवधािसभाओ ां के सभी सदस्य।
3. राज्यों की नवधाि सभाओ ां के निवागनचत सदस्य।
िीचे नदए गए कूटों में से सही उत्तर चुनिए:
a) के वल 1
b) के वल 1 और 2
c) के वल 3
d) 1, 2 और 3
Q. 61) Solution (c)

www.iasbaba.com rrs.iasbaba.com Page 49


IASbaba’s Rapid Revision (RaRe) Series Polity Day 1-120 2021
HINDI

सांघ की कायगकाररणी में राष्ट्रपनत, उपराष्ट्रपनत, प्धाि मांत्री, मांनत्रपररषद और भारत के महान्यायवादी होते हैं।
राष्ट्रपनत भारतीय राज्य का प्मख
ु होता है। वह भारत के प्थम िागररक हैं और राष्ट्र की एकता, अखांडता और एकजटु ता के प्तीक के
रूप में कायग करते हैं।
राष्ट्रपनत का चिु ाव सीधे लोगों द्वारा िहीं बनपक निवागचक मडां ल के सदस्यों द्वारा नकया जाता है, नजसमें शानमल हैं:
• सांसद के दोिों सदिों के निवागनचत सदस्य;
• राज्यों की नवधािसभाओ ां के निवागनचत सदस्य;
• कें द्र शानसत प्देशों नदपली और पडु ु चेरी की नवधािसभाओ ां के निवागनचत सदस्य।
इस प्कार, सांसद के दोिों सदिों के मिोिीत सदस्य, राज्य नवधाि सभाओ ां के मिोिीत सदस्य, राज्य नवधाि पररषदों के सदस्य (निवागनचत
और मिोिीत दोिों) (नद्वसदिीय नवधानयका के मामले में) तथा नदपली और पडु ु चरे ी की नवधाि सभाओ ां के मिोिीत सदस्य राष्ट्रपनत के
चिु ाव में भाग िहीं लेते हैं। जहाां एक नवधािसभा को भांग कर नदया जाता है, सदस्यों को राष्ट्रपनत चिु ाव में मतदाि करिे के नलए अहगता
प्ाप्त िहीं होती है, भले ही राष्ट्रपनत चिु ाव से पहले भगां नवधािसभा के नलए िए चिु ाव ि हुए हों।

Q. 62) राष्ट्रपनत के महानभयोग के संदभा में, निम्िनिनित कथिों पर नवचार कीनजएः:


1. महानभयोग की प्निया सांसद के नकसी भी सदि द्वारा शरू
ु की जा सकती है।
2. महानभयोग के आरोपों पर सदि के एक चौथाई सदस्यों के हस्ताक्षर होिे चानहए
3. सांसद के नकसी भी सदि के मिोिीत सदस्य राष्ट्रपनत के महानभयोग में भाग िहीं लेते हैं।
4. राज्यों तथा कें द्र शानसत प्देशों नदपली और पडु ु चेरी की नवधािसभाओ ां के निवागनचत सदस्य राष्ट्रपनत के महानभयोग में भाग
लेते हैं।
उपरोक्त में से कौि सा कथि सही हैं?
a) के वल 1 और 2
b) के वल 2 और 3
c) के वल 1 और 4
d) के वल 1 और 4
Q. 62) Solution (a)
राष्ट्रपनत को 'सांनवधाि के उपलांघि' के नलए महानभयोग की प्निया द्वारा पद से हटाया जा सकता है। हालााँनक, सांनवधाि 'सांनवधाि का
उपलांघि' वाक्याांश के अथग को पररभानषत िहीं करता है।
महानभयोग की प्निया सांसद के नकसी भी सदि द्वारा शरू ु की जा सकती है। इि आरोपों पर सदि के एक चौथाई सदस्यों (नजन्होंिे
आरोप तय नकए थे) द्वारा हस्ताक्षर नकए जािे चानहए और राष्ट्रपनत को 14 नदिों का िोनटस नदया जािा चानहए। महानभयोग प्स्ताव उस

www.iasbaba.com rrs.iasbaba.com Page 50


IASbaba’s Rapid Revision (RaRe) Series Polity Day 1-120 2021
HINDI

सदि की कुल सदस्यता के दो-नतहाई बहुमत से पाररत होिे के बाद, इसे दसू रे सदि में भेजा जाता है, नजसे आरोपों की जाचां करिी चानहए।
राष्ट्रपनत को ऐसी जाचां में उपनस्थत होिे और प्नतनिनधत्व करिे का अनधकार है। यनद दसू रा सदि भी आरोपों को बरकरार रखता है और
कुल सदस्यता के दो-नतहाई बहुमत से महानभयोग प्स्ताव पाररत करता है, तो राष्ट्रपनत को उस नतनथ से अपिे पद से हटा नदया जाता है
नजस नदि प्स्ताव पाररत नकया जाता है।
इस प्कार, महानभयोग सांसद में एक अधग-न्यानयक प्निया है। इस सदां भग में दो बातों का ध्याि रखिा चानहए:
• ससां द के नकसी भी सदि के मिोिीत सदस्य राष्ट्रपनत के महानभयोग में भाग ले सकते हैं, हालानां क वे उसके चिु ाव में भाग िहीं
लेते हैं।
• राज्यों और कें द्र शानसत प्देशों नदपली और पडु ु चरे ी की नवधािसभाओ ां के निवागनचत सदस्य राष्ट्रपनत के महानभयोग में भाग
िहीं लेते हैं, हालानां क वे उिके चिु ाव में भाग लेते हैं।
अभी तक नकसी राष्ट्रपनत पर महानभयोग िहीं लगाया गया है।

Q. 63) निम्िनिनित कथिों पर नवचार करें:


1. राष्ट्रपनत चिु ाव आयोग के परामशग से सांसद के सदस्यों की अयोग्यता के प्श्नों पर निणगय लेता है।
2. राष्ट्रपनत द्वारा प्ख्यानपत अध्यादेशों को सांसद द्वारा इसकी पिु : बैठक से छह महीिे के भीतर अिुमोनदत नकया जािा चानहए।
3. धि नवधेयक को राष्ट्रपनत की पवू ग अिुशांसा से ही सांसद में प्स्ततु नकया जा सकता है।
उपरोक्त में से कौि सा कथि सही हैं?
a) के वल 1 और 2
b) के वल 1 और 3
c) के वल 2 और 3
d) 1, 2 और 3
Q. 63) Solution (b)
राष्ट्रपनत भारत की सांसद का एक अनभन्ि अगां है, और निम्िनलनखत नवधायी शनक्तयों का आिांद लेता है:
• वह चिु ाव आयोग के परामशग से ससां द के सदस्यों की अयोग्यता के प्श्नों पर निणगय लेता है।
• जब ससां द का सत्र िहीं चल रहा हो तो वह अध्यादेश जारी कर सकता है। इि अध्यादेशों को ससां द द्वारा अपिे पिु : सयां ोजि
से छह सप्ताह के भीतर अिमु ोनदत नकया जािा चानहए। वह नकसी भी समय अध्यादेश को वापस भी ले सकता है
• वह सांसद को बल ु ा सकता है या उसका सत्रावसाि कर सकता है और लोकसभा को भांग कर सकता है। वह सांसद के दोिों
सदिों की सांयक्त
ु बैठक भी बुला सकता है, नजसकी अध्यक्षता लोकसभा के अध्यक्ष करते हैं।
• वह प्त्येक आम चिु ाव और प्त्येक वषग के पहले सत्र के बाद पहले सत्र के प्ारांभ में सांसद को सांबोनधत कर सकता है।

www.iasbaba.com rrs.iasbaba.com Page 51


IASbaba’s Rapid Revision (RaRe) Series Polity Day 1-120 2021
HINDI

• वह लोकसभा के नकसी भी सदस्य को इसकी कायगवाही की अध्यक्षता करिे के नलए नियक्त


ु कर सकता है जब अध्यक्ष और
उपाध्यक्ष दोिों के पद ररक्त हो जाते हैं।
• ससां द में कुछ प्कार के नवधेयकों को पेश करिे के नलए उिकी पवू ग नसफाररश या अिुमनत की आवश्यकता होती है। उदाहरण
के नलए, एक नवधेयक नजसमें भारत की सांनचत निनध से व्यय शानमल है, या राज्यों की सीमाओ ां में पररवतगि या एक िए राज्य
के निमागण के नलए एक नवधेयक।
राष्ट्रपनत की नवत्तीय शनक्तयाां और कायग हैं:
• उिकी पवू ग नसफाररश से ही धि नवधेयक सांसद में पेश नकए जा सकते हैं।
• वह वानषगक नवत्तीय नववरण (यािी, कें द्रीय बजट) को सांसद के समक्ष रखिे को कहता है।
• उसकी नसफाररश के अलावा अिदु ाि की कोई मागां िहीं की जा सकती है।
• वह नकसी भी अप्त्यानशत व्यय को परू ा करिे के नलए भारत की आकनस्मक निनध से अनग्रम ले सकता है।
• वह कें द्र और राज्यों के बीच राजस्व के नवतरण की नसफाररश करिे के नलए हर पाचां साल के बाद एक नवत्त आयोग का गठि
करता है।

Q. 64) निम्िनिनित कथिों पर नवचार करें:


1. गैर-सरकारी सदस्यों के नवधेयकों के सांबांध में राष्ट्रपनत पणू ग वीटो (absolute veto) का प्योग करता है।
2. जब राष्ट्रपनत सांसद के पिु नवगचार के नलए नकसी नवधेयक को लौटाता है तो वह निलांबिकारी वीटो (suspensive veto) का
प्योग करता है।
3. राष्ट्रपनत राज्य नवधाि के सांबांध में पॉके ट वीटो का प्योग कर सकते हैं।
उपरोक्त में से कौि सा कथि सही हैं?
a) के वल 1 और 2
b) के वल 2 और 3
c) के वल 1 और 3
d) 1, 2 और 3
Q. 64) Solution (d)
ससां द द्वारा पाररत नवधेयक तभी अनधनियम बि सकता है जब उसे राष्ट्रपनत की सहमनत प्ाप्त हो। जब ऐसा नवधेयक राष्ट्रपनत की सहमनत
के नलए प्स्ततु नकया जाता है, तो उसके पास तीि नवकपप होते हैं (सनां वधाि के अिुच्छे द 111 के तहत):
1. वह नवधेयक पर अपिी सहमनत दे सकता है, या
2. वह नवधेयक पर अपिी सहमनत रोक सकता है, या
www.iasbaba.com rrs.iasbaba.com Page 52
IASbaba’s Rapid Revision (RaRe) Series Polity Day 1-120 2021
HINDI

3. वह सांसद के पिु नवगचार के नलए नवधेयक (यनद यह धि नवधेयक िहीं है) लौटा सकता है। हालााँनक, यनद नवधेयक को सांसद
द्वारा सांशोधिों के साथ या नबिा सश
ां ोधि के नफर से पाररत नकया जाता है और राष्ट्रपनत को नफर से प्स्ततु नकया जाता है, तो
राष्ट्रपनत को नवधेयक पर अपिी सहमनत देिी होगी।
इस प्कार, राष्ट्रपनत के पास ससां द द्वारा पाररत नवधेयकों पर वीटो शनक्त होती है, अथागत वह नवधेयकों पर अपिी सहमनत रोक सकता है।
राष्ट्रपनत द्वारा निम्िनलनखत दो मामलों में पणू ग वीटो का प्योग नकया जाता है:
• गैर-सरकारी सदस्यों के नवधेयकों के सबां धां में (अथागत, ससां द के नकसी ऐसे सदस्य द्वारा प्स्ततु नवधेयक जो मत्रां ी िहीं है); तथा
• सरकारी नवधेयकों के सबां धां में जब मनां त्रमडां ल इस्तीफा दे देता है (नबलों के पाररत होिे के बाद लेनकि राष्ट्रपनत द्वारा सहमनत
से पहले) और िया मनां त्रमडां ल राष्ट्रपनत को ऐसे नवधेयकों पर अपिी सहमनत ि देिे की सलाह देता है।
जब राष्ट्रपनत ससां द के पिु नवगचार के नलए नकसी नवधेयक को लौटाता है तो वह निलबां िकारी वीटो का प्योग करता है। हालााँनक, यनद
नवधेयक को ससां द द्वारा सश ां ोधिों के साथ या नबिा सश
ां ोधि के नफर से पाररत नकया जाता है और नफर से राष्ट्रपनत को प्स्ततु नकया जाता
है, तो राष्ट्रपनत के नलए नवधेयक पर अपिी सहमनत देिा अनिवायग है।
जब राष्ट्रपनत के नवचार के नलए राज्यपाल द्वारा एक नवधेयक सरु नक्षत रखा जाता है, तो राष्ट्रपनत के पास तीि नवकपप होते हैं (सांनवधाि
के अिुच्छे द 201 के तहत):
• वह नवधेयक पर अपिी सहमनत दे सकता है, या
• वह नबल पर अपिी सहमनत रोक सकता है, या
• वह राज्यपाल को राज्य नवधािमांडल के पिु नवगचार के नलए नवधेयक (यनद यह धि नवधेयक िहीं है) वापस करिे का निदेश दे
सकता है। यनद नवधेयक को राज्य नवधािमांडल द्वारा सांशोधिों के साथ या नबिा सांशोधि के नफर से पाररत कर नदया जाता है
और राष्ट्रपनत की सहमनत के नलए नफर से प्स्ततु नकया जाता है, तो राष्ट्रपनत नवधेयक पर अपिी सहमनत देिे के नलए बाध्य
िहीं होते हैं। इसका मतलब यह है नक राज्य नवधानयका राष्ट्रपनत की वीटो शनक्त को समाप्त िहीं कर सकती है।
इसनलए, राष्ट्रपनत राज्य नवधाि के सांबांध में पॉके ट वीटो का प्योग कर सकते हैं।

Q. 65) उपराष्ट्रपनत के चुिाव के निए निवााचक मडं ि में शानमि हैं:


1. सांसद के निवागनचत सदस्य।
2. राज्य नवधािसभाओ ां के निवागनचत सदस्य।
िीचे नदए गए कूटों में से सही उत्तर चुनिए:
a) के वल 1
b) के वल 2
c) दोिों 1 और 2

www.iasbaba.com rrs.iasbaba.com Page 53


IASbaba’s Rapid Revision (RaRe) Series Polity Day 1-120 2021
HINDI

d) ि तो 1 और ि ही 2
Q. 65) Solution (d)
उपराष्ट्रपनत का देश में दसू रा सवोच्च पद होता है। उन्हें वरीयता के आनधकाररक अनधपत्र में राष्ट्रपनत के बाद में एक रैं क नदया गया है।
यह पद अमेररकी उपराष्ट्रपनत की तजग पर नडजाइि नकया गया है।
उपराष्ट्रपनत, राष्ट्रपनत की तरह, सीधे लोगों द्वारा िहीं बनपक अप्त्यक्ष चिु ाव की पद्धनत से चिु ा जाता है। वह ससां द के दोिों सदिों के
सदस्यों वाले निवागचक मडां ल के सदस्यों द्वारा चिु ा जाता है। इस प्कार, यह निवागचक मडां ल निम्िनलनखत दो मामलों में राष्ट्रपनत के
चिु ाव के नलए निवागचक मडां ल से अलग है:
1. इसमें सांसद के निवागनचत और मिोिीत दोिों सदस्य होते हैं (राष्ट्रपनत के मामले में, के वल निवागनचत सदस्य)।
2. इसमें राज्य नवधािसभाओ ां के सदस्य शानमल िहीं हैं (राष्ट्रपनत के मामले में, राज्य नवधािसभाओ ां के निवागनचत सदस्य शानमल
हैं)।
दोिों ही मामलों में चिु ाव का तरीका एक ही है। इस प्कार, उपराष्ट्रपनत का चिु ाव, राष्ट्रपनत के चिु ाव की तरह, एकल सांिमणीय मत के
माध्यम से आिपु ानतक प्नतनिनधत्व प्णाली के अिुसार होता है और मतदाि गप्तु मतदाि द्वारा होता है।

Q. 66) निम्िनिनित कथि पर नवचार करें:


1. उपराष्ट्रपनत, राष्ट्रपनत पद तब ग्रहण करता है जब वह पद खाली हो जाता है और वह अपिे पवू गवती की अपणू ग अवनध तक के
नलए राष्ट्रपनत बिा रहता है।
2. राष्ट्रपनत के रूप में कायग करते हुए उपराष्ट्रपनत राज्य सभा के सभापनत के कायागलय के कतगव्यों का पालि िहीं करता है।
उपरोक्त में से कौि सा कथि सही है?
a) के वल 1
b) के वल 2
c) दोिों 1 और 2
d) ि तो 1 और ि ही 2
Q. 66) Solution (b)
हालानां क भारतीय उपराष्ट्रपनत का कायागलय अमेररकी उपराष्ट्रपनत की तजग पर बिाया गया है, लेनकि इसमें अतां र है। अमेररकी उपराष्ट्रपनत
पद ररक्त होिे पर राष्ट्रपनत पद ग्रहण करता है, और अपिे पवू गवती की अपणू ग अवनध तक के नलए राष्ट्रपनत बिा रहता है। दसू री ओर,
भारतीय उपराष्ट्रपनत, राष्ट्रपनत का पद ग्रहण िहीं करता है, जब वह अपणू ग अवनध के नलए ररक्त हो जाता है। वह के वल एक कायगवाहक
राष्ट्रपनत के रूप में कायग करता है जब तक नक िया राष्ट्रपनत कायगभार ग्रहण िहीं करता।

www.iasbaba.com rrs.iasbaba.com Page 54


IASbaba’s Rapid Revision (RaRe) Series Polity Day 1-120 2021
HINDI

उपराष्ट्रपनत राष्ट्रपनत के रूप में तब कायग करता है जब राष्ट्रपनत के कायागलय में उिके इस्तीफे , महानभयोग, मृत्यु या अन्यथा कारण ररनक्त
होती है। वह के वल छह महीिे की अनधकतम अवनध के नलए राष्ट्रपनत के रूप में कायग कर सकता है नजसके भीतर एक िया राष्ट्रपनत
चिु ा जािा होता है।
राष्ट्रपनत के रूप में कायग करते हुए या राष्ट्रपनत के कायों का निवगहि करते हुए, उपराष्ट्रपनत राज्य सभा के सभापनत के कायागलय के
कतगव्यों का पालि िहीं करता है। इस अवनध के दौराि, उि कतगव्यों का पालि राज्यसभा के उपसभापनत द्वारा नकया जाता है।

Q. 67) नकसी व्यनक्त को राज्य के राज्यपाि के रूप में नियक्त


ु करिे के निए सनं वधाि द्वारा क्या योग्यताएं निधााररत की गई
हैं?
1. वह भारत का िागररक होिा चानहए।
2. उसे 30 वषग की आयु परू ी कर लेिी चानहए।
3. उसे नवधाि सभा के सदस्य के रूप में चिु ाव के नलए योग्य होिा चानहए।
िीचे नदए गए कूटों में से सही उत्तर चुनिए:
a) के वल 1
b) के वल 1 और 2
c) के वल 3
d) 1, 2 और 3
Q. 67) Solution (a)
राज्यपाल राज्य का मख्ु य कायगकारी प्मख ु होता है। लेनकि, राष्ट्रपनत की तरह, वह िाममात्र का कायगकारी प्मख
ु (िाममात्र या सांवैधानिक
प्मखु ) होता है। राज्यपाल कें द्र सरकार के एजेंट के रूप में भी कायग करता है। इसनलए, राज्यपाल के कायागलय की दोहरी भनू मका होती
है।
राज्यपाल ि तो प्त्यक्ष रूप से जिता द्वारा चिु ा जाता है और ि ही परोक्ष रूप से नकसी नवशेष रूप से गनठत निवागचक मांडल द्वारा चिु ा
जाता है जैसा नक राष्ट्रपनत के मामले में होता है। उन्हें राष्ट्रपनत द्वारा उिके हाथ और महु र के तहत अनधपत्र द्वारा नियक्त
ु नकया जाता है।
एक तरह से वह कें द्र सरकार के िामनिनदगष्ट हैं।
राज्यपाल के रूप में नकसी व्यनक्त की नियनु क्त के नलए सांनवधाि के वल दो योग्यताएां निधागररत करता है। ये:
a) वह भारत का िागररक होिा चानहए।
b) उसे 35 वषग की आयु परू ी कर लेिी चानहए।
इसके अनतररक्त, नपछले कुछ वषों में इस सांबांध में दो कन्वेन्शि भी नवकनसत हुए हैं। सबसे पहले, वह एक बाहरी व्यनक्त होिा चानहए,
यािी वह उस राज्य से सांबांनधत िहीं होिा चानहए जहाां उसे नियक्त
ु नकया गया है, तानक वह स्थािीय राजिीनत से मक्त
ु हो। दसू रा, राज्यपाल

www.iasbaba.com rrs.iasbaba.com Page 55


IASbaba’s Rapid Revision (RaRe) Series Polity Day 1-120 2021
HINDI

की नियनु क्त करते समय, राष्ट्रपनत को सांबांनधत राज्य के मख्ु यमांत्री से परामशग करिे की आवश्यकता होती है, तानक राज्य में सांवैधानिक
तांत्र का सचु ारू सांचालि सनु िनित हो सके । हालानां क, कुछ मामलों में दोिों परांपराओ ां का उपलांघि नकया गया है।

Q. 68) राज्यपाि की कायाकारी शनक्तयों के संदभा में, निम्िनिनित कथिों पर नवचार करें:
1. वह राष्ट्रपनत को नकसी राज्य में सवां ैधानिक आपातकाल लगािे की नसफाररश कर सकता है।
2. महानधवक्ता हालानां क राज्यपाल द्वारा नियक्त
ु नकया जाता है, नजसे के वल राष्ट्रपनत द्वारा हटाया जा सकता है।
3. राज्य चिु ाव आयक्त
ु राज्यपाल के प्सादपयंत पद धारण करता है।
4. राज्य मनां त्रपररषद राज्यपाल के प्सादपयगन्त पद धारण करती है।
उपरोक्त में से कौि सा कथि सही हैं?
a) के वल 1 और 2
b) के वल 2 और 3
c) के वल 3 और 4
d) के वल 1 और 4
Q. 68) Solution (d)
राज्यपाल के पास भारत के राष्ट्रपनत के समाि कायगकारी, नवधायी, नवत्तीय और न्यानयक शनक्तयाां होती हैं। हालााँनक, उसके पास राष्ट्रपनत
की तरह कोई राजिनयक, सैन्य या आपातकालीि शनक्तयााँ िहीं हैं।
राज्यपाल की कायगकारी शनक्तयााँ और कायग हैं:
1. नकसी राज्य की सरकार के सभी कायगकारी कायग औपचाररक रूप से उसके िाम पर नकए जाते हैं।
2. वह उस तरीके को निनदगष्ट करते हुए नियम बिा सकता है नजसमें उसके िाम पर नकए गए और निष्ट्पानदत नकए गए आदेश और
अन्य साधि प्मानणत नकए जाएांगे।
3. वह नकसी राज्य सरकार के व्यवसाय के अनधक सनु वधाजिक लेि-देि और उक्त व्यवसाय के मांनत्रयों के बीच आवांटि के नलए
नियम बिा सकता है।
4. वह मख्ु यमांत्री और अन्य मांनत्रयों की नियनु क्त करता है। वे उसके प्सादपयंत पद धारण करते हैं।
5. वह एक राज्य के महानधवक्ता की नियनु क्त करता है और उसका पाररश्रनमक निधागररत करता है। महानधवक्ता राज्यपाल के
प्सादपयगन्त पद धारण करता है।
6. वह राज्य चिु ाव आयक्त ु की नियनु क्त करता है और इसकी सेवा की शतों और कायागलय के कायगकाल को निधागररत करता है।
हालाांनक, राज्य चिु ाव आयक्त
ु को उच्च न्यायालय के न्यायाधीश के समाि के वल उसी तरह और समाि आधार पर हटाया जा
सकता है।

www.iasbaba.com rrs.iasbaba.com Page 56


IASbaba’s Rapid Revision (RaRe) Series Polity Day 1-120 2021
HINDI

7. वह राज्य लोक सेवा आयोग के अध्यक्ष और सदस्यों की नियनु क्त करता है। हालााँनक, उन्हें के वल राष्ट्रपनत द्वारा हटाया जा
सकता है, राज्यपाल द्वारा िहीं।
8. वह मख्ु यमांत्री से राज्य के मामलों के प्शासि और कािूि के प्स्तावों से सांबांनधत कोई भी जािकारी प्ाप्त कर सकता है।
9. वह मख्ु यमत्रां ी को नकसी भी मामले पर मांनत्रपररषद के नवचार को प्स्तुत करिे की आवश्यकता कर सकता है, नजस पर एक
मत्रां ी द्वारा निणगय नलया गया है लेनकि नजस पर पररषद द्वारा नवचार िहीं नकया गया है।
10. वह राष्ट्रपनत को नकसी राज्य में सवां ैधानिक आपातकाल लगािे की नसफाररश कर सकता है। नकसी राज्य में राष्ट्रपनत शासि
की अवनध के दौराि, राज्यपाल को राष्ट्रपनत के एजेंट के रूप में व्यापक कायगकारी शनक्तयााँ प्ाप्त होती हैं।
11. वह राज्य में नवश्वनवद्यालयों के कुलानधपनत के रूप में कायग करता है। वह राज्य में नवश्वनवद्यालयों के कुलपनतयों की भी नियनु क्त
करता है।
Q. 69) राष्ट्रपनत निम्िनिनित नवशेष ज्ञाि या व्यावहाररक अिभ
ु व रििे वािे व्यनक्तयों में से राज्यसभा के बारह सदस्यों
को िानमत करता है:
1. सानहत्य
2. नवज्ञाि
3. कला
4. सहकारी आदां ोलि
5. समाज सेवा
िीचे नदए गए कूटों में से सही उत्तर चुनिए:
a) के वल 1, 2, 3 और 4
b) के वल 1, 2, 3 और 5
c) के वल 2, 3 और 5
d) 1, 2, 3, 4 और 5
Q. 69) Solution: (b)
राष्ट्रपनत राज्यसभा के 12 सदस्यों को सानहत्य, नवज्ञाि, कला और समाज सेवा में नवशेष ज्ञाि या व्यावहाररक अिभु व रखिे वाले
व्यनक्तयों में से िानमत करता है, जबनक राज्यपाल राज्य नवधाि पररषद के सदस्यों में से एक-छठे सदस्यों को सानहत्य, नवज्ञाि, कला,
सहकारी आांदोलि और समाज सेवा में नवशेष ज्ञाि या व्यावहाररक अिुभव रखिे वाले व्यनक्तयों में से िानमत करता है।

Q.70) राज्यपाि की क्षमादाि शनक्तयों के संदभा में, निम्िनिनित कथिों पर नवचार कीनजएः
1. वह मृत्यु दडां को माफ कर सकता है, निलनां बत कर सकता है, पररहार कर सकता है या लघक
ु रण कर सकता है।
2. वह कोटग-माशगल द्वारा सजा या दडां के सांबांध में क्षमा, दण्डनवराम, नवराम, निलांबि, पररहार या लघक
ु रण कर सकता है।

www.iasbaba.com rrs.iasbaba.com Page 57


IASbaba’s Rapid Revision (RaRe) Series Polity Day 1-120 2021
HINDI

उपरोक्त में से कौि सा कथि सही हैं?


a) के वल 1
b) के वल 2
c) दोिों 1 और 2
d) ि तो 1 और ि ही 2
Q. 70) Solution (d)
राज्यपाल की क्षमादाि शनक्तया:ां
• वह राज्य के कािूि के नखलाफ नकसी भी अपराध के नलए दोषी ठहराए गए नकसी भी व्यनक्त की सजा या दडां के सांबांध में क्षमा,
दण्डनवराम, नवराम, निलांबि, पररहार या लघक
ु रण कर सकता है।
• वह मृत्यु दडां को माफ िहीं कर सकता। यहाां तक नक अगर राज्य के कािूि में मृत्यु दडां का प्ावधाि है, तो माफी देिे की शनक्त
राष्ट्रपनत के पास होती है, ि नक राज्यपाल के पास। लेनकि, राज्यपाल मृत्यु दडां को निलांनबत, कम कर सकता है।
• उसके पास कोटग-माशगल (सैन्य न्यायालय) द्वारा सजा या दडां के सांबांध में क्षमा, दण्डनवराम, नवराम, निलांबि, पररहार या
लघक
ु रण के सांबांध में कोई शनक्त िहीं होती है। यह शनक्त के वल राष्ट्रपनत के पास होती है।
Q. 71) निम्िनिनित को ध्याि मे रिते हुए:
1. राष्ट्रीय आपदा प्बांधि प्ानधकरण
2. जैव प्ौद्योनगकी नवभाग (Department of Biotechnology)
3. राष्ट्रीय गांगा पररषद
4. राष्ट्रीय वन्यजीव बोडग (NBWL)
5. अांतररक्ष नवभाग
उपरोक्त में से नकि संगठिों की अध्यक्षता प्रधाि मंत्री द्वारा की जाती हैं?
a) के वल 1 और 3
b) के वल 2, 3 और 4
c) के वल 1, 3, 4 और 5
d) 1, 2, 3, 4 और 5
Q. 71) Solution (c)

www.iasbaba.com rrs.iasbaba.com Page 58


IASbaba’s Rapid Revision (RaRe) Series Polity Day 1-120 2021
HINDI

भारत के प्धाि मांत्री की अध्यक्षता में राष्ट्रीय आपदा प्बांधि प्ानधकरण (एिडीएमए), भारत में आपदा प्बांधि के नलए शीषग निकाय है।
एिडीएमए की स्थापिा और राज्य और नजला स्तर पर सांस्थागत तांत्र के नलए एक सक्षम वातावरण का निमागण आपदा प्बांधि अनधनियम,
2005 द्वारा अनिवायग है।
राष्ट्रीय गांगा पररषद का गठि पयागवरण (सांरक्षण) अनधनियम (EPA), 1986 के तहत नकया गया है। इसे गांगा और उसकी सहायक
िनदयों सनहत गांगा बेनसि के प्दषू ण की रोकथाम और कायाकपप के पयगवेक्षण के नलए समग्र नजम्मेदारी दी गई है।इसकी अध्यक्षता
प्धािमत्रां ी करते हैं।
राष्ट्रीय वन्यजीव बोडग का गठि 2002 में वन्यजीव अनधनियम के सश ां ोधि के माध्यम से नकया गया था। यह एक 47 सदस्यीय सनमनत
है, नजसकी अध्यक्षता प्धाि मत्रां ी और पयागवरण, वि और जलवायु पररवतगि मत्रां ी (पयागवरण मत्रां ी) उपाध्यक्ष के रूप में करते हैं।
अतां ररक्ष नवभाग की स्थापिा 1972 में तत्कालीि भारतीय अांतररक्ष कायगिम को बढ़ावा देिे के नलए की गई थी। यह भारतीय अांतररक्ष
आयोग की िीनतगत नसफाररशों पर कायग करता है और मोटे तौर पर भारत की अांतररक्ष गनतनवनधयों के नदि-प्नतनदि के प्बांधि के नलए
उत्तरदायी है। एक सनचव की अध्यक्षता में, जो इसरो का िेतत्ृ व भी करता है, नवभाग प्धाि मांत्री कायागलय (पीएमओ) के सीधे दायरे में
आता है। परांपरागत रूप से इस नवभाग के नलए उत्तरदायी मांत्री हमेशा प्धाि मत्रां ी होता है।यह नवभाग कै नबिेट में नकसी और को िहीं
बनपक स्वयां प्धािमांत्री को आवांनटत नकया जाता है।
जैव प्ौद्योनगकी नवभाग (डीबीटी) भारत में आधनु िक जीव नवज्ञाि और जैव प्ौद्योनगकी के क्षेत्र में नवकास और व्यावसायीकरण के
प्शासि के नलए नजम्मेदार है। इस नवभाग के नलए उत्तरदायी मत्रां ी कें द्रीय नवज्ञाि और प्ौद्योनगकी मत्रां ी हैं।

Q. 72) निम्िनिनित कथिों पर नवचार करें:


1. हालााँनक सांवैधानिक रूप से, प्धाि मांत्री सांसद के दोिों सदिों में से नकसी एक का सदस्य हो सकता है, लेनकि स्वतांत्रता के
बाद से भारत के सभी प्धाि मांत्री के वल लोकसभा के सदस्य रहे हैं।
2. एक व्यनक्त जो सांसद के नकसी भी सदि का सदस्य िहीं है, उसे छह महीिे के नलए प्धाि मांत्री के रूप में नियक्त
ु नकया जा
सकता है।
3. प्धाि मत्रां ी के चयि और नियनु क्त में राष्ट्रपनत कभी भी अपिे व्यनक्तगत निणगय का प्योग िहीं कर सकते हैं; उन्हें सबसे बडी
पाटी के िेता को प्धािमत्रां ी के रूप में नियक्त
ु करिा होता है।
उपरोक्त में से कौि सा कथि सही हैं?
a) के वल 1 और 2
b) के वल 2
c) के वल 3
d) 1, 2 और 3
Q. 72) Solution (b)

www.iasbaba.com rrs.iasbaba.com Page 59


IASbaba’s Rapid Revision (RaRe) Series Polity Day 1-120 2021
HINDI

सवां ैधानिक रूप से, प्धाि मांत्री सांसद के दोिों सदिों में से नकसी एक का सदस्य हो सकता है। उदाहरण के नलए, तीि प्धाि मांत्री, इनां दरा
गाांधी (1966), देवेगौडा (1996) और मिमोहि नसहां (2004) राज्यसभा के सदस्य थे।
एक व्यनक्त जो सांसद के नकसी भी सदि का सदस्य िहीं है, उसे छह महीिे के नलए प्धाि मांत्री के रूप में नियक्त
ु नकया जा सकता है,
नजसके भीतर उसे सांसद के नकसी भी सदि का सदस्य बििा चानहए; अन्यथा, वह प्धाि मांत्री के पद पर िहीं रहेगा।
सरकार की ससां दीय प्णाली की परांपराओ ां के अिसु ार, राष्ट्रपनत को लोकसभा में बहुमत दल के िेता को प्धाि मत्रां ी के रूप में नियक्त

करिा होता है। लेनकि, जब लोकसभा में नकसी भी दल के पास स्पष्ट बहुमत िहीं होता है, तब राष्ट्रपनत प्धािमत्रां ी के चयि और नियनु क्त
में अपिे व्यनक्तगत नववेक का प्योग कर सकता है। ऐसी नस्थनत में, राष्ट्रपनत आमतौर पर लोकसभा में सबसे बडे दल या गठबधां ि के िेता
को प्धाि मत्रां ी के रूप में नियक्त
ु करता है और उसे एक महीिे के भीतर सदि में नवश्वास मत प्ाप्त करिे के नलए कहता है। राष्ट्रपनत द्वारा
इस नववेक का प्योग 1979 में पहली बार नकया गया था, जब िीलम सजां ीव रे ड्डी (तत्कालीि राष्ट्रपनत) िे मोरारजी देसाई के िेतत्ृ व
वाली जिता पाटी सरकार के पति के बाद चरण नसहां (गठबधां ि िेता) को प्धाि मत्रां ी नियक्त
ु नकया था।
एक और नस्थनत यह भी है नक राष्ट्रपनत को प्धाि मांत्री के चयि और नियनु क्त में अपिे व्यनक्तगत निणगय का प्योग करिा पड सकता है,
अथागत, जब प्धाि मांत्री की अचािक मृत्यु हो जाती है और कोई स्पष्ट उत्तरानधकारी िहीं होता है। ऐसा ही तब हुआ जब 1984 में इनां दरा
गाांधी की हत्या कर दी गई थी। तत्कालीि राष्ट्रपनत जैल नसांह िे कायगवाहक प्धािमत्रां ी नियक्त
ु करिे की नमसाल को िजरअांदाज करते हुए
राजीव गाांधी को प्धािमत्रां ी नियक्तु नकया था। बाद में काांग्रेस सांसदीय दल िे सवगसम्मनत से उन्हें अपिा िेता चिु ा। हालााँनक, यनद, एक
मौजदू ा प्धाि मत्रां ी की मृत्यु पर, सत्ताधारी दल एक िए िेता का चिु ाव करता है, तो राष्ट्रपनत के पास उसे प्धाि मत्रां ी के रूप में नियक्त

करिे के अलावा कोई नवकपप िहीं होता है।

Q. 73) निम्िनिनित कथिों पर नवचार करें:


1. मनां त्रपररषद में प्धािमत्रां ी सनहत मनां त्रयों की कुल सख्ां या लोकसभा की कुल सख्ां या के 25% से अनधक िहीं होिी चानहए।
2. मनां त्रपररषद सामनू हक रूप से ससां द के प्नत उत्तरदायी होती है।
3. ससां द के नकसी भी सदि का सदस्य जो दलबदल के आधार पर अयोग्य घोनषत नकया जाता है, वह मत्रां ी के रूप में भी नियक्त

होिे के नलए अयोग्य होगा।
उपरोक्त में से कौि सा कथि सही हैं?
a) के वल 1 और 2
b) के वल 2
c) के वल 3
d) 1, 2 और 3
Q. 73) Solution (c)

www.iasbaba.com rrs.iasbaba.com Page 60


IASbaba’s Rapid Revision (RaRe) Series Polity Day 1-120 2021
HINDI

सरकार की सांसदीय प्णाली के नसद्धाांत सांनवधाि में नवस्तृत िहीं हैं, लेनकि दो अिुच्छे द (74 और 75) उिके साथ व्यापक, सांनक्षप्त और
सामान्य तरीके से निपटते हैं। अिच्ु छे द 74 मांनत्रपररषद की नस्थनत से सांबांनधत है जबनक अिुच्छे द 75 मांनत्रयों की नियनु क्त, कायगकाल,
नजम्मेदारी, योग्यता, शपथ और वेति और भत्ते से सांबांनधत है।
मांनत्रपररषद में प्धाि मांत्री सनहत मनां त्रयों की कुल सांख्या लोकसभा सदस्य की कुल सांख्या के 15% से अनधक िहीं होिी चानहए। यह
प्ावधाि 2003 के 91वें सांशोधि अनधनियम द्वारा जोडा गया था।
सरकार की ससां दीय प्णाली के कामकाज का मल ू नसद्धातां सामनू हक उत्तरदानयत्व का नसद्धातां है। अिच्ु छे द 75 में स्पष्ट रूप से कहा गया
है नक मनां त्रपररषद सामनू हक रूप से लोकसभा के प्नत उत्तरदायी होती है। इसका अथग यह है नक सभी मत्रां ी लोक सभा के प्नत सयां क्त ु
उत्तरदानयत्व रखते हैं। वे एक टीम के रूप में काम करते हैं और एक साथ तैरते या डूबते हैं। जब लोकसभा मनां त्रपररषद के नखलाफ
अनवश्वास प्स्ताव पाररत करती है, तो सभी मनां त्रयों को इस्तीफा देिा पडता है, नजसमें वे मत्रां ी भी शानमल हैं जो राज्यसभा से हैं।
नकसी भी राजिीनतक दल से सांबांनधत सांसद के नकसी भी सदि के सदस्य को दलबदल के आधार पर अयोग्य घोनषत नकया जाता है, वह
मांत्री के रूप में भी नियक्त
ु होिे के नलए अयोग्य होगा। यह प्ावधाि 2003 के 91वें सांशोधि अनधनियम द्वारा भी जोडा गया था।

Q. 74) निम्िनिनित कथिों पर नवचार करें:


1. मांनत्रमांडल सांसद के निचले सदि में मांनत्रपररषद के सामनू हक उत्तरदानयत्व को लागू करता है।
2. मांनत्रपररषद एक छोटा निकाय है नजसमें 15 से 20 मांत्री होते हैं जबनक कै नबिेट एक व्यापक निकाय होता है नजसमें 60 से 70
मांत्री होते हैं।
3. मांनत्रपररषद, एक निकाय के रूप में, अक्सर और आमतौर पर सप्ताह में एक बार बैठक करती है तानक सरकारी कामकाज के
लेि-देि के सबां धां में नवचार-नवमशग नकया जा सके और निणगय नलया जा सके ।
उपरोक्त में से कौि सा कथि सही हैं?
a) के वल 1
b) के वल 1 और 2
c) के वल 3
d) के वल 2 और 3
Q. 74) Solution (a)
'मांनत्रपररषद' और 'कै नबिेट' शब्दों का प्योग अक्सर एक दसू रे के स्थाि पर नकया जाता है, हालाांनक उिके बीच एक निनित अतां र है। वे
सांरचिा, कायग और भनू मका के मामले में एक दसू रे से नभन्ि होते हैं।
S.No. मांनत्रपररषद मांनत्रमांडल
1. यह एक व्यापक निकाय है नजसमें 60 से 70 मत्रां ी होते हैं। यह एक छोटा निकाय है नजसमें 15 से 20 मत्रां ी होते हैं।
2. इसमें मनां त्रयों की सभी तीि श्रेनणयाां शानमल हैं, यािी कै नबिेट इसमें के वल कै नबिेट मत्रां ी शानमल हैं। इस प्कार, यह
मांत्री, राज्य मांत्री, और उप मांत्री होते है। मांनत्रपररषद का एक नहस्सा है।
www.iasbaba.com rrs.iasbaba.com Page 61
IASbaba’s Rapid Revision (RaRe) Series Polity Day 1-120 2021
HINDI

3. यह एक निकाय के रूप में सरकारी कामकाज के नलए बैठक यह एक निकाय के रूप में, अक्सर और आमतौर पर सप्ताह
िहीं करता है। इसका कोई सामनू हक कायग िहीं है में एक बार सरकारी व्यवसाय के लेि-देि के सबां धां में
नवचार-नवमशग करिे और निणगय लेिे के नलए नमलते
है। इस प्कार, इसके सामनू हक कायग हैं।
4. यह सभी शनक्तयों के साथ निनहत है लेनकि नसद्धाांत रूप में यह, व्यवहार में, मनां त्रपररषद की शनक्तयों का प्योग करता
है ।
5. इसके कायग कै नबिेट द्वारा निधागररत नकए जाते हैं। यह सभी मांनत्रयों के नलए बाध्यकारी िीनतगत निणगय लेकर
मांनत्रपररषद को निदेश देता है।
6. यह कै नबिेट द्वारा नलए गए निणगयों को लागू करता है। यह मांनत्रपररषद द्वारा अपिे निणगय के कायागन्वयि की
निगरािी करता है

7. यह एक सांवैधानिक निकाय है, नजसे सांनवधाि के अिुच्छे द 74इसे 44वें सांनवधाि सांशोधि अनधनियम द्वारा 1978 में
और 75 द्वारा नवस्तार से पेश नकया गया है। हालााँनक, इसके सांनवधाि के अिुच्छे द 352 में सनम्मनलत नकया गया
आकार और वगीकरण का उपलेख सांनवधाि में िहीं नकया था। इस प्कार, इसे सांनवधाि के मल ू पाठ में स्थाि िहीं
गया है। इसका आकार प्धाि मत्रां ी द्वारा समय की नमला। अब भी, अिुच्छे द 352 के वल कै नबिेट को यह
आवश्यकताओ ां और नस्थनत की आवश्यकताओ ां के अिसु ार कहते हुए पररभानषत करता है नक यह 'प्धाि मत्रां ी और
निधागररत नकया जाता है। नत्र-स्तरीय निकाय में इसका अिच्ु छे द 75 के तहत नियक्त
ु कै नबिेट रैंक के अन्य मनां त्रयों
वगीकरण निटेि में नवकनसत ससां दीय सरकार की कन्वेन्शिों से नमलकर बिी पररषद' है और इसकी शनक्तयों और कायों
पर आधाररत है।हालानां क, इसे नवधायी मजां रू ी नमल गई है। का वणगि िहीं करता है। दसू रे शब्दों में, हमारी राजिीनतक-
प्शासनिक व्यवस्था में इसकी भनू मका निटेि में नवकनसत
ससां दीय सरकार की पररपाटी पर आधाररत है।
8. यह सांसद के निचले सदि के प्नत सामनू हक रूप से उत्तरदायी यह सांसद के निचले सदि में मनां त्रपररषद के सामनू हक
है। उत्तरदानयत्व को लागू करता है।

Q. 75) कै नबिेट सनमनतयों के सदं भा में, निम्िनिनित कथिों पर नवचार कीनजएः


1. वे अनतररक्त-सवां ैधानिक निकाय हैं।
2. नवशेष समस्याओ ां से निपटिे के नलए समय-समय पर तदथग सनमनतयों का गठि नकया जाता है।
3. प्धािमांत्री सभी कै नबिेट सनमनतयों का पदेि प्मख
ु (Ex-officio head) होता है
उपरोक्त में से कौि सा कथि सही हैं?
a) के वल 1 और 2
b) के वल 2
c) के वल 3
d) 1, 2 और 3

www.iasbaba.com rrs.iasbaba.com Page 62


IASbaba’s Rapid Revision (RaRe) Series Polity Day 1-120 2021
HINDI

Q. 75) Solution (a)


कै नबिेट सनमनतयों की नवशेषताएां निम्िनलनखत हैं:
1. वे उद्भव में अनतररक्त-सांवैधानिक हैं। दसू रे शब्दों में, उिका सांनवधाि में उपलेख िहीं है। हालानां क, व्यापार के नियमों में उिकी
स्थापिा का प्ावधाि है।
2. वे दो प्कार के होते हैं-स्थायी और तदथग। पवू ग स्थायी प्कृ नत के हैं जबनक बाद वाले अस्थायी प्कृ नत के हैं। नवशेष समस्याओ ां
से निपटिे के नलए समय-समय पर तदथग सनमनतयों का गठि नकया जाता है। काम परू ा होिे के बाद उन्हें भगां कर नदया जाता
है।
3. वे समय की आवश्यकताओ ां और नस्थनत की आवश्यकताओ ां के अिसु ार प्धाि मत्रां ी द्वारा स्थानपत नकए जाते हैं। इसनलए,
उिकी सख्ां या, िामकरण और सरां चिा समय-समय पर बदलती रहती है।
4. वे ज्यादातर प्धाि मत्रां ी के िेतत्ृ व में होती हैं। कभी-कभी अन्य कै नबिेट मत्रां ी, नवशेष रूप से गृह मत्रां ी या नवत्त मत्रां ी भी उिके
अध्यक्ष के रूप में कायग करते हैं। लेनकि, यनद प्धाि मत्रां ी नकसी सनमनत का सदस्य होता है, तो वह हमेशा इसकी अध्यक्षता
करता है।
5. उिकी सदस्यता तीि से आठ तक होती है। इिमें आमतौर पर के वल कै नबिेट मांत्री शानमल होते हैं। हालााँनक, गैर-कै नबिेट
मांनत्रयों को उिकी सदस्यता से वांनचत िहीं नकया जाता है।
6. इिमें ि के वल उिके द्वारा कवर नकए गए नवषयों के प्भारी मांत्री शानमल हैं बनपक अन्य वररष्ठ मांत्री भी शानमल होते हैं।
7. वे ि के वल मद्दु ों को सल
ु झाते हैं और कै नबिेट के नवचार के नलए प्स्ताव तैयार करते हैं, बनपक निणगय भी लेते हैं।हालाांनक
कै नबिेट उिके फै सलों की समीक्षा कर सकती है।
8. वे मनां त्रमांडल के भारी कायगभार को कम करिे के नलए एक सांगठिात्मक उपकरण हैं। वे िीनतगत मद्दु ों की गहि जाचां और
प्भावी समन्वय की सनु वधा भी प्दाि करते हैं। वे श्रम नवभाजि और प्भावी प्नतनिनधमांडल के नसद्धाांतों पर आधाररत हैं।

Q. 76) निम्िनिनित में से कौि सी सनमनत प्रधाि मंत्री की अध्यक्षता में होती है?
1. राजिीनतक मामलों की सनमनत
2. आनथगक मामलों की सनमनत
3. नियनु क्त सनमनत
4. सांसदीय कायग सनमनत
िीचे नदए गए कूटों में से सही उत्तर चुनिए:
a) के वल 2 और 4
b) के वल 2, 3 और 4
c) के वल 1 और 3

www.iasbaba.com rrs.iasbaba.com Page 63


IASbaba’s Rapid Revision (RaRe) Series Polity Day 1-120 2021
HINDI

d) के वल 1, 2 और 3
Q. 76) Solution (d)
महत्वपणू ग सनमनतयाां हैं:
1. राजिीनतक मामलों की सनमनत घरेलू और नवदेशी मामलों से सबां नां धत सभी िीनतगत मामलों को देखती है।
2. आनथगक मामलों की सनमनत आनथगक क्षेत्र में सरकारी गनतनवनधयों का निदेशि और समन्वय करती है।
3. नियनु क्त सनमनत कें द्रीय सनचवालय, सावगजनिक उद्यमों, बैंकों और नवत्तीय सस्ां थािों में सभी उच्च स्तरीय नियनु क्तयों का फै सला
करती है।
4. ससां दीय कायग सनमनत ससां द में सरकारी कायग की प्गनत को देखती है।
पहली तीि सनमनतयों की अध्यक्षता प्धाि मत्रां ी करते हैं और अनां तम की गृह मत्रां ी द्वारा की जाती है। सभी कै नबिेट सनमनतयों में, सबसे
शनक्तशाली राजिीनतक मामलों की सनमनत है, नजसे अक्सर "सपु र-कै नबिेट" के रूप में वनणगत नकया जाता है।

Q. 77) मुख्यमंत्री निम्िनिनित में से नकसके सदस्य होते हैं:


1. िीनत आयोग
2. आांचनलक पररषद (Zonal Council)
3. राष्ट्रीय कौशल नवकास पररषद
4. अांतरागज्यीय पररषद
िीचे नदए गए कूटों में से सही उत्तर चुनिए:
a) के वल 1, 2 और 3
b) के वल 1, 2 और 4
c) के वल 3 और 4
d) 1, 2, 3 और 4
Q. 77) Solution (b)
मख्ु यमांत्री बारी-बारी से सांबांनधत आांचनलक पररषद (Zonal Council) के उपाध्यक्ष के रूप में कायग करता है, एक समय में एक वषग की
अवनध के नलए पद धारण करता है।
मख्ु यमत्रां ी अतां र-राज्य पररषद और िीनत आयोग की शासी पररषद के सदस्य होते हैं, दोिों प्धाि मत्रां ी की अध्यक्षता में हैं।

www.iasbaba.com rrs.iasbaba.com Page 64


IASbaba’s Rapid Revision (RaRe) Series Polity Day 1-120 2021
HINDI

"कौशल नवकास के नलए समनन्वत कारग वाई और राष्ट्रीय कौशल नवकास निगम की स्थापिा" पर 15 मई 2008 को हुई बैठक में कै नबिेट
के निणगय के अिुसरण में प्धािमांत्री की राष्ट्रीय कौशल नवकास पररषद का गठि 1 जल
ु ाई 2008 को नकया गया था। इसकी अध्यक्षता
प्धािमत्रां ी करते हैं।
पररषद के सदस्य हैं:
• मािव ससां ाधि नवकास मत्रां ी
• नवत्त मत्रां ी
• भारी उद्योग और सावगजनिक उद्यम मत्रां ी
• ग्रामीण नवकास मत्रां ी
• आवास और शहरी गरीबी उपशमि मत्रां ी
• श्रम और रोजगार मत्रां ी
• उपाध्यक्ष, योजिा आयोग
• अध्यक्ष, राष्ट्रीय नवनिमागण प्नतस्पधागत्मकता पररषद
• राष्ट्रीय कौशल नवकास निगम के अध्यक्ष

Q. 78) राज्य मंनत्रपररषद के संदभा में, निम्िनिनित कथिों पर नवचार कीनजएः


1. एक राज्य में मख्ु यमत्रां ी सनहत मनां त्रयों की सांख्या 12 से कम िहीं होिी चानहए।
2. नबहार, छत्तीसगढ़, झारखांड, मध्य प्देश और ओनडशा राज्यों में आनदवासी कपयाण के प्भारी मांत्री की नियनु क्त एक सांवैधानिक
आवश्यकता है।
उपरोक्त में से कौि सा कथि सही हैं?
a) के वल 1
b) के वल 2
c) दोिों 1 और 2
d) ि तो 1 और ि ही 2
Q. 78) Solution (a)
नकसी राज्य में मांनत्रपररषद में मख्ु यमांत्री सनहत मांनत्रयों की कुल सांख्या उस राज्य की नवधाि सभा सदस्यों की कुल सांख्या के 15 प्नतशत
से अनधक िहीं होिी चानहए। लेनकि, एक राज्य में मख्ु यमत्रां ी सनहत मनां त्रयों की सांख्या 12 से कम िहीं होगी।यह प्ावधाि 2003 के 91वें
सांशोधि अनधनियम द्वारा जोडा गया था।

www.iasbaba.com rrs.iasbaba.com Page 65


IASbaba’s Rapid Revision (RaRe) Series Polity Day 1-120 2021
HINDI

छत्तीसगढ़, झारखांड, मध्य प्देश और ओनडशा राज्यों में आनदवासी कपयाण के प्भारी मत्रां ी की नियनु क्त एक सांवैधानिक आवश्यकता
है जो अिुसनू चत जानतयों और नपछडे वगों के कपयाण या नकसी अन्य कायग के अनतररक्त प्भारी हो सकते हैं। 2006 के 94वें सांशोधि
अनधनियम द्वारा नबहार राज्य को इस प्ावधाि से बाहर रखा गया था।
Q. 79) निम्िनिनित में से कौि सा यग्ु म सही सुमेनित है/हैं?
S. No. अिच्ु छे द सबां नां धत प्ावधाि
1 अिुच्छे द 75 राष्ट्रपनत की सहायता और सलाह देिे के नलए मांनत्रपररषद
2 अिुच्छे द 78 राष्ट्रपनत को सचू िा देिे के सांबांध में प्धािमांत्री के कतगव्य
3 अिुच्छे द 88 सदिों के सांबांध में मनां त्रयों के अनधकार

िीचे नदए गए कूटों में से सही उत्तर चुनिए:


a) के वल 1 और 2
b) के वल 2 और 3
c) के वल 1 और 3
d) 1, 2 और 3
Q. 79) Solution (b)
अिुच्छे द 74: राष्ट्रपनत को सहायता और सलाह देिे के नलए प्धाि मांत्री के साथ एक मांनत्रपररषद होगी, जो अपिे कायों के प्वतगि में
ऐसी सलाह के अिुसार कायग करेगा।
अिच्ु छे द 75:

• प्धाि मांत्री की नियनु क्त राष्ट्रपनत द्वारा की जाएगी और अन्य मनां त्रयों की नियनु क्त राष्ट्रपनत द्वारा प्धाि मत्रां ी की सलाह पर की
जाएगी;
• राष्ट्रपनत के प्सादपयंत मांत्री पद धारण करेंगे; तथा
• मांनत्रपररषद सामनू हक रूप से लोक सभा के प्नत उत्तरदायी होगी।
अिुच्छे द 78: प्धािमत्रां ी का यह कतगव्य होगा:

• राष्ट्रपनत को सघां के मामलों के प्शासि और काििू के प्स्तावों से सबां नां धत मनां त्रपररषद के सभी निणगयों से अवगत करािा;
• सघां के मामलों के प्शासि और काििू के प्स्तावों से सबां नां धत ऐसी जािकारी प्स्ततु करिे के नलए, जो राष्ट्रपनत मागां े; तथा
• यनद राष्ट्रपनत को आवश्यकता हो, तो नकसी भी मामले को मनां त्रपररषद के नवचार के नलए प्स्ततु करिा, नजस पर एक मत्रां ी द्वारा
निणगय नलया गया हो, लेनकि नजस पर पररषद द्वारा नवचार िहीं नकया गया हो।
अिच्ु छे द 88: भारत के प्त्येक मत्रां ी और महान्यायवादी को नकसी भी सदि, सदिों की नकसी भी सयां क्त
ु बैठक और ससां द की नकसी भी
सनमनत, नजसमें वह हो सकता है, की कायगवाही में बोलिे और अन्यथा भाग लेिे का अनधकार होगा। लेनकि इस अिच्ु छे द के आधार पर
मत देिे का हकदार िहीं होगा।

www.iasbaba.com rrs.iasbaba.com Page 66


IASbaba’s Rapid Revision (RaRe) Series Polity Day 1-120 2021
HINDI

Q.80) राज्यपाि और मुख्यमंत्री के बीच संबध


ं ों से संबंनधत संनवधाि के निम्िनिनित प्रावधािों में से कौि-सा/से सही है/हैं?
1. अिुच्छे द 163: राज्यपाल की सहायता और सलाह देिे के नलए मख्ु यमांत्री के साथ एक मांनत्रपररषद होगी
2. अिुच्छे द 164 : मत्रां ी राज्यपाल के प्सादपयगन्त पद धारण करेंगे।
3. अिुच्छे द 167: मांनत्रपररषद राज्य की नवधाि सभा के प्नत सामनू हक रूप से उत्तरदायी होगी
िीचे नदए गए कूटों में से सही उत्तर चुनिए:
a) के वल 1
b) के वल 1 और 2
c) के वल 3
d) के वल 2 और 3
Q. 80) Solution (b)
सांनवधाि के निम्िनलनखत प्ावधाि राज्यपाल और मख्ु यमांत्री के बीच सांबांधों से सांबांनधत हैं:
1. अिुच्छे द 163:
राज्यपाल को अपिे कायों के अभ्यास में सहायता और सलाह देिे के नलए मख्ु यमत्रां ी के साथ एक मांनत्रपररषद होगी, नसवाय इसके नक
जहाां तक उसे अपिे कायों या उिमें से नकसी को अपिे नववेक से करिे की आवश्यकता हो।
2. अिुच्छे द 164:
a. मख्ु यमांत्री की नियनु क्त राज्यपाल द्वारा की जाएगी और अन्य मांनत्रयों की नियनु क्त राज्यपाल द्वारा मख्ु यमत्रां ी की सलाह पर की
जाएगी;
b. राज्यपाल के प्सादपयंत मत्रां ी पद धारण करें गे; तथा
c. मनां त्रपररषद राज्य की नवधाि सभा के प्नत सामनू हक रूप से उत्तरदायी होगी।
3. अिच्ु छे द 167: मख्ु यमत्रां ी का यह कतगव्य होगा:
a. राज्य के मामलों के प्शासि और कािूि के प्स्तावों से सांबांनधत मांनत्रपररषद के सभी निणगयों को राज्य के राज्यपाल को सनू चत
करिा;
b. राज्य के मामलों के प्शासि और कािूि के प्स्तावों से सांबांनधत ऐसी जािकारी प्स्ततु करिे के नलए जो राज्यपाल माांगे; तथा
c. यनद राज्यपाल को आवश्यकता हो, तो नकसी भी मामले को मांनत्रपररषद के नवचार के नलए प्स्ततु करिा, नजस पर एक मत्रां ी
द्वारा निणगय नलया गया हो, लेनकि नजस पर पररषद द्वारा नवचार िहीं नकया गया हो।

www.iasbaba.com rrs.iasbaba.com Page 67


IASbaba’s Rapid Revision (RaRe) Series Polity Day 1-120 2021
HINDI

Q. 81) निम्िनिनित में से कौि-सा/से कारक हैं जो भारतीय सस


ं द की सप्रं भुता को सीनमत करते हैं?
1. मौनलक अनधकार
2. सरकार की सांघीय व्यवस्था
3. नलनखत सांनवधाि
िीचे नदए गए कूटों में से सही उत्तर चुनिए:
a) के वल 1
b) के वल 1 और 2
c) के वल 3
d) 1, 2 और 3
Q. 81) Solution (d)
'ससां द की सांप्भतु ा' का नसद्धाांत निनटश सांसद से जडु ा है। सांप्भतु ा का अथग राज्य के भीतर सवोच्च शनक्त है । ग्रेट निटेि में वह सवोच्च
शनक्त सांसद के पास है। इसके प्ानधकार और अनधकार क्षेत्र पर कोई 'कािूिी' प्नतबांध िहीं हैं।
भारतीय सांसद को इस अथग में एक सांप्भु निकाय िहीं मािा जा सकता है क्योंनक इसके प्ानधकार और अनधकार क्षेत्र पर 'कािूिी' प्नतबांध
हैं। भारतीय सांसद की सांप्भतु ा को सीनमत करिे वाले कारक हैं:
• मौनलक अनधकार: सांनवधाि के भाग 3 के तहत न्यायालयों योग्य मल ू अनधकारों की सांनहता को शानमल करके सांसद के
प्ानधकार को भी प्नतबनां धत नकया गया है। अिुच्छे द 13 राज्य को ऐसा कािूि बिािे से रोकता है जो या तो परू ी तरह से छीि
लेता है या आांनशक रूप से मौनलक अनधकार को समाप्त कर देता है। इसनलए, मौनलक अनधकारों का उपलांघि करिे वाला
ससां दीय काििू शन्ू य होगा।
• सरकार की सघां ीय प्णाली: भारत में सरकार की एक सघां ीय प्णाली है नजसमें सघां और राज्यों के बीच शनक्तयों का सवां ैधानिक
नवभाजि है। दोिों को आवनां टत क्षेत्रों के भीतर काम करिा है। इसनलए, ससां द का काििू बिािे का अनधकार सघां सचू ी और
समवती सचू ी में शानमल नवषयों तक ही सीनमत हो जाता है और राज्य सचू ी में शानमल नवषयों तक इसका नवस्तार िहीं होता
है।
• सनां वधाि की नलनखत प्कृ नत: सनां वधाि हमारे देश में भनू म का मौनलक काििू है। इसिे कें द्र सरकार के तीिों अगां ों के अनधकार
और अनधकार क्षेत्र और उिके बीच अतां संबधां की प्कृ नत को पररभानषत नकया है। इसनलए, ससां द को सनां वधाि द्वारा निधागररत
सीमाओ ां के भीतर काम करिा है। नवधायी प्ानधकरण और सांसद के घटक प्ानधकरण के बीच कािूिी अांतर भी है। इसके
अलावा, सांनवधाि में कुछ सांशोधिों को लागू करिे के नलए, आधे राज्यों के अिुसमथगि की भी आवश्यकता है।

Q. 82) संसदीय नवशेषानधकारों (parliamentary privileges) के संदभा में निम्िनिनित कथिों पर नवचार कीनजएः
1. सांसद के सत्र के दौराि नकसी भी दीवािी या फौजदारी मामले में सांसद सदस्य को नगरफ्तार िहीं नकया जा सकता है।

www.iasbaba.com rrs.iasbaba.com Page 68


IASbaba’s Rapid Revision (RaRe) Series Polity Day 1-120 2021
HINDI

2. कोई भी सांसद सदस्य सांसद या उसकी सनमनतयों में उसके द्वारा कही गई नकसी बात या उसके द्वारा नदए गए नकसी भी वोट के
नलए नकसी भी अदालत में नकसी भी कायगवाही के नलए उत्तरदायी िहीं है।
3. जब सांसद सत्र चल रहा हो तो सांसद सदस्य अदालत में लांनबत मामले में साक्ष्य देिे से और गवाह के रूप में पेश होिे से इिकार
कर सकता है।
उपरोक्त में से कौि सा कथि सही हैं?
a) के वल 1 और 2
b) के वल 2 और 3
c) के वल 1 और 3
d) 1, 2 और 3
Q. 82) Solution (b)
ससां दीय नवशेषानधकार सांसद के दोिों सदिों, उिकी सनमनतयों और उिके सदस्यों द्वारा प्ाप्त नवशेष अनधकार, उन्मनु क्तयाां और छूट हैं। यह
उन्हें अपिे कायों की स्वतांत्रता और प्भावशीलता को सरु नक्षत रखिे के नलए आवश्यक हैं।
इि नवशेषानधकारों के नबिा, सदि ि तो अपिे अनधकार, गररमा और सम्माि को बिाए रख सकते हैं और ि ही अपिे सदस्यों को उिकी
सांसदीय नजम्मेदाररयों के निवगहि में नकसी भी बाधा से बचा सकते हैं।
सांसदीय नवशेषानधकारों को दो व्यापक श्रेनणयों में वगीकृ त नकया जा सकता है:
• जो सांसद के प्त्येक सदि द्वारा सामनू हक रूप से आिांद नलया जाता है, और
• नजिका सदस्यों द्वारा व्यनक्तगत रूप से लाभ उठाया जाता है
सामनू हक रूप से ससां द के प्त्येक सदि से सबां नां धत नवशेषानधकार हैं:
1. इसे अपिी ररपोटग, वाद-नववाद और कायगवानहयों को प्कानशत करिे का अनधकार है और दसू रों को इसे प्कानशत करिे से
रोकिे का भी अनधकार है।
2. यह बाहरी व्यनक्तयों को अपिी कायगवाही से बाहर कर सकता है और कुछ महत्वपणू ग मामलों पर चचाग करिे के नलए गप्तु बैठकें
कर सकता है।
3. यह अपिी प्निया और अपिे व्यवसाय के सांचालि को नवनियनमत करिे और ऐसे मामलों पर निणगय लेिे के नलए नियम बिा
सकता है।
4. यह सदस्यों के साथ-साथ बाहरी लोगों को अपिे नवशेषानधकारों के उपलांघि के नलए या उसकी अवमाििा के नलए फटकार,
चेताविी या कारावास (सदस्यों के मामले में निलांबि या निष्ट्कासि) द्वारा दनां डत कर सकता है।
5. इसे नकसी सदस्य की नगरफ्तारी, िजरबांदी, दोषनसनद्ध, कारावास और ररहाई की तत्काल सचू िा प्ाप्त करिे का अनधकार है।

www.iasbaba.com rrs.iasbaba.com Page 69


IASbaba’s Rapid Revision (RaRe) Series Polity Day 1-120 2021
HINDI

6. यह पछू ताछ कर सकता है और गवाहों की उपनस्थनत का आदेश दे सकता है और प्ासांनगक कागजात और ररकॉडग भेज सकता
है।
7. न्यायालयों को नकसी सदि या उसकी सनमनतयों की कायगवाही की जाचां करिे की मिाही है।
8. पीठासीि अनधकारी की अिुमनत के नबिा नकसी भी व्यनक्त (या तो सदस्य या बाहरी व्यनक्त) को नगरफ्तार िहीं नकया जा सकता
है, और सदि के पररसर के भीतर कोई काििू ी प्निया (नसनवल या आपरानधक) िहीं की जा सकती है।
व्यनक्तगत रूप से सदस्यों से सबां नां धत नवशेषानधकार हैं:
1. उन्हें सांसद के सत्र के दौराि और सत्र शरू
ु होिे से 40 नदि पहले और सत्र की समानप्त के 40 नदि बाद नगरफ्तार िहीं नकया जा
सकता है। यह नवशेषानधकार के वल दीवािी मामलों में उपलब्ध है, ि नक आपरानधक मामलों या निवारक निरोध के मामलों
में।
2. उन्हें सांसद में बोलिे की आजादी है। कोई भी सदस्य सांसद या उसकी सनमनतयों में उसके द्वारा कही गई नकसी बात या उसके
द्वारा नदए गए वोट के नलए नकसी भी अदालत में नकसी कायगवाही के नलए उत्तरदायी िहीं है। यह स्वतत्रां ता सांनवधाि के प्ावधािों
और सांसद की प्निया को नवनियनमत करिे वाले नियमों और स्थायी आदेशों के अधीि है।
3. उन्हें जरू ी सेवा से छूट दी गई है। जब सांसद सत्र चल रहा हो तो सांसद सदस्य अदालत में लांनबत मामले में साक्ष्य देिे से और
गवाह के रूप में पेश होिे से इिकार कर सकता है।

Q. 83) निम्िनिनित में से कौि-सी शनक्तयाँ के वि राज्य सभा को प्राप्त हैं?


1. यह सांसद को राज्य सचू ी में उनपलनखत नवषय पर कािूि बिािे के नलए अनधकृ त कर सकता है।
2. यह अके ले ही राष्ट्रपनत को हटािे की पहल कर सकता है।
3. यह सवोच्च न्यायालय के अनधकार क्षेत्र के नवस्तार के सांबांध में एकमात्र प्ानधकरण है।
िीचे नदए गए कूटों में से सही उत्तर चुनिए:
a) के वल 1
b) के वल 2 और 3
c) के वल 1 और 3
d) 1, 2 और 3
Q. 83) Solution (a)
राज्य सभा को चार नवनशष्ट या नवशेष शनक्तयााँ दी गई हैं जो लोकसभा को प्ाप्त िहीं हैं:
1. यह सांसद को राज्य सचू ी (अिच्ु छे द 249) में सचू ीबद्ध नवषय पर कािूि बिािे के नलए अनधकृ त कर सकता है।
2. यह सांसद को कें द्र और राज्यों दोिों के नलए िई अनखल भारतीय सेवाएां बिािे के नलए अनधकृ त कर सकता है (अिुच्छे द 312)।

www.iasbaba.com rrs.iasbaba.com Page 70


IASbaba’s Rapid Revision (RaRe) Series Polity Day 1-120 2021
HINDI

3. यह अके ले ही उप-राष्ट्रपनत को हटािे का फै सला ले सकता है। दसू रे शब्दों में, उपराष्ट्रपनत को हटािे का प्स्ताव के वल राज्यसभा में
पेश नकया जा सकता है, ि नक लोकसभा में (अिुच्छे द 67)।
4. यनद राष्ट्रपनत द्वारा राष्ट्रीय आपातकाल या राष्ट्रपनत शासि या नवत्तीय आपातकाल लागू करिे की घोषणा ऐसे समय में की जाती है
जब लोकसभा भगां हो गई हो या लोकसभा का नवघटि उसके अिमु ोदि के नलए अिमु त अवनध के भीतर हो, तो उद्घोषणा अके ले राज्य
सभा द्वारा अिमु ोनदत होिे पर भी प्भावी रह सकता है (अिुच्छे द 352, 356 और 360)।
हमारी सवां ैधानिक व्यवस्था में राज्य सभा की नस्थनत उतिी कमजोर िहीं है नजतिी नक निनटश सवां ैधानिक व्यवस्था में हाउस ऑफ
लॉड्गस की या अमेररकी सवां ैधानिक व्यवस्था में सीिेट की नस्थनत नजतिी मजबतू है। नवत्तीय मामलों और मनां त्रपररषद पर नियत्रां ण को
छोडकर, अन्य सभी क्षेत्रों में राज्य सभा की शनक्तयााँ और नस्थनत मोटे तौर पर समाि हैं और लोकसभा के साथ समन्वय करती हैं।

Q. 84) निम्िनिनित कथिों पर नवचार करें:


1. भनवष्ट्य निनध जमा, बचत बैंक जमा और सरकार द्वारा रेजरी नबल जारी करके एकनत्रत नकए गए ऋण को भारत की सांनचत
निनध में रखा जाता है।
2. भारत के लोक लेखा (Public Account) से भगु ताि सांसदीय नवनियोग के नबिा नकए जा सकते हैं।
3. भारत सरकार की ओर से सभी कािूिी रूप से अनधकृ त भगु ताि भारत की आकनस्मकता निनध से नकए जाते हैं।
उपरोक्त में से कौि सा कथि सही हैं?
a) के वल 1 और 2
b) के वल 2
c) के वल 3
d) के वल 2 और 3
Q. 84) Solution (b)
भारत का सांनवधाि कें द्र सरकार के नलए निम्िनलनखत तीि प्कार की निनधयों का प्ावधाि करता है:
1. भारत की सनां चत निनध (अिच्ु छे द 266)
2. भारत का लोक लेखा (अिच्ु छे द 266)
3. भारत की आकनस्मकता निनध (अिुच्छे द 267)
भारत की सांनचत/ समेनकत निनध:
यह एक ऐसा कोष है नजसमें सभी प्ानप्तयों को जमा नकया जाता है और सभी भगु तािों को डेनबट नकया जाता है। दसू रे शब्दों में:
• भारत सरकार द्वारा प्ाप्त सभी राजस्व;

www.iasbaba.com rrs.iasbaba.com Page 71


IASbaba’s Rapid Revision (RaRe) Series Polity Day 1-120 2021
HINDI

• सरकार द्वारा रेजरी नबल, ऋण या अथोपाय अनग्रम और साधि जारी करके एकनत्रत नकए गए सभी ऋण;
• ऋणों के पिु भगगु ताि में सरकार द्वारा प्ाप्त सभी धि भारत की सांनचत निनध का निमागण करते हैं।
भारत सरकार की ओर से सभी कािूिी रूप से अनधकृ त भगु ताि इस फांड से नकए जाते हैं। इस निनध में से कोई भी धि सांसदीय कािूि के
नबिा नवनियोनजत (जारी या आहररत) िहीं नकया जा सकता है।
भारत का सावगजनिक खाता:
• भारत सरकार द्वारा या उसकी ओर से प्ाप्त अन्य सभी सावगजनिक धि (उिके अलावा जो भारत की सनां चत निनध में जमा नकए
गए हैं) को भारत के लोक खाते में जमा नकया जाएगा।
• इसमें भनवष्ट्य निनध जमा, न्यानयक जमा, बचत बैंक जमा, नवभागीय जमा, प्ेषण (remittances) आनद शानमल हैं।
• यह खाता कायगकारी कारग वाई द्वारा सच
ां ानलत होता है, अथागत इस खाते से भगु ताि ससां दीय नवनियोग के नबिा नकया जा सकता
है। इस तरह के भगु ताि ज्यादातर बैंनकांग लेिदेि की प्कृ नत में होते हैं।
भारत की आकनस्मकता निनध:
• सनां वधाि िे सांसद को 'भारत की आकनस्मकता निनध' स्थानपत करिे के नलए अनधकृ त नकया, नजसमें समय-समय पर कािूि
द्वारा निधागररत रानश का भगु ताि नकया जाता है।तद्नसु ार, सांसद िे 1950 में भारतीय आकनस्मकता निनध अनधनियम बिाया।
• यह कोष राष्ट्रपनत के पास रखा जाता है और वह सांसद द्वारा इसकी अिुमनत नमलिे तक अप्त्यानशत व्यय को परू ा करिे के
नलए इसमें से अनग्रम रानश जमा कर सकता है।
• यह कोष राष्ट्रपनत की ओर से नवत्त सनचव के पास होता है। भारत के सावगजनिक खाते की तरह, यह भी कायगकारी कारग वाई
द्वारा सांचानलत होता है।

Q. 85) अप्रत्यानशत पररनस्थनतयों को परू ा करिे के निए सस


ं द द्वारा नदए गए नवनभन्ि अिदु ािों के सबं ध
ं में निम्िनिनित
कथिों पर नवचार करें:
1. अनतररक्त अिुदाि (Additional Grant) तब नदया जाता है जब सेवा के पररमाण या अनिनित स्वरूप के कारण, आम तौर
पर बजट में नदए गए नववरण के साथ मागां को िहीं बताया जा सकता है।
2. प्त्यािुदाि (Vote of Credit) तब नदया जाता है जब चालू नवत्तीय वषग के दौराि नकसी िई सेवा पर अनतररक्त व्यय की
आवश्यकता उत्पन्ि होती है, नजस पर उस वषग के बजट में नवचार िहीं नकया जाता है।
3. अिुपरू क अिुदाि (Supplementary Grant) तब प्दाि नकया जाता है जब सांसद द्वारा नवनियोग अनधनियम के माध्यम से
चालू नवत्तीय वषग के नलए नकसी नवशेष सेवा के नलए अनधकृ त रानश उस वषग के नलए अपयागप्त पाई जाती है।
उपरोक्त में से कौि सा कथि सही हैं?
a) के वल 1
b) के वल 1 और 2

www.iasbaba.com rrs.iasbaba.com Page 72


IASbaba’s Rapid Revision (RaRe) Series Polity Day 1-120 2021
HINDI

c) के वल 3
d) के वल 2 और 3
Q. 85) Solution (c)
एक नवत्तीय वषग के नलए आय और व्यय के सामान्य अिुमाि वाले बजट के अलावा, सांसद द्वारा असाधारण या नवशेष पररनस्थनतयों में
कई अन्य अिुदाि नदए जाते हैं:
• अनतररक्त अिदु ाि: यह तब प्दाि नकया जाता है जब चालू नवत्तीय वषग के दौराि उस वषग के बजट में नवचार िहीं की गई नकसी
िई सेवा पर अनतररक्त व्यय की आवश्यकता उत्पन्ि होती है।
• प्त्यािदु ाि (Vote of Credit): यह भारत के ससां ाधिों पर एक अप्त्यानशत मागां को परू ा करिे के नलए नदया जाता है, जब
सेवा के पररमाण या अनिनित प्कृ नत के कारण, मागां को आमतौर पर बजट में नदए गए नववरण के साथ िहीं बताया जा सकता
है। अत: यह लोक सभा द्वारा कायगपानलका को नदए गए ब्लैंक चेक के समाि है।
• अिुपरू क अिुदाि (Supplementary Grant) : यह तब नदया जाता है जब सांसद द्वारा नवनियोग अनधनियम के माध्यम से
चालू नवत्तीय वषग के नलए नकसी नवशेष सेवा के नलए अनधकृ त रानश उस वषग के नलए अपयागप्त पाई जाती है।
• अनतररक्त अिदु ाि (Excess Grant): यह तब नदया जाता है जब नकसी नवत्तीय वषग के दौराि नकसी सेवा पर उस वषग के बजट
में उस सेवा के नलए दी गई रानश से अनधक रानश खचग की जाती है। नवत्तीय वषग के बाद लोकसभा द्वारा इसे वोट नदया जाता है।
मतदाि के नलए लोकसभा में अनतररक्त अिदु ाि की मागां प्स्तुत करिे से पहले, उन्हें सांसद की लोक लेखा सनमनत द्वारा
अिुमोनदत नकया जािा चानहए।
• असाधारण अिुदाि (Exceptional Grant) : यह एक नवशेष उद्देश्य के नलए नदया जाता है और नकसी भी नवत्तीय वषग की
वतगमाि सेवा का नहस्सा िहीं होता है।
• टोकि अिुदाि (Token Grant): यह तब नदया जाता है जब नकसी िई सेवा पर प्स्तानवत व्यय को परू ा करिे के नलए धि
पिु नवगनियोजि द्वारा उपलब्ध कराया जा सकता है। एक साांकेनतक रानश के अिुदाि की माांग (रू 1 का) लोकसभा के वोट के
नलए प्स्तुत की जाती है और यनद सहमनत हो जाती है, तो धि उपलब्ध कराया जाता है। पिु नवगनियोग में एक मद से दसू रे मद
में निनधयों का अांतरण शानमल है।इसमें कोई अनतररक्त खचग िहीं होता है।

Q. 86) बजट में भारत की संनचत निनध पर 'प्रभाररत' व्यय और भारत की संनचत निनध से नकए गए व्यय को शानमि नकया
जाता है। इस संदभा में निम्िनिनित कथिों पर नवचार कीनजएः
1. भारत की सांनचत निनध से नकया गया व्यय सांसद द्वारा गैर-मतदाि योग्य है।
2. उच्च न्यायालय के न्यायाधीशों के वेति, भत्ते और पेंशि भारत की सनां चत निनध पर भाररत होते हैं।
उपरोक्त में से कौि सा कथि सही हैं?
a) के वल 1

www.iasbaba.com rrs.iasbaba.com Page 73


IASbaba’s Rapid Revision (RaRe) Series Polity Day 1-120 2021
HINDI

b) के वल 2
c) दोिों 1 और 2
d) ि तो 1 और ि ही 2
Q. 86) Solution (d)
बजट में दो प्कार के व्यय होते हैं-भारत की सांनचत निनध पर 'प्भाररत' व्यय और भारत की सांनचत निनध से नकया गया व्यय। भाररत व्यय
सांसद द्वारा गैर-मतदाि योग्य है, अथागत इस पर के वल सांसद द्वारा चचाग की जा सकती है, जबनक दसू रे प्कार के नलए सांसद द्वारा मतदाि
नकया जािा है।
भारत की सनां चत निनध पर के वल उच्च न्यायालयों के न्यायाधीशों की पेंशि ली जाती है। उच्च न्यायालय के न्यायाधीशों के वेति और
भत्ते सबां नां धत राज्यों की सनां चत निनध पर भाररत होते हैं।
Q. 87) निम्िनिनित नवधेयक से संबंनधत गनतरोध को समाप्त करिे के निए राष्ट्रपनत संसद के दोिों सदिों को सयं ुक्त बैठक
में बुिा सकता है:
a) साधारण नवधेयक
b) नवत्तीय नबल
c) धि नवधेयक
d) सांनवधाि सांशोधि नवधेयक
िीचे नदए गए कूटों में से सही उत्तर चुनिए:
• के वल 1 और 2
• के वल 1, 2 और 3
• के वल 2, 3 और 4
• 1, 2, 3 और 4
Q. 87) Solution (a)
सयां क्त
ु बैठक नकसी नवधेयक को पाररत करिे के बाद दोिों सदिों के बीच गनतरोध के समाधाि के नलए सांनवधाि द्वारा प्दत्त एक असाधारण
तांत्र है। एक सदि द्वारा एक नवधेयक को पाररत करिे और दसू रे सदि को प्ेनषत करिे के बाद निम्िनलनखत तीि नस्थनतयों में से नकसी एक
के तहत एक गनतरोध हुआ मािा जाता है:
• यनद नबल दसू रे सदि द्वारा खाररज कर नदया जाता है;
• यनद नबल में नकए जािे वाले सांशोधिों के बारे में सदिों िे अांततः असहमनत जताई है; या
• यनद दसू रे सदि द्वारा नवधेयक पाररत नकए नबिा नवधेयक की प्ानप्त की तारीख से छह महीिे से अनधक बीत चक
ु े हैं।

www.iasbaba.com rrs.iasbaba.com Page 74


IASbaba’s Rapid Revision (RaRe) Series Polity Day 1-120 2021
HINDI

उपरोक्त तीि नस्थनतयों में, राष्ट्रपनत नवधेयक पर नवचार-नवमशग करिे और मतदाि करिे के उद्देश्य से दोिों सदिों को सांयक्त
ु बैठक में बल
ु ा
सकता है।
यहाां यह ध्याि नदया जािा चानहए नक सांयक्त
ु बैठक का प्ावधाि के वल साधारण नवधेयकों या नवत्तीय नवधेयकों पर लागू होता है ि नक
धि नवधेयकों या सांनवधाि सांशोधि नवधेयकों पर।
धि नवधेयक के मामले में, लोकसभा के पास अनधभावी शनक्तयाां होती हैं, जबनक एक सवां ैधानिक सश
ां ोधि नवधेयक को प्त्येक सदि
द्वारा पृथक रूप से पाररत नकया जािा चानहए।

Q. 88) एक नवधेयक को धि नवधेयक मािा जाता है यनद इसमें निम्िनिनित से संबनं धत प्रावधाि हैं:
1. भारत की सनां चत निनध से धि का नवनियोग।
2. स्थािीय प्ानधकरण द्वारा नकसी भी कर का अनधरोपण, उन्मल
ू ि, छूट, पररवतगि या नवनियमि।
3. कें द्र सरकार द्वारा धि उधार लेिे का नवनियमि।
िीचे नदए गए कूटों में से सही उत्तर चुनिए:
a) के वल 1 और 2
b) के वल 2 और 3
c) के वल 1 और 3
d) 1, 2 और 3
Q. 88) Solution (c)
सनां वधाि का अिुच्छे द 110 धि नवधेयकों की पररभाषा से सांबांनधत है। इसमें कहा गया है नक एक नवधेयक को धि नवधेयक मािा जाता
है यनद इसमें निम्िनलनखत सभी या नकसी भी मामले से सांबांनधत 'के वल' प्ावधाि होते हैं:
• नकसी भी कर का अनधरोपण, उन्मल
ू ि, छूट, पररवतगि या नवनियमि;
• कें द्र सरकार द्वारा धि उधार लेिे का नवनियमि;
• भारत की सनां चत निनध या भारत की आकनस्मकता निनध की अनभरक्षा, ऐसी नकसी निनध में धि का भगु ताि या उसमें से धि
की निकासी;
• भारत की सांनचत निनध से धि का नवनियोग;
• भारत की सांनचत निनध पर प्भाररत नकसी व्यय की घोषणा या ऐसे नकसी व्यय की रानश में वृनद्ध करिा;
• भारत की सांनचत निनध या भारत के सावगजनिक खाते के नलए धि की प्ानप्त या ऐसे धि की अनभरक्षा या जारी करिा, या सांघ
या राज्य के खातों की लेखा परीक्षा; या

www.iasbaba.com rrs.iasbaba.com Page 75


IASbaba’s Rapid Revision (RaRe) Series Polity Day 1-120 2021
HINDI

• ऊपर निनदगष्ट मामलों में से नकसी के नलए आकनस्मक कोई भी मामला।


हालााँनक, नकसी नवधेयक को के वल इस कारण से धि नवधेयक िहीं मािा जाता है नक वह निम्िनलनखत के नलए प्ावधाि करता है:
• जमु ागिा या अन्य आनथगक दडां लगािा, या
• प्दाि की गई सेवाओ ां के नलए लाइसेंस या शपु क के नलए शपु क की माांग या भगु ताि; या
• स्थािीय उद्देश्यों के नलए नकसी स्थािीय प्ानधकरण या निकाय द्वारा नकसी भी कर का अनधरोपण, उन्मल
ू ि, छूट, पररवतगि या
नवनियमि।
यनद कोई प्श्न उठता है नक कोई नवधेयक धि नवधेयक है या िहीं, तो लोकसभा अध्यक्ष का निणगय अांनतम होता है। इस सांबांध में उिके
निणगय पर नकसी भी न्यायालय में या सांसद के नकसी सदि या यहाां तक नक राष्ट्रपनत द्वारा भी प्श्न िहीं उठाया जा सकता है। जब कोई
धि नवधेयक नसफाररश के नलए राज्य सभा में भेजा जाता है और राष्ट्रपनत को स्वीकृ नत के नलए प्स्तुत नकया जाता है, तो अध्यक्ष उसे
धि नवधेयक के रूप में समथगि देता है।

Q. 89) सावाजनिक नवधेयक और निजी नवधेयक के बीच अंतर के संदभा में, निम्िनिनित कथिों पर नवचार करें:
1. सांसद में सावगजनिक नवधेयक एक मांत्री द्वारा प्स्तुत नकया जाता है।
2. निजी नवधेयक को सदि में पेश करिे के नलए सात नदि का िोनटस देिा जरूरी है।
3. सदि द्वारा निजी नवधेयक को अस्वीकार करिा सरकार में सांसदीय नवश्वास की कमी की अनभव्यनक्त है और और उससे वह
इस्तीफा दे सकती है।
उपरोक्त में से कौि सा कथि सही हैं?
a) के वल 1
b) के वल 1 और 2
c) के वल 3
d) के वल 2 और 3
Q. 89) Solution (a)
ससां द के दोिों सदिों में नवधायी प्निया समाि है। प्त्येक नवधेयक को प्त्येक सदि में समाि चरणों से गजु रिा होता है। एक नवधेयक
कािूि के नलए एक प्स्ताव है और नवनधवत अनधनियनमत होिे पर यह एक अनधनियम या काििू बि जाता है।
ससां द में पेश नकए गए नवधेयक दो प्कार के होते हैं: सावगजनिक नवधेयक और निजी नवधेयक (िमशः सरकारी नवधेयक और निजी
सदस्यों के नवधेयक के रूप में भी जािे जाते हैं)। यद्यनप दोिों एक ही सामान्य प्निया द्वारा शानसत होते हैं और सदि में समाि चरणों से
गजु रते हैं, वे नवनभन्ि मामलों में नभन्ि होते हैं:
S. No. सावगजनिक नवधेयक निजी नवधेयक

www.iasbaba.com rrs.iasbaba.com Page 76


IASbaba’s Rapid Revision (RaRe) Series Polity Day 1-120 2021
HINDI

1. इसे एक मत्रां ी द्वारा ससां द में पेश नकया जाता है यह एक मत्रां ी के अलावा ससां द के नकसी भी सदस्य द्वारा पेश
नकया जाता है
2. यह सरकार (सत्तारूढ़ दल) की िीनतयों को दशागता है। यह सावगजनिक मामले पर नवपक्षी दल के रुख को दशागता है।
3. इसके सांसद द्वारा अिमु ोनदत होिे की अनधक सांभाविा है इसके सांसद द्वारा अिमु ोनदत होिे की सांभाविा कम होती है।
4. सदि द्वारा इसकी अस्वीकृ नत सरकार में सांसदीय नवश्वास सदि द्वारा इसकी अस्वीकृ नत का सरकार में सांसदीय नवश्वास
की कमी की अनभव्यनक्त के समाि है और इससे उसका या उसके इस्तीफे पर कोई प्भाव िहीं पडता है।
इस्तीफा हो सकता है
5. सदि में इसे पेश करिे के नलए सात नदिों के िोनटस की सदि में इसे पेश करिे के नलए एक महीिे का िोनटस चानहए
आवश्यकता है
6. यह कािूि नवभाग के परामशग से सांबनां धत नवभाग द्वारा तैयार इसका प्ारूप तैयार करिे की नजम्मेदारी सांबांनधत सदस्य की
नकया गया है। होती है।

Q.90) क्िोजर मोशि (closure motion) के संबंध में निम्िनिनित कथिों पर नवचार करें:
1. नगलोनटि क्लोजर के तहत, के वल महत्वपणू ग खांडों को बहस और मतदाि के नलए नलया जाता है और हस्तक्षेप करिे वाले
खांडों को छोड नदया जाता है और पाररत नकया जाता है।
2. कांगारू क्लोजर के तहत समय की कमी के कारण नकसी नवधेयक या सांकपप के नबिा चचाग वाले खांडों को भी चचाग के साथ
मतदाि के नलए रखा जाता है।
उपरोक्त में से कौि सा कथि सही हैं?
a) के वल 1
b) के वल 2
c) दोिों 1 और 2
d) ि तो 1 और ि ही 2
Q. 90) Solution (d)
पीठासीि अनधकारी की सहमनत से नकए गए प्स्ताव के अलावा आम सावगजनिक महत्व के मामले पर कोई चचाग िहीं हो सकती है। सदि
नवनभन्ि मद्दु ों पर या तो मनां त्रयों या गैर-सरकारी सदस्यों द्वारा पेश नकए गए प्स्तावों को स्वीकार या अस्वीकार करके अपिे निणगय या राय
व्यक्त करता है।
क्लोजर मोशि:
यह सदि के समक्ष नकसी मामले पर बहस को कम करिे के नलए एक सदस्य द्वारा पेश नकया गया प्स्ताव है। यनद प्स्ताव को सदि द्वारा
अिुमोनदत नकया जाता है, तो बहस को तुरांत रोक नदया जाता है और मामले पर मतदाि नकया जाता है। क्लोजर मोशि चार प्कार के
होते हैं:

www.iasbaba.com rrs.iasbaba.com Page 77


IASbaba’s Rapid Revision (RaRe) Series Polity Day 1-120 2021
HINDI

• साधारण या नसम्पल क्लोजर (Simple Closure) : यह तब होता है जब कोई सदस्य प्स्ताव करता है नक 'इस मामले पर
पयागप्त चचाग हो चक
ु ी है, अब मतदाि के नलए रखा जाए'।
• क्लोजर बॉय कम्पाटगमेंट (Closure by Compartments) : इस मामले में, बहस शरू ु होिे से पहले एक नवधेयक या एक
लांबे सांकपप के खांड को भागों में बाांटा जाता है। बहस परू े नहस्से को कवर करती है और परू े नहस्से को वोट देिे के नलए रखा
जाता है।
• कांगारू क्लोजर (Kangaroo Closure): इस प्कार के तहत, के वल महत्वपणू ग खडां बहस और मतदाि के नलए नलए जाते हैं
और हस्तक्षेप करिे वाले खडां ों को छोड नदया जाता है और पाररत के रूप में नलया जाता है।
• नगलोनटि क्लोजर (Guillotine Closure): यह तब होता है जब समय की कमी के कारण नकसी नवधेयक या सक ां पप के
नबिा चचाग वाले खडां ों को भी चचाग के नलए मतदाि के नलए रखा जाता है (चनांू क चचाग के नलए आवनां टत समय समाप्त हो गया
है)।
Q. 91) निंदा प्रस्ताव (censure motion) और अनवश्वास प्रस्ताव (no-confidence motion) के बीच अंतर के संदभा में,
निम्िनिनित कथिों पर नवचार करें:
1. निन्दा प्स्ताव को लोकसभा में स्वीकार नकए जािे के कारणों का उपलेख करिे की आवश्यकता िहीं है।
2. जब लोकसभा में अनवश्वास प्स्ताव पाररत हो जाता है, तो मांनत्रपररषद को अपिे पद से इस्तीफा दे देिा चानहए।
3. अनवश्वास प्स्ताव नकसी एक मांत्री या मनां त्रयों के समहू या परू ी मनां त्रपररषद के नखलाफ पेश नकया जा सकता है।
उपरोक्त में से कौि सा कथि सही हैं?
a) के वल 1 और 2
b) के वल 2
c) के वल 3
d) के वल 2 और 3
Q. 91) Solution (b)
निदां ा प्स्ताव और अनवश्वास प्स्ताव के बीच अतां र:

ि. निांदा प्स्ताव अनवश्वास प्स्ताव


सां.

1. इसे लोकसभा में इसे अपिािे के कारणों का उपलेख करिा चानहए। इसे लोकसभा में अपिािे के कारणों को बतािे की
आवश्यकता िहीं है।

www.iasbaba.com rrs.iasbaba.com Page 78


IASbaba’s Rapid Revision (RaRe) Series Polity Day 1-120 2021
HINDI

2. इसे एक व्यनक्तगत मत्रां ी या मांनत्रयों के समहू या परू ी मांनत्रपररषद के इसे के वल मांनत्रपररषद के नवरुद्ध ही पेश नकया जा सकता
नखलाफ पेश नकया जा सकता है। है।

3. यह नवनशष्ट िीनतयों और कायों के नलए मांनत्रपररषद की निदां ा करिे यह मनां त्रपररषद में लोकसभा के नवश्वास को सनु िनित
के नलए पेश नकया गया है। करिे के नलए पेश नकया जाता है।

4. यनद यह लोकसभा में पाररत हो जाता है, तो मांनत्रपररषद को यनद यह लोकसभा में पाररत हो जाता है, तो मांनत्रपररषद
कायागलय से इस्तीफा देिे की आवश्यकता िहीं होती है। को अपिे पद से इस्तीफा दे देिा चानहए।

Q. 92) िोक िेिा सनमनत (Public Accounts Committee) के संदभा में, निम्िनिनित कथिों पर नवचार कीनजएः
1. इस सनमनत का गठि भारतीय पररषद अनधनियम 1909 के प्ावधािों के तहत नकया गया था।
2. सदस्यों का चिु ाव लोकसभा द्वारा प्नतवषग अपिे सदस्यों में से आिुपानतक प्नतनिनधत्व के नसद्धाांत के अिुसार एकल
सांिमणीय मत द्वारा नकया जाता है।
3. सनमनत का कायग भारत के नियत्रां क और महालेखा परीक्षक की वानषगक लेखा परीक्षा ररपोटग की जाांच करिा है।
उपरोक्त में से कौि सा कथि सही हैं?
a) के वल 1 और 2
b) के वल 2
c) के वल 3
d) के वल 2 और 3
Q. 92) Solution (c)
लोक लेखा सनमनत:
• यह सनमनत पहली बार 1921 में भारत सरकार अनधनियम 1919 के प्ावधािों के तहत स्थानपत की गई थी और तब से यह
अनस्तत्व में है।
• वतगमाि में, इसमें 22 सदस्य (लोकसभा से 15 और राज्य सभा से 7) शानमल हैं। सदस्यों का चिु ाव ससां द द्वारा प्नतवषग अपिे
सदस्यों में से आिपु ानतक प्नतनिनधत्व के नसद्धातां के अिसु ार एकल सि
ां मणीय मत द्वारा नकया जाता है। इस प्कार, सभी दलों
को इसमें उनचत प्नतनिनधत्व नमलता है। सदस्यों का कायगकाल एक वषग का होता है।
• एक मांत्री को सनमनत के सदस्य के रूप में िहीं चिु ा जा सकता है।

www.iasbaba.com rrs.iasbaba.com Page 79


IASbaba’s Rapid Revision (RaRe) Series Polity Day 1-120 2021
HINDI

• सनमनत के अध्यक्ष की नियनु क्त अध्यक्ष द्वारा अपिे सदस्यों में से की जाती है।
• 1966-67 तक, सनमनत के अध्यक्ष सत्ताधारी दल के थे। हालााँनक, 1967 के बाद से एक परांपरा नवकनसत हुई है नजसके तहत
सनमनत के अध्यक्ष को हमेशा नवपक्ष से चिु ा जाता है।
• सनमनत का कायग भारत के नियत्रां क और महालेखा परीक्षक (CAG) की वानषगक ऑनडट ररपोटग की जाचां करिा है, नजसे
राष्ट्रपनत द्वारा ससां द के समक्ष रखा जाता है।
• सीएजी (CAG) राष्ट्रपनत को तीि ऑनडट ररपोटग प्स्तुत करता है, अथागत् नवनियोग खातों पर ऑनडट ररपोटग, नवत्त खातों पर
ऑनडट ररपोटग और सावगजनिक उपिमों पर ऑनडट ररपोटग।
• सनमनत तकिीकी अनियनमतताओ ां का पता लगािे के नलए ि के वल काििू ी और औपचाररक दृनष्टकोण से सावगजनिक व्यय
की जाचां करती है, बनपक बबागदी, हानि, भ्रष्टाचार, अपव्यय, अक्षमता और निरथगक व्यय के मामलों को सामिे लािे के नलए
अथगव्यवस्था, नववेक, बनु द्धमत्ता और औनचत्य के दृनष्टकोण से भी जाांच करती है।

Q. 93) प्राक्किि सनमनत (Estimates Committee) के संबध


ं में निम्िनिनित में से कौि सा/से कथि सही है/हैं?
1. इस सनमनत में लोकसभा का कोई प्नतनिनधत्व िहीं है।
2. एक मांत्री को इस सनमनत के सदस्य के रूप में िहीं चिु ा जा सकता है।
3. सनमनत का अध्यक्ष निरपवाद रूप से नवपक्ष से होता है।
िीचे नदए गए कूटों में से सही उत्तर चुनिए:
a) के वल 1 और 2
b) के वल 2
c) के वल 3
d) 1, 2 और 3
Q. 93) Solution (b)
प्ाक्कलि सनमनत:
• इस सनमनत का उद्भव 1921 में स्थानपत स्थायी नवत्तीय सनमनत के रूप में देखा जा सकता है। स्वतत्रां ता के बाद के यगु में पहली
प्ाक्कलि सनमनत का गठि 1950 में तत्कालीि नवत्त मत्रां ी जॉि मथाई की नसफाररश पर नकया गया था।
• मल
ू रूप से, इसमें 25 सदस्य थे लेनकि 1956 में इसकी सदस्यता बढ़ाकर 30 कर दी गई थी। सभी तीस सदस्य के वल
लोकसभा से होते हैं। इस सनमनत में राज्यसभा का कोई प्नतनिनधत्व िहीं है।

www.iasbaba.com rrs.iasbaba.com Page 80


IASbaba’s Rapid Revision (RaRe) Series Polity Day 1-120 2021
HINDI

• इि सदस्यों को लोकसभा द्वारा प्त्येक वषग अपिे ही सदस्यों में से आिुपानतक प्नतनिनधत्व के नसद्धातां ों के अिुसार एकल
सांिमणीय मत के माध्यम से चिु ा जाता है। इस प्कार, सभी दलों को इसमें उनचत प्नतनिनधत्व नमलता है। कायागलय की
अवनध एक वषग है।
• एक मत्रां ी को सनमनत के सदस्य के रूप में िहीं चिु ा जा सकता है। सनमनत के अध्यक्ष की नियनु क्त अध्यक्ष द्वारा अपिे सदस्यों
में से की जाती है और वह निरपवाद रूप से सत्ताधारी दल से होता है।

Q. 94) सावाजनिक उपक्रम सनमनत (Committee on Public Undertakings) के संदभा में, निम्िनिनित कथिों पर नवचार
कीनजएः
1. सनमनत में 15 सदस्य होते हैं नजिमें 10 लोकसभा से और 5 राज्यसभा से होते हैं।
2. सनमनत सावगजनिक उपिमों के व्यवसाय या वानणनज्यक कायों से अलग प्मख
ु सरकारी िीनत के मामलों की जाचां करती है।
3. सनमनत के अध्यक्ष की नियनु क्त अध्यक्ष द्वारा अपिे सदस्यों में से की जाती है जो के वल लोकसभा से चिु े जाते हैं।
उपरोक्त में से कौि सा कथि सही हैं?
a) के वल 1
b) के वल 1 और 2
c) के वल 3
d) के वल 2 और 3
Q. 94) Solution (c)
सावगजनिक उपिम सनमनत:
• यह सनमनत 1964 में कृ ष्ट्णा मेिि सनमनत की नसफाररश पर बिाई गई थी।
• मल ू रूप से, इसमें 15 सदस्य थे (लोकसभा से 10 और राज्य सभा से 5)। हालााँनक, 1974 में, इसकी सदस्यता 22 (लोकसभा
से 15 और राज्य सभा से 7) तक बढ़ा दी गई थी।
• इस सनमनत के सदस्यों का चिु ाव ससां द द्वारा प्नतवषग अपिे सदस्यों में से आिुपानतक प्नतनिनधत्व के नसद्धातां के अिुसार एकल
सांिमणीय मत द्वारा नकया जाता है। इस प्कार, सभी दलों को इसमें उनचत प्नतनिनधत्व नमलता है। सदस्यों का कायगकाल एक
वषग का होता है। एक मांत्री को सनमनत के सदस्य के रूप में िहीं चिु ा जा सकता है।
• सनमनत के अध्यक्ष की नियनु क्त अध्यक्ष द्वारा अपिे सदस्यों में से की जाती है जो के वल लोकसभा से चिु े जाते हैं।
• इस प्कार, सनमनत के सदस्य जो राज्यसभा से हैं, उन्हें अध्यक्ष के रूप में नियक्त
ु िहीं नकया जा सकता है।

www.iasbaba.com rrs.iasbaba.com Page 81


IASbaba’s Rapid Revision (RaRe) Series Polity Day 1-120 2021
HINDI

सनमनत के कायग हैं:


• सावगजनिक उपिमों की ररपोटग और खातों की जाांच करिे के नलए
• सावगजनिक उपिमों पर नियत्रां क-महालेखापरीक्षक की ररपोटों की जाचां करिा।
• (सावगजनिक उपिमों की स्वायत्तता और दक्षता के सांदभग में) जाचां करिा नक क्या सावगजनिक उपिमों के मामलों का प्बांधि
ठोस व्यावसानयक नसद्धातां ों और नववेकपणू ग वानणनज्यक प्थाओ ां के अिुसार नकया जा रहा है।
• सावगजनिक उपिमों के सबां धां में लोक लेखा सनमनत और प्ाक्कलि सनमनत में निनहत ऐसे अन्य कायों का प्योग करिा जो
अध्यक्ष द्वारा समय-समय पर उसे आवनां टत नकए जाते हैं।
सनमनत को निम्िनलनखत में से नकसी की परीक्षण और जाचां िहीं करिी है:
• सावगजनिक उपिमों के व्यवसाय या वानणनज्यक कायों से अलग प्मख
ु सरकारी िीनत के मामले।
• नदि-प्नतनदि के प्शासि के मामले।
• नजि मामलों पर नवचार करिे के नलए मशीिरी नकसी नवशेष कािूि द्वारा स्थानपत की जाती है नजसके तहत एक नवशेष
सावगजनिक उपिम स्थानपत नकया जाता है।
Q. 95) निम्िनिनित में से नकस सनमनत का गठि सरकारी मंत्राियों और नवभागों के िचा और धि के उपयोग के मामिे में
कामकाज की जांच करिे के उद्देश्य से सस
ं द द्वारा नकया जाता है?
a) लोक लेखा सनमनत
b) सरकारी आश्वासिों पर सनमनत
c) सावगजनिक उपिमों पर सनमनत
d) प्ाक्कलि सनमनत
Q. 95) Solution (d)
प्ाक्कलि सनमनत ससां द के चयनित सदस्यों की एक सनमनत है, नजसका गठि ससां द द्वारा सरकारी मत्रां ालयों और नवभागों के खचग और
धि के उपयोग की जाचां के उद्देश्य से नकया जाता है।
सनमनत का कायग बजट में शानमल अिुमािों की जाचां करिा और सावगजनिक व्यय में 'अथगव्यवस्था' का सझु ाव देिा है। इसनलए, इसे
'सतत अथगव्यवस्था सनमनत' के रूप में वनणगत नकया गया है।
अनधक नवस्तार से, सनमनत के कायग हैं:
• यह ररपोटग करिे के नलए नक अिुमािों में अांतनिगनहत िीनत के अिुरूप अथगव्यवस्था, सांगठि में सधु ार, दक्षता और प्शासनिक
सधु ार क्या प्भानवत हो सकते हैं

www.iasbaba.com rrs.iasbaba.com Page 82


IASbaba’s Rapid Revision (RaRe) Series Polity Day 1-120 2021
HINDI

• प्शासि में दक्षता और नमतव्यनयता लािे के नलए वैकनपपक िीनतयों का सझु ाव देिा
• यह जाचां िे के नलए नक क्या अिुमािों में निनहत िीनत की सीमाओ ां के भीतर पैसा अच्छी तरह से रखा गया है
• उस रूप का सझु ाव देिा नजसमें अिुमािों को सांसद में प्स्तुत नकया जािा है
सनमनत ऐसे सावगजनिक उपिमों के सांबांध में अपिे कायों का प्योग िहीं करेगी जो सावगजनिक उपिम सनमनत को आवांनटत नकए गए हैं।
सनमनत परू े नवत्तीय वषग में समय-समय पर प्ाक्कलिों की जाचां जारी रख सकती है और जैसे-जैसे इसकी जाांच होती है, सदि को ररपोटग
कर सकती है। नकसी एक वषग के समस्त प्ाक्कलिों की जाचां करिा सनमनत का दानयत्व िहीं होगा। इस त्य के बावजदू नक सनमनत िे
कोई ररपोटग िहीं दी है, अिुदाि की माांगों पर अांततः मतदाि नकया जा सकता है।
Q. 96) नवभागीय स्थायी सनमनतयों (departmental standing committees) के सबं ध
ं में, निम्िनिनित कथिों पर नवचार
करें:
1. स्थायी सनमनतयों का मख्ु य उद्देश्य सांसद के नलए मांनत्रपररषद की नवत्तीय जवाबदेही सनु िनित करिा है।
2. प्त्येक स्थायी सनमनत में 22 सदस्य होते हैं; जो लोकसभा से 15 और राज्यसभा से 7 होते हैं
उपरोक्त में से कौि सा कथि सही हैं?
a) के वल 1
b) के वल 2
c) दोिों 1 और 2
d) ि तो 1 और ि ही 2
Q. 96) Solution (a)
नवभागीय स्थायी सनमनतया:ां
• लोकसभा की नियम सनमनत की नसफाररश पर, 1993 में सांसद में 17 नवभागीय-सांबांनधत स्थायी सनमनतयों (डीआरएससी) का
गठि नकया गया था। 2004 में, ऐसी सात और सनमनतयााँ स्थानपत की गई,ां इस प्कार उिकी सांख्या 17 से बढ़कर 24 हो गई।
• स्थायी सनमनतयों का मख्ु य उद्देश्य कायगपानलका (अथागत् मनां त्रपररषद) की ससां द के प्नत अनधक जवाबदेही, नवशेष रूप से
नवत्तीय जवाबदेही सनु िनित करिा है। वे बजट पर अनधक प्भावी ढगां से बहस करिे में ससां द की सहायता भी करते हैं।
• 24 स्थायी सनमनतयााँ अपिे अनधकार क्षेत्र में कें द्र सरकार के सभी मत्रां ालयों/नवभागों को कवर करती हैं।
• प्त्येक स्थायी सनमनत में 31 सदस्य होते हैं (21 लोकसभा से और 10 राज्य सभा से)। लोकसभा के सदस्यों को अध्यक्ष द्वारा
अपिे सदस्यों में से िानमत नकया जाता है, जैसे राज्यसभा के सदस्यों को सभापनत द्वारा अपिे सदस्यों में से िानमत नकया जाता
है।

www.iasbaba.com rrs.iasbaba.com Page 83


IASbaba’s Rapid Revision (RaRe) Series Polity Day 1-120 2021
HINDI

• एक मत्रां ी नकसी भी स्थायी सनमनत के सदस्य के रूप में िानमत होिे के योग्य िहीं है। यनद कोई सदस्य नकसी स्थायी सनमनत में
िामाांकि के बाद मांत्री नियक्त
ु हो जाता है, तो वह सनमनत का सदस्य िहीं रह जाता है।
• प्त्येक स्थायी सनमनत का कायगकाल उसके गठि की तारीख से एक वषग का होता है।
• 24 स्थायी सनमनतयों में से 8 राज्य सभा के अधीि और 16 लोक सभा के अधीि कायग करती हैं।

Q. 97) निम्िनिनित में से कौि सी नवभागीय स्थायी सनमनतयाँ राज्य सभा के अनधकार क्षेत्र में आती हैं ?
1. गृह मामलों की सनमनत
2. नवज्ञाि और प्ौद्योनगकी, पयागवरण और वि सबां धां ी सनमनत
3. कृ नष सांबांधी सनमनत
िीचे नदए गए कूटों में से सही उत्तर चुनिए:
a) के वल 1 और 2
b) के वल 2 और 3
c) के वल 1 और 3
d) 1, 2 और 3
Q. 97) Solution (a)
दोिों सदिों की चौबीस नवभाग-सांबनां धत सांसदीय स्थायी सनमनतयों में से, निम्िनलनखत आठ सनमनतयााँ राज्य सभा के सभापनत के निदेशि
और नियांत्रण में कायग करती हैं:
1. वानणज्य पर सनमनत
2. गृह मामलों की सनमनत
3. मािव ससां ाधि नवकास सनमनत
4. उद्योग पर सनमनत
5. नवज्ञाि और प्ौद्योनगकी, पयागवरण और वि सबां धां ी सनमनत
6. पररवहि, पयगटि और सांस्कृ नत पर सनमनत
7. स्वास््य और पररवार कपयाण सनमनत
8. कानमगक, लोक नशकायत, काििू और न्याय सांबांधी सनमनत
www.iasbaba.com rrs.iasbaba.com Page 84
IASbaba’s Rapid Revision (RaRe) Series Polity Day 1-120 2021
HINDI

अन्य सोलह सनमनतयााँ लोकसभा अध्यक्ष के निदेशि और नियत्रां ण में कायग करती हैं।
Q. 98) निम्िनिनित में से कौि सी सनमनत िोकसभा के कायाक्रम और समय साररणी को नियंनत्रत करती है तथा सरकार
द्वारा सदि के समक्ष िाए गए नवधायी और अन्य कायों के िेि-देि के निए समय आवनं टत करती है।?
a) सामान्य प्योजि सनमनत (General Purposes Committee)
b) सदि सनमनत
c) नियम सनमनत
d) कायग सलाहकार सनमनत (Business Advisory Committee)
Q. 98) Solution (d)
कायग सलाहकार सनमनत: यह सनमनत सदि के कायगिम और समय साररणी को नियांनत्रत करती है। यह सरकार द्वारा सदि के समक्ष लाए
गए नवधायी और अन्य कायों के लेि-देि के नलए समय आवांनटत करता है। लोकसभा सनमनत में अध्यक्ष के रूप में लोकसभा अध्यक्ष
सनहत 15 सदस्य होते हैं। राज्यसभा में इसके पदेि अध्यक्ष के रूप में सभापनत सनहत 11 सदस्य होते हैं।

Q. 99) संसदीय मंचों (Parliamentary forums) के संदभा में, निम्िनिनित कथिों पर नवचार कीनजएः
1. लोकसभा का अध्यक्ष सभी सांसदीय मचां ों का पदेि अध्यक्ष होता है।
2. ससां दीय मांचों के गठि के पीछे का उद्देश्य सदस्यों को महत्वपणू ग मद्दु ों पर कें नद्रत और साथगक चचाग करिे की दृनष्ट से सांबांनधत
मांनत्रयों के साथ बातचीत करिे के नलए एक मांच प्दाि करिा है।
उपरोक्त में से कौि सा कथि सही हैं?
a) के वल 1
b) के वल 2
c) दोिों 1 और 2
d) ि तो 1 और ि ही 2
Q.99) Solution (b)
सांसदीय मचां ों के गठि के पीछे उद्देश्य हैं:
i.कायागन्वयि प्निया को तेज करिे के नलए पररणामोन्मख ु ी दृनष्टकोण के साथ महत्वपणू ग मद्दु ों पर कें नद्रत और साथगक चचाग करिे की दृनष्ट से
सदस्यों को सांबांनधत मांनत्रयों, नवशेषज्ञों और िोडल मांत्रालयों के प्मखु अनधकाररयों के साथ बातचीत करिे के नलए एक मांच प्दाि करिा;

www.iasbaba.com rrs.iasbaba.com Page 85


IASbaba’s Rapid Revision (RaRe) Series Polity Day 1-120 2021
HINDI

ii. सदस्यों को नचतां ा के प्मख


ु क्षेत्रों और जमीिी स्तर की नस्थनत के बारे में सांवेदिशील बिािा और उन्हें देश और नवदेश दोिों के नवशेषज्ञों
से िवीितम जािकारी, ज्ञाि, तकिीकी जािकारी और मपू यवाि इिपटु से लैस करिा तानक वे इि मद्दु ों को प्भावी ढांग से उठा सकें । सदि
के पटल और नवभागीय-सांबांनधत स्थायी सनमनतयों (डीआरएससी) की बैठकों में; तथा
iii. सांबांनधत मत्रां ालयों, नवश्वसिीय गैर सरकारी सांगठिों, समाचार पत्रों, सांयक्त
ु राष्ट्र, इटां रिेट आनद से महत्वपणू ग मद्दु ों पर डेटा के सांग्रह के
माध्यम से एक डेटा-बेस तैयार करिा और सदस्यों को इसका प्सार करिा तानक वे मचां ों की चचाग में साथगक रूप से भाग ले सकें और
स्पष्टीकरण माांग सकें । बैठकों में उपनस्थत मत्रां ालय के नवशेषज्ञों या अनधकाररयों से।

लोकसभा का अध्यक्ष जिसांख्या और सावगजनिक स्वास््य पर सांसदीय मचां को छोडकर सभी मचां ों का पदेि अध्यक्ष होता है, नजसमें
राज्यसभा का सभापनत पदेि अध्यक्ष होता है और अध्यक्ष पदेि सह-अध्यक्ष होता है। राज्य सभा के उपसभापनत, लोकसभा के उपाध्यक्ष,
सांबांनधत मांत्री और नवभाग से सांबांनधत स्थायी सनमनतयों के अध्यक्ष सबां ांनधत मांचों के पदेि उपाध्यक्ष होते हैं।
प्त्येक फोरम में 31 से अनधक सदस्य िहीं होते (राष्ट्रपनत, उप-राष्ट्रपनत को छोडकर) नजिमें से 21 से अनधक लोकसभा से िहीं होते हैं
और 10 से अनधक राज्यसभा से िहीं होते हैं।

Q.100) निम्िनिनित कथिों पर नवचार करें:


1. अधीिस्थ नवधाि सांबांधी सनमनत यह देखिे के नलए सभा पटल पर रखे गए सभी पत्रों की जाचां करती है नक क्या वे सांनवधाि
के उपबांधों अथवा सांबांनधत अनधनियम या नियम का पालि करते हैं।
2. नवशेषानधकार सनमनत कदाचार के मामलों की जाचां करके और उनचत कारग वाई की नसफाररश करके सांसद सदस्यों की आचार
सांनहता लागू करती है।
उपरोक्त में से कौि सा कथि सही हैं?
a) के वल 1
b) के वल 2
c) दोिों 1 और 2
d) ि तो 1 और ि ही 2
Q. 100) Solution (d)
अधीिस्थ नवधाि सांबांधी सनमनत: यह सनमनत जाांच करती है और सदि को ररपोटग करती है नक क्या सांसद द्वारा प्त्यायोनजत या सांनवधाि
द्वारा कायगपानलका को प्दत्त नवनियमों, नियमों और उपनियमों को बिािे की शनक्तयों का इसके द्वारा उनचत रूप से प्योग नकया जा रहा
है। दोिों सदिों में सनमनत में 15 सदस्य होते हैं। इसका गठि 1953 में नकया गया था।

www.iasbaba.com rrs.iasbaba.com Page 86


IASbaba’s Rapid Revision (RaRe) Series Polity Day 1-120 2021
HINDI

सभा पटल पर रखे गए पत्रों सांबांधी सनमनत (Committee on Papers Laid on the Table) : इस सनमनत का गठि 1975 में नकया
गया था। लोक सभा सनमनत में 15 सदस्य होते हैं, जबनक राज्य सभा सनमनत में 10 सदस्य होते हैं। यह मांनत्रयों द्वारा सदि के पटल पर
रखे गए सभी कागजातों की जाांच करता है नक वे सांनवधाि के प्ावधािों, या सांबांनधत अनधनियम या नियम का पालि करते हैं या िहीं।
यह वैधानिक अनधसचू िाओ ां और आदेशों की जाच ां िहीं करता है जो अधीिस्थ नवधाि सबां धां ी सनमनत के अनधकार क्षेत्र में आते हैं।
नवशेषानधकार सनमनत (Committee of Privileges) : इस सनमनत के कायग प्कृ नत में अधग-न्यानयक हैं। यह सदि और उसके सदस्यों
के नवशेषानधकारों के उपलांघि के मामलों की जाांच करता है और उनचत कारग वाई की नसफाररश करता है। लोकसभा सनमनत में 15 सदस्य
होते हैं, जबनक राज्यसभा सनमनत में 10 सदस्य होते हैं।
आचार सनमनत (Ethics Committee) : इस सनमनत का गठि 1997 में राज्यसभा में और 2000 में लोकसभा में नकया गया था। यह
ससां द सदस्यों की आचार सनां हता को लागू करती है। यह कदाचार के मामलों की जाचां करता है और उनचत कारग वाई की नसफाररश करता
है। इस प्कार, यह ससां द में अिश
ु ासि और मयागदा बिाए रखिे से सबां नां धत है।
Q. 101) निम्िनिनित कथिों पर नवचार करें:
1. कें द्रीय कािूिों के साथ-साथ राज्य कािूिों दोिों के नलए न्यायालयों की एकल प्णाली को भारत सरकार अनधनियम 1919 से
अपिाया गया था।
2. भारतीय सांनवधाि का भाग VI सवोच्च न्यायालय के सांगठि, स्वतांत्रता, अनधकार क्षेत्र और शनक्तयों से सांबांनधत है।
उपरोक्त में से कौि सा कथि सही हैं?
a) के वल 1
b) के वल 2
c) दोिों 1 और 2
d) ि तो 1 और ि ही 2
Q. 101) Solution (d)
भारतीय सांनवधाि िे एक एकीकृ त न्यानयक प्णाली की स्थापिा की है नजसके शीषग पर सवोच्च न्यायालय और उसके िीचे उच्च न्यायालय
हैं।
एक उच्च न्यायालय (और राज्य स्तर से िीचे) के तहत, अधीिस्थ न्यायालयों का एक पदािि
ु म होता है, अथागत नजला अदालतें और
अन्य निचली अदालतें।
1935 के भारत सरकार अनधनियम से अपिाई गई अदालतों की यह एकल प्णाली, कें द्रीय कािूिों के साथ-साथ राज्य के काििू ों को
भी लागू करती है।
भारत के सवोच्च न्यायालय का उद्घाटि 28 जिवरी 1950 को हुआ था। यह भारत सरकार अनधनियम 1935 के तहत स्थानपत भारत
के सांघीय न्यायालय का स्थाि लेता है। हालााँनक, सवोच्च न्यायालय का अनधकार क्षेत्र उसके पवू गवती न्यायालय की तल
ु िा में अनधक
है। यह इसनलए है क्योंनक; सप्ु ीम कोटग िे अपील की सवोच्च अदालत के रूप में निनटश नप्वी काउांनसल की जगह ले ली है।

www.iasbaba.com rrs.iasbaba.com Page 87


IASbaba’s Rapid Revision (RaRe) Series Polity Day 1-120 2021
HINDI

सनां वधाि के भाग V में अिुच्छे द 124 से 147 सवोच्च न्यायालय के सांगठि, स्वतत्रां ता, अनधकार क्षेत्र, शनक्तयों, प्नियाओ ां आनद से
सांबांनधत है। सांसद भी उन्हें नवनियनमत करिे के नलए अनधकृ त है।

Q. 102) निम्िनिनित कथिों पर नवचार करें:


1. सवोच्च न्यायालय के न्यायाधीश के रूप में नियक्त
ु होिे वाले व्यनक्त को दस वषग तक उच्च न्यायालय का न्यायाधीश होिा
चानहए।
2. उच्चतम न्यायालय के न्यायाधीशों के वेति, भत्ते और पेंशि भारत की सनां चत निनध पर भाररत होते हैं।
उपरोक्त में से कौि सा कथि सही हैं?
a) के वल 1
b) के वल 2
c) दोिों 1 और 2
d) ि तो 1 और ि ही 2
Q. 102) Solution (b)
सवोच्च न्यायालय के न्यायाधीशों की योग्यताएाँ:
सवोच्च न्यायालय के न्यायाधीश के रूप में नियक्त
ु होिे वाले व्यनक्त में निम्िनलनखत योग्यताएां होिी चानहए:
1. वह भारत का िागररक होिा चानहए।
2. (ए) वह पाचां साल के नलए एक उच्च न्यायालय (या उत्तरानधकार में उच्च न्यायालयों) का न्यायाधीश होिा चानहए था; या (बी) वह
दस साल के नलए एक उच्च न्यायालय (या उत्तरानधकार में उच्च न्यायालयों) का वकील होिा चानहए था; या (सी) वह राष्ट्रपनत की राय
में एक प्नतनष्ठत नवनधवेत्ता होिा चानहए।
उपरोक्त से, यह स्पष्ट है नक सांनवधाि िे सवोच्च न्यायालय के न्यायाधीश के रूप में नियनु क्त के नलए न्यिू तम आयु निधागररत िहीं की है।
उच्चतम न्यायालय के न्यायाधीशों के वेति, भत्ते और पेंशि भारत की सनां चत निनध पर भाररत होते हैं। नवत्तीय आपात नस्थनत के दौराि
उिकी नियनु क्त के बाद उिके नलए अलाभकारी पररवतगि िहीं नकया जा सकता।

Q. 103) निम्िनिनित कथिों पर नवचार करें:


1. सवोच्च न्यायालय का एक न्यायाधीश पाचां साल के नलए या 65 वषग की आयु प्ाप्त करिे तक, जो भी पहले हो, तक पद धारण
करता है।
2. सवोच्च न्यायालय का एक न्यायाधीश भारत के मख्ु य न्यायाधीश को पत्र नलखकर अपिे पद से त्यागपत्र दे सकता है।

www.iasbaba.com rrs.iasbaba.com Page 88


IASbaba’s Rapid Revision (RaRe) Series Polity Day 1-120 2021
HINDI

उपरोक्त में से कौि सा कथि सही हैं?


a) के वल 1
b) के वल 2
c) दोिों 1 और 2
d) ि तो 1 और ि ही 2
Q. 103) Solution (d)
न्यायाधीशों का कायगकाल:
सनां वधाि िे सवोच्च न्यायालय के न्यायाधीश का कायगकाल निनित िहीं नकया है। हालााँनक, यह इस सांबांध में निम्िनलनखत तीि प्ावधाि
करता है:
1. वह 65 वषग की आयु प्ाप्त करिे तक पद पर बिे रहते हैं। उसकी उम्र के सबां धां में कोई भी प्श्न ऐसे प्ानधकारी द्वारा और ससां द
द्वारा प्दाि नकए गए तरीके से निधागररत नकया जािा है।
2. वह राष्ट्रपनत को पत्र नलखकर अपिे पद से त्यागपत्र दे सकता है।
3. उन्हें सांसद की नसफाररश पर राष्ट्रपनत द्वारा उिके पद से हटाया जा सकता है।

Q. 104) उच्चतम न्यायािय के न्यायाधीशों को हटािे के संदभा में निम्िनिनित कथिों पर नवचार कीनजएः
1. न्यायाधीश जाचां अनधनियम 1968 उच्चतम न्यायालय के न्यायाधीशों को हटािे की प्निया निधागररत करता है।
2. राष्ट्रपनत को सांबोनधत एक न्यायाधीश के महानभयोग के प्स्ताव पर लोकसभा या राज्यसभा के कम से कम 50 सदस्यों द्वारा
हस्ताक्षर नकए जािे चानहए।
3. न्यायाधीश को हटािे का प्स्ताव ससां द के प्त्येक सदि द्वारा नवशेष बहुमत से पाररत नकया जािा चानहए।
उपरोक्त में से कौि सा कथि सही हैं?
a) के वल 1 और 2
b) के वल 2 और 3
c) के वल 1 और 3
d) 1, 2 और 3
Q. 104) Solution (c)
न्यायाधीशों को हटािा:

www.iasbaba.com rrs.iasbaba.com Page 89


IASbaba’s Rapid Revision (RaRe) Series Polity Day 1-120 2021
HINDI

सवोच्च न्यायालय के नकसी न्यायाधीश को राष्ट्रपनत के आदेश से उसके पद से हटाया जा सकता है। राष्ट्रपनत हटािे का आदेश तभी
जारी कर सकता है जब सांसद द्वारा उसे उसी सत्र में इस तरह के निष्ट्कासि के नलए एक अनभभाषण प्स्ततु नकया गया हो।
अनभभाषण को सांसद के प्त्येक सदि के नवशेष बहुमत (अथागत उस सदि की कुल सदस्यता का बहुमत और उस सदि के उपनस्थत
और मतदाि करिे वाले सदस्यों के कम से कम दो-नतहाई बहुमत) द्वारा समनथगत होिा चानहए। निष्ट्कासि का आधार दो है- नसद्ध कदाचार
या अक्षमता हैं।
जज इक्ां वायरी एक्ट (1968) महानभयोग की प्निया द्वारा सवोच्च न्यायालय के एक न्यायाधीश को हटािे से सबां नां धत प्निया को
नियनां त्रत करता है:
1. 100 सदस्यों (लोकसभा के मामले में) या 50 सदस्यों (राज्य सभा के मामले में) द्वारा हस्ताक्षररत हटािे का प्स्ताव
अध्यक्ष/सभापनत को नदया जािा है।
2. अध्यक्ष/सभापनत प्स्ताव को स्वीकार कर सकते हैं या इसे स्वीकार करिे से इिकार कर सकते हैं।
3. यनद इसे स्वीकार कर नलया जाता है, तो अध्यक्ष/सभापनत को आरोपों की जाचां के नलए तीि सदस्यीय सनमनत का गठि करिा
होता है।
4. सनमनत में शानमल होिा चानहए:
• मख् ु य न्यायाधीश या सवोच्च न्यायालय के न्यायाधीश,
• एक उच्च न्यायालय के मख्ु य न्यायाधीश, और
• एक प्नतनष्ठत न्यायनवद।्
5. यनद सनमनत न्यायाधीश को दव्ु यगवहार या अक्षमता से पीनडत होिे का दोषी पाती है, तो सदि प्स्ताव पर नवचार कर सकता है।
6. सांसद के प्त्येक सदि द्वारा नवशेष बहुमत से प्स्ताव पाररत होिे के बाद, न्यायाधीश को हटािे के नलए राष्ट्रपनत को एक
अनभभाषण प्स्ततु नकया जाता है।
7. अतां में, राष्ट्रपनत न्यायाधीश को हटािे का आदेश पाररत करता है।
सप्ु ीम कोटग के नकसी न्यायाधीश पर अब तक महानभयोग िहीं लगाया गया है, महानभयोग का पहला मामला सप्ु ीम कोटग के जनस्टस वी.
रामास्वामी (1991-1993) का है। हालानां क जाच
ां सनमनत िे उन्हें दव्ु यगवहार का दोषी पाया, लेनकि उन्हें हटाया िहीं जा सका क्योंनक
महानभयोग प्स्ताव लोकसभा में नवफल रहा था। काग्रां ेस पाटी मतदाि से अलग रही।

Q. 105) निम्िनिनित कथिों पर नवचार करें:


1. राष्ट्रपनत एक उच्च न्यायालय के न्यायाधीश को एक अस्थायी अवनध के नलए सवोच्च न्यायालय के तदथग न्यायाधीश के रूप
में नियक्त
ु करता है जब स्थायी न्यायाधीशों की कोरम की कमी होती है।
2. एक अस्थायी अवनध के नलए सवोच्च न्यायालय के न्यायाधीश के रूप में कायग करिे के नलए एक उच्च न्यायालय के
सेवानिवृत्त न्यायाधीश को नियक्त
ु करिे के नलए भारत के मख्ु य न्यायाधीश द्वारा राष्ट्रपनत की सहमनत आवश्यक है।
3. भारत के मख्ु य न्यायाधीश का पद ररक्त होिे पर राष्ट्रपनत सवोच्च न्यायालय के न्यायाधीश को भारत के कायगवाहक मख्ु य
न्यायाधीश के रूप में नियक्त
ु कर सकता है।
उपरोक्त में से कौि सा कथि सही हैं?

www.iasbaba.com rrs.iasbaba.com Page 90


IASbaba’s Rapid Revision (RaRe) Series Polity Day 1-120 2021
HINDI

a) के वल 1
b) के वल 1 और 2
c) के वल 3
d) के वल 2 और 3
Q. 105) Solution (d)
तदथग न्यायाधीश:
• जब सवोच्च न्यायालय के नकसी भी सत्र को आयोनजत करिे या जारी रखिे के नलए स्थायी न्यायाधीशों की गणपनू तग की कमी
होती है, तो भारत के मख्ु य न्यायाधीश एक उच्च न्यायालय के न्यायाधीश को अस्थायी अवनध के नलए सवोच्च न्यायालय के
तदथग न्यायाधीश के रूप में नियक्तु कर सकते हैं।
• वह ऐसा के वल सांबांनधत उच्च न्यायालय के मख्ु य न्यायाधीश के परामशग और राष्ट्रपनत की पवू ग सहमनत से ही कर सकता है।
• इस प्कार नियक्त
ु न्यायाधीश को सवोच्च न्यायालय के न्यायाधीश के रूप में नियनु क्त के नलए योग्य होिा चानहए। इस प्कार
नियक्त
ु न्यायाधीश का यह कतगव्य है नक वह अपिे कायागलय के अन्य कतगव्यों को प्ाथनमकता देते हुए सवोच्च न्यायालय की
बैठकों में भाग लें।
• सवोच्च न्यायालय की बैठकों में भाग लेिे के दौराि वह सवोच्च न्यायालय के न्यायाधीश के सभी अनधकार क्षेत्र, शनक्तयों
और नवशेषानधकारों (और कतगव्यों का निवगहि) का लाभ लेता है।
सेवानिवृत्त न्यायाधीश:
• नकसी भी समय, भारत का मख्ु य न्यायाधीश सवोच्च न्यायालय के सेवानिवृत्त न्यायाधीश या उच्च न्यायालय के सेवानिवृत्त
न्यायाधीश (जो सवोच्च न्यायालय के न्यायाधीश के रूप में नियनु क्त के नलए नवनधवत रूप से योग्य है) से उच्चतम न्यायालय
के न्यायाधीश के रूप में एक अस्थायी अवनध के नलए कायग करिे का अिरु ोध कर सकता है।
• वह ऐसा के वल राष्ट्रपनत और इस प्कार नियक्त
ु नकए जािे वाले व्यनक्त की पवू ग सहमनत से ही कर सकता है।
• ऐसा न्यायाधीश ऐसे भत्तों का हकदार है जो राष्ट्रपनत निधागररत करें ।
• वह सवोच्च न्यायालय के न्यायाधीश के सभी अनधकार क्षेत्र, शनक्तयों और नवशेषानधकारों का भी लाभ लेगा। लेनकि, उन्हें
अन्यथा सवोच्च न्यायालय का न्यायाधीश िहीं मािा जाएगा
कायगवाहक मख्ु य न्यायाधीश:
राष्ट्रपनत सवोच्च न्यायालय के न्यायाधीश को भारत के कायगवाहक मख्ु य न्यायाधीश के रूप में नियक्त
ु कर सकता है जब:
1. भारत के मख्ु य न्यायाधीश का कायागलय ररक्त है; या
2. भारत के मख्ु य न्यायाधीश अस्थायी रूप से अिपु नस्थत हैं; या
3. भारत के मख्ु य न्यायाधीश अपिे कायागलय के कतगव्यों का पालि करिे में असमथग हैं।

www.iasbaba.com rrs.iasbaba.com Page 91


IASbaba’s Rapid Revision (RaRe) Series Polity Day 1-120 2021
HINDI

Q. 106) निम्िनिनित कथिों पर नवचार करें:


1. अिुच्छे द 143 के तहत भारत के राष्ट्रपनत द्वारा सवोच्च न्यायालय में नकए गए सांदभों का निणगय कम से कम पाांच न्यायाधीशों
की एक पीठ द्वारा नकया जाता है।
2. सनां वधाि भारत की सांसद को सवोच्च न्यायालय की सीट के रूप में नदपली के अलावा अन्य स्थािों या स्थािों को नियक्त

करिे के नलए अनधकृ त करता है।
उपरोक्त में से कौि सा कथि सही हैं?
a) के वल 1
b) के वल 2
c) दोिों 1 और 2
d) ि तो 1 और ि ही 2
Q. 106) Solution (a)
भारत के सवोच्च न्यायालय की प्निया:
• सवोच्च न्यायालय, राष्ट्रपनत के अिुमोदि से, न्यायालय के व्यवहार और प्निया को सामान्य रूप से नवनियनमत करिे के नलए
नियम बिा सकता है।
• अिुच्छे द 143 के तहत राष्ट्रपनत द्वारा नदए गए सांवैधानिक मामलों या सांदभों का निणगय कम से कम पाांच न्यायाधीशों वाली
पीठ द्वारा नकया जाता है।
• अन्य सभी मामलों का निणगय एकल न्यायाधीशों और खांडपीठों द्वारा नकया जाता है।
• निणगय खलु ी अदालत द्वारा नदए जाते हैं। सभी निणगय बहुमत से होते हैं लेनकि यनद नभन्ि होते हैं, तो न्यायाधीश असहमनतपणू ग
निणगय या राय दे सकते हैं
सप्ु ीम कोटग की सीट
• सनां वधाि नदपली को सवोच्च न्यायालय की सीट घोनषत करता है। लेनकि, यह भारत के मख्ु य न्यायाधीश को सवोच्च
न्यायालय की सीट के रूप में अन्य स्थाि या स्थािों को नियक्त
ु करिे के नलए भी अनधकृ त करता है। वह इस सबां धां में राष्ट्रपनत
के अिुमोदि से ही निणगय ले सकता है।
• यह प्ावधाि के वल वैकनपपक है और अनिवायग िहीं है। इसका अथग यह है नक कोई भी न्यायालय राष्ट्रपनत या मख्ु य न्यायाधीश
को सवोच्च न्यायालय की सीट के रूप में नकसी अन्य स्थाि को नियक्त
ु करिे के नलए कोई निदेश िहीं दे सकता है।

Q. 107) सनं वधाि सवोच्च न्यायािय के स्वतंत्र और निष्ट्पक्ष कामकाज की सुरक्षा और सुनिनित करिे के निए निम्िनिनित
में से कौि सा प्रावधाि प्रदाि करता है?

www.iasbaba.com rrs.iasbaba.com Page 92


IASbaba’s Rapid Revision (RaRe) Series Polity Day 1-120 2021
HINDI

1. सनां वधाि सांसद या राज्य नवधािमांडल में सवोच्च न्यायालय के न्यायाधीशों के उिके कतगव्यों के निवगहि में आचरण के सांबांध
में नकसी भी चचाग पर रोक लगाता है।
2. भारत का मख्ु य न्यायाधीश कायगपानलका के हस्तक्षेप के नबिा सवोच्च न्यायालय के अनधकाररयों और सेवकों की नियनु क्त
कर सकता है।
3. ससां द सवोच्च न्यायालय के अनधकार क्षेत्र और शनक्तयों को कम करिे के नलए अनधकृ त िहीं है।
िीचे नदए गए कूटों में से सही उत्तर चुनिए:
a) के वल 1 और 2
b) के वल 2 और 3
c) के वल 1 और 3
d) 1, 2 और 3
Q. 107) Solution (d)
सांनवधाि िे सवोच्च न्यायालय के स्वतांत्र और निष्ट्पक्ष कामकाज की सरु क्षा और सनु िनित करिे के नलए निम्िनलनखत प्ावधाि नकए हैं:
1. नियनु क्त का तरीका: सवोच्च न्यायालय के न्यायाधीशों की नियनु क्त राष्ट्रपनत (नजसका अथग है कै नबिेट) द्वारा न्यायपानलका के
सदस्यों (यािी, सवोच्च न्यायालय और उच्च न्यायालयों के न्यायाधीश) के परामशग से की जाती है। यह प्ावधाि कायगपानलका
के पणू ग नववेक को कम करता है और साथ ही यह सनु िनित करता है नक न्यानयक नियनु क्तयाां नकसी भी राजिीनतक या व्यावहाररक
नवचारों पर आधाररत िहीं हैं।
2. कायगकाल की सरु क्षा: सवोच्च न्यायालय के न्यायाधीशों को कायगकाल की सरु क्षा प्दाि की जाती है। उन्हें राष्ट्रपनत द्वारा
के वल सांनवधाि में उनपलनखत तरीके और आधार पर ही पद से हटाया जा सकता है। इसका अथग यह है नक वे राष्ट्रपनत के
प्सादपयगन्त अपिे पद पर िहीं रहते, यद्यनप उिकी नियनु क्त उिके द्वारा की जाती है। यह इस त्य से स्पष्ट है नक अब तक
सवोच्च न्यायालय के नकसी भी न्यायाधीश को हटाया (या महानभयोग) िहीं लगाया गया है।
3. निनित सेवा शतें: उच्चतम न्यायालय के न्यायाधीशों के वेति, भत्ते, नवशेषानधकार, अवकाश और पेंशि समय-समय पर
ससां द द्वारा निधागररत की जाती हैं। नवत्तीय आपात नस्थनत को छोडकर उिकी नियनु क्त के बाद उन्हें उिके अलाभकारी स्वरूप
में िहीं बदला जा सकता है। इस प्कार, सवोच्च न्यायालय के न्यायाधीशों की सेवा की शतें उिके कायगकाल के दौराि समाि
रहती हैं।
4. सनां चत निनध पर प्भाररत व्यय: न्यायाधीशों और कमगचाररयों के वेति, भत्ते और पेंशि के साथ-साथ सवोच्च न्यायालय के
सभी प्शासनिक व्यय भारत की सांनचत निनध पर भाररत होते हैं। इस प्कार, वे सांसद द्वारा गैर-मतदाि योग्य हैं (हालाांनक इसके
द्वारा उि पर चचाग की जा सकती है)।
5. न्यायाधीशों के आचरण पर चचाग िहीं की जा सकती: सांनवधाि सांसद या राज्य नवधािमांडल में सवोच्च न्यायालय के
न्यायाधीशों के अपिे कतगव्यों के निवगहि के सांबांध में नकसी भी चचाग को प्नतबांनधत करता है, नसवाय इसके नक जब महानभयोग
प्स्ताव सांसद के नवचाराधीि हो।

www.iasbaba.com rrs.iasbaba.com Page 93


IASbaba’s Rapid Revision (RaRe) Series Polity Day 1-120 2021
HINDI

6. सेवानिवृनत्त के बाद अभ्यास पर प्नतबांध: सवोच्च न्यायालय के सेवानिवृत्त न्यायाधीशों को भारत के क्षेत्र के भीतर नकसी भी
न्यायालय में या नकसी भी प्ानधकरण के समक्ष याचिा करिे या कायग करिे से प्नतबांनधत नकया जाता है। यह सनु िनित करता
है नक वे भनवष्ट्य के पक्ष की आशा में नकसी का पक्ष िहीं लेते हैं।
7. अवमाििा के नलए दडां देिे की शनक्त: सवोच्च न्यायालय नकसी भी व्यनक्त को उसकी अवमाििा के नलए दनां डत कर सकता
है। इस प्कार, इसके कायों और निणगयों की नकसी के द्वारा आलोचिा और नवरोध िहीं नकया जा सकता है। यह शनक्त सवोच्च
न्यायालय में अपिे अनधकार, गररमा और सम्माि को बिाए रखिे के नलए निनहत है।
8. अपिे कमगचाररयों को नियक्त
ु करिे की स्वतत्रां ता: भारत का मख्ु य न्यायाधीश कायगपानलका के हस्तक्षेप के नबिा सवोच्च
न्यायालय के अनधकाररयों और कमगचाररयों को नियक्त ु कर सकता है। वह उिकी सेवा की शतें भी निधागररत कर सकता है।
9. इसके अनधकार क्षेत्र में कटौती िहीं की जा सकती: ससां द सवोच्च न्यायालय के अनधकार क्षेत्र और शनक्तयों को कम करिे के
नलए अनधकृ त िहीं है। सनां वधाि िे सवोच्च न्यायालय को नवनभन्ि प्कार के क्षेत्रानधकार की गारांटी दी है। हालााँनक, ससां द इसे
बढ़ा सकती है।
10. कायगपानलका से अलगाव: सांनवधाि राज्य को सावगजनिक सेवाओ ां में न्यायपानलका को कायगपानलका से अलग करिे के नलए
कदम उठािे का निदेश देता है। इसका मतलब है नक कायगकारी अनधकाररयों के पास न्यानयक शनक्तयाां िहीं होिी चानहए।
ितीजति, इसके लागू होिे पर, न्यानयक प्शासि में कायगकारी अनधकाररयों की भनू मका समाप्त हो गई।

Q. 108) निम्िनिनित में से नकस मामिे के निए सवोच्च न्यायािय के पास अिन्य मि
ू क्षेत्रानधकार है?
1. कें द्र और एक या अनधक राज्यों के बीच नववाद
2. दो या दो से अनधक राज्यों के बीच नववाद
3. अतां रागज्यीय जल नववाद
िीचे नदए गए कूटों में से सही उत्तर चुनिए:
a) के वल 1
b) के वल 1 और 2
c) के वल 3
d) के वल 2 और 3
Q. 108) Solution (b)
एक सांघीय अदालत के रूप में, सप्ु ीम कोटग भारतीय सांघ की नवनभन्ि इकाइयों के बीच नववादों का फै सला करता है। अनधक नवस्तृत
रूप से, कोई नववाद:
a. कें द्र और एक या अनधक राज्यों के बीच; या
b. एक तरफ कें द्र और नकसी राज्य या राज्यों के बीच और दसू री तरफ एक या अनधक अन्य राज्यों के बीच; या

www.iasbaba.com rrs.iasbaba.com Page 94


IASbaba’s Rapid Revision (RaRe) Series Polity Day 1-120 2021
HINDI

c. दो या दो से अनधक राज्यों के बीच।


उपरोक्त सांघीय नववादों में, सवोच्च न्यायालय के पास अिन्य मल ू अनधकार क्षेत्र है। अिन्य साधि, कोई अन्य अदालत ऐसे नववादों
और मल ू साधिों, ऐसे नववादों को पहली बार में सिु िे की शनक्त, अपील के माध्यम से िहीं तय कर सकती है।
सवोच्च न्यायालय के अिन्य मल
ू क्षेत्रानधकार के सांबांध में, दो नबांदओ
ु ां पर ध्याि नदया जािा चानहए:
• नववाद में एक प्श्न शानमल होिा चानहए (चाहे काििू या त्य का) नजस पर काििू ी अनधकार का अनस्तत्व या सीमा निभगर
करती है। इस प्कार, राजिीनतक प्कृ नत के प्श्नों को इससे बाहर रखा गया है।
• नकसी निजी िागररक द्वारा कें द्र या राज्य के नखलाफ सप्ु ीम कोटग में लाए गए नकसी भी मक
ु दमे पर इसके तहत नवचार िहीं
नकया जा सकता है।
इसके अलावा, सवोच्च न्यायालय के इस अनधकार क्षेत्र का नवस्तार निम्िनलनखत तक िहीं है::
a. नकसी सांनवधाि-पवू ग सांनध, समझौते, वाचा, समझोते , सिद या अन्य समाि साधि से उत्पन्ि होिे वाला नववाद।
b. नकसी सांनध, समझौते आनद से उत्पन्ि नववाद, जो नवशेष रूप से यह प्दाि करता है नक उक्त अनधकार क्षेत्र ऐसे नववाद तक
सीनमत िहीं है।
c. अांतरागज्यीय जल नववाद।
d. नवत्त आयोग को भेजे गए मामलें
e. कें द्र और राज्यों के बीच कुछ खचों और पेंशि का समायोजि।
f. कें द्र और राज्यों के बीच वानणनज्यक प्कृ नत का साधारण नववाद।
g. कें द्र के नखलाफ एक राज्य द्वारा हजागिे की वसल
ू ी।
1961 में, सवोच्च न्यायालय के मल ू अनधकार क्षेत्र के तहत पहला मक ु दमा पनिम बांगाल द्वारा कें द्र के नखलाफ लाया गया था। राज्य
सरकार िे ससां द द्वारा पाररत कोयला क्षेत्र (अनधग्रहण और नवकास) अनधनियम, 1957 की सवां ैधानिक वैधता को चिु ौती दी। हालानां क,
सप्ु ीम कोटग िे अनधनियम की वैधता को बरकरार रखते हुए मक ु दमे को खाररज कर नदया।

Q. 109) निम्िनिनित कथिों पर नवचार करें:


1. सवोच्च न्यायालय का ररट अनधकार क्षेत्र मौनलक और अिन्य है।
2. सवोच्च न्यायालय के वल मौनलक अनधकारों के प्वतगि के नलए ररट जारी कर सकता है।
उपरोक्त में से कौि सा कथि सही हैं?
a) के वल 1
b) के वल 2

www.iasbaba.com rrs.iasbaba.com Page 95


IASbaba’s Rapid Revision (RaRe) Series Polity Day 1-120 2021
HINDI

c) दोिों 1 और 2
d) ि तो 1 और ि ही 2
Q. 109) Solution (b)
सप्ु ीम कोटग का ररट क्षेत्रानधकार:
• सनां वधाि िे िागररकों के मौनलक अनधकारों के गारांटर और रक्षक के रूप में सवोच्च न्यायालय का गठि नकया है।
• सप्ु ीम कोटग को एक पीनडत िागररक के मौनलक अनधकारों के प्वतगि के नलए बदां ी प्त्यक्षीकरण, परमादेश, प्नतषेध, अनधकार
पृच्छा और उत्प्ेषण सनहत ररटें जारी करिे की शनक्त दी गई है। इस सबां धां में, सवोच्च न्यायालय का मल
ू अनधकार क्षेत्र इस
अथग में है नक एक पीनडत िागररक जरूरी िहीं नक अपील के माध्यम से सीधे सवोच्च न्यायालय जा सकता है,
• हालानां क, सप्ु ीम कोटग का ररट अनधकार क्षेत्र अिन्य िहीं है। उच्च न्यायालयों को मौनलक अनधकारों के प्वतगि के नलए ररट
जारी करिे का भी अनधकार है। इसका मतलब है, जब नकसी िागररक के मौनलक अनधकारों का उपलघां ि होता है, तो पीनडत
पक्ष के पास सीधे उच्च न्यायालय या सवोच्च न्यायालय जािे का नवकपप होता है।
• इसनलए, सांघीय नववादों के सांबांध में सवोच्च न्यायालय का मल ू अनधकार क्षेत्र मौनलक अनधकारों से सांबांनधत नववादों के
सांबांध में अपिे मलू क्षेत्रानधकार से अलग है। पहले मामले में, यह अिन्य है और दसू रे मामले में, यह उच्च न्यायालयों के
अनधकार क्षेत्र के साथ समवती है। इसके अलावा, पहले मामले में शानमल पक्ष सांघ (कें द्र और राज्य) की इकाइयााँ हैं जबनक
दसू रे मामले में नववाद एक िागररक और सरकार (कें द्र या राज्य) के बीच है।
• सवोच्च न्यायालय और उच्च न्यायालय के ररट क्षेत्रानधकार में भी अतां र है। सवोच्च न्यायालय के वल मौनलक अनधकारों के
प्वतगि के नलए ररट जारी कर सकता है ि नक अन्य उद्देश्यों के नलए। दसू री ओर, उच्च न्यायालय ि के वल मौनलक अनधकारों
के प्वतगि के नलए बनपक अन्य उद्देश्यों के नलए भी ररट जारी कर सकता है।
• इसका अथग है नक उच्च न्यायालय का ररट क्षेत्रानधकार सवोच्च न्यायालय की तल
ु िा में व्यापक है। लेनकि, सांसद सवोच्च
न्यायालय को अन्य प्योजिों के नलए भी ररट जारी करिे की शनक्त प्दाि कर सकती है।

Q.110) निम्िनिनित कथिों पर नवचार करें:


1. भारत का सवोच्च न्यायालय काििू या सावगजनिक महत्व के नकसी भी प्श्न पर राष्ट्रपनत को अपिी राय देिे से मिा कर सकता
है जो उत्पन्ि हुआ है या जो उत्पन्ि होिे की सभां ाविा है।
2. भारत के सवोच्च न्यायालय द्वारा राष्ट्रपनत को व्यक्त की गई राय के वल सलाहकारी है, न्यानयक अनभव्यनक्त िहीं।
उपरोक्त में से कौि सा कथि सही हैं?
a) के वल 1
b) के वल 2
c) दोिों 1 और 2

www.iasbaba.com rrs.iasbaba.com Page 96


IASbaba’s Rapid Revision (RaRe) Series Polity Day 1-120 2021
HINDI

d) ि तो 1 और ि ही 2
Q. 110) Solution (c)
सवोच्च न्यायालय के सलाहकार क्षेत्रानधकार:
सनां वधाि (अिुच्छे द 143) राष्ट्रपनत को दो श्रेनणयों के मामलों में सवोच्च न्यायालय की राय लेिे के नलए अनधकृ त करता है:
a. कािूि या सावगजनिक महत्व के त्य के नकसी भी प्श्न पर जो उत्पन्ि हो गया हो या नजसके उत्पन्ि होिे की सांभाविा हो।
b. नकसी भी सांनवधाि-पवू ग सांनध, समझौते, प्सांनवदा, अिुबांध, सिद या इसी तरह के अन्य साधिों से उत्पन्ि होिे वाले नकसी भी
नववाद पर।
पहले मामले में, सवोच्च न्यायालय राष्ट्रपनत को अपिी राय देिे के नलए निनवदा दे सकता है या मिा कर सकता है। लेनकि, दसू रे मामले
में, सप्ु ीम कोटग को राष्ट्रपनत को अपिी राय देिी चानहए।
दोिों ही मामलों में, सवोच्च न्यायालय द्वारा दी गई राय के वल सलाहकारी है, न्यानयक अनभव्यनक्त िहीं। इसनलए, यह राष्ट्रपनत पर
बाध्यकारी िहीं है; वह राय का पालि कर सकता है या िहीं कर सकता है। हालााँनक, यह सरकार को उसके द्वारा तय नकए जािे वाले
मामले पर एक आनधकाररक कािूिी राय रखिे की सनु वधा देता है।
Q. 111) भारत में उच्च न्यायाियों के संबध
ं में निम्िनिनित में से कौि सा/से कथि सही है/हैं?
1. उच्च न्यायालय के न्यायाधीशों की नियनु क्त सांबांनधत राज्य के राज्यपाल द्वारा की जाती है।
2. उच्च न्यायालय के मख्ु य न्यायाधीश की नियनु क्त राष्ट्रपनत द्वारा भारत के मख्ु य न्यायाधीश और सांबांनधत राज्य के राज्यपाल
के परामशग से की जाती है।
िीचे नदए गए कूटों में से सही उत्तर चुनिए:
a) के वल 1
b) के वल 2
c) दोिों 1 और 2
d) ि तो 1 और ि ही 2
Q. 111) Solution (b)
उच्च न्यायालय की स्थापिा भारत में 1862 में हुई जब कलकत्ता, बॉम्बे और मद्रास में उच्च न्यायालय स्थानपत नकए गए।
भारत का सनां वधाि प्त्येक राज्य के नलए एक उच्च न्यायालय का प्ावधाि करता है, लेनकि 1956 के सातवें सश ां ोधि अनधनियम िे
सांसद को दो या दो से अनधक राज्यों या दो या अनधक राज्यों और एक कें द्र शानसत प्देश के नलए एक सामान्य उच्च न्यायालय स्थानपत
करिे के नलए अनधकृ त नकया गया।
न्यायाधीशों की नियनु क्त:

www.iasbaba.com rrs.iasbaba.com Page 97


IASbaba’s Rapid Revision (RaRe) Series Polity Day 1-120 2021
HINDI

• उच्च न्यायालय के न्यायाधीशों की नियनु क्त राष्ट्रपनत करता है।


• मख्ु य न्यायाधीश की नियनु क्त राष्ट्रपनत द्वारा भारत के मख्ु य न्यायाधीश और सांबांनधत राज्य के राज्यपाल के परामशग के बाद
की जाती है।
• अन्य न्यायाधीशों की नियनु क्त के नलए सांबांनधत उच्च न्यायालय के मख्ु य न्यायाधीश से भी परामशग नकया जाता है। दो या दो
से अनधक राज्यों के नलए एक सामान्य उच्च न्यायालय के मामले में, राष्ट्रपनत द्वारा सभी सबां नां धत राज्यों के राज्यपालों से
परामशग नकया जाता है।

Q. 112) निम्िनिनित कथिों पर नवचार करें:


1. एक उच्च न्यायालय के न्यायाधीश को उसी तरीके से और उसी आधार पर हटाया जा सकता है नजस तरह से उच्चतम न्यायालय
के न्यायाधीश को हटाया जा सकता है।
2. उच्च न्यायालय का न्यायाधीश 65 वषग की आयु प्ाप्त करिे तक पद धारण करता है।
3. राष्ट्रपनत भारत के मख्ु य न्यायाधीश से परामशग करके नकसी न्यायाधीश को एक उच्च न्यायालय से दसू रे उच्च न्यायालय में
स्थािाांतररत कर सकता है।
उपरोक्त में से कौि सा कथि सही हैं?
a) के वल 1 और 2
b) के वल 2 और 3
c) के वल 1 और 3
d) 1, 2 और 3
Q. 112) Solution (c)
उच्च न्यायालय के न्यायाधीशों का कायगकाल:
सनां वधाि िे नकसी उच्च न्यायालय के न्यायाधीश का कायगकाल निनित िहीं है। हालााँनक, यह इस सांबांध में निम्िनलनखत चार प्ावधाि
करता है:
1. वह 62 वषग की आयु प्ाप्त करिे तक पद पर बिे रहते हैं। उिकी उम्र के सबां धां में कोई भी प्श्न भारत के मख्ु य न्यायाधीश के
परामशग के बाद राष्ट्रपनत द्वारा तय नकया जािा है और राष्ट्रपनत का निणगय अांनतम होता है।
2. वह राष्ट्रपनत को पत्र नलखकर अपिे पद से त्यागपत्र दे सकता है।
3. उन्हें सांसद की नसफाररश पर राष्ट्रपनत द्वारा उिके पद से हटाया जा सकता है।
4. जब वह सवोच्च न्यायालय के न्यायाधीश के रूप में नियक्त
ु होता है या जब उसे नकसी अन्य उच्च न्यायालय में स्थािाांतररत
नकया जाता है तो वह अपिा पद खाली कर देता है।

www.iasbaba.com rrs.iasbaba.com Page 98


IASbaba’s Rapid Revision (RaRe) Series Polity Day 1-120 2021
HINDI

उच्च न्यायालय के न्यायाधीशों को हटािा:


• नकसी उच्च न्यायालय के न्यायाधीश को राष्ट्रपनत के आदेश से उसके पद से हटाया जा सकता है।
• राष्ट्रपनत हटािे का आदेश तभी जारी कर सकता है जब सांसद द्वारा उसे उसी सत्र में इस तरह के निष्ट्कासि के नलए एक
अनभभाषण प्स्ततु नकया गया हो।
• अनभभाषण को सांसद के प्त्येक सदि के नवशेष बहुमत (अथागत उस सदि की कुल सदस्यता का बहुमत और उस सदि के
उपनस्थत और मतदाि करिे वाले सदस्यों के कम से कम दो-नतहाई बहुमत) द्वारा समनथगत होिा चानहए।
• निष्ट्कासि के आधार दो नसद्ध कदाचार या अक्षमता हैं।
• इस प्कार, एक उच्च न्यायालय के न्यायाधीश को उसी तरीके से और उसी आधार पर हटाया जा सकता है नजस तरह से
उच्चतम न्यायालय के न्यायाधीश को हटाया जा सकता है।
उच्च न्यायालय के न्यायाधीशों का स्थािाांतरण:
• राष्ट्रपनत भारत के मख्ु य न्यायाधीश से परामशग करके नकसी न्यायाधीश को एक उच्च न्यायालय से दसू रे उच्च न्यायालय में
स्थािाांतररत कर सकता है।
• स्थािान्तरण पर, वह अपिे वेति के अनतररक्त ऐसा प्नतपरू क भत्ता प्ाप्त करिे का हकदार है जैसा नक सांसद द्वारा निधागररत
नकया जा सकता है।

Q. 113) निम्िनिनित कथिों पर नवचार करें:


1. उच्च न्यायालय के न्यायाधीशों की सेवा की शतें सांबांनधत राज्य नवधािमांडल द्वारा निधागररत की जाती हैं।
2. उच्च न्यायालय के न्यायाधीशों के वेति, भत्ते और पेंशि राज्य की सनां चत निनध पर भाररत होते हैं।
उपरोक्त में से कौि सा कथि सही हैं?
a) के वल 1
b) के वल 2
c) दोिों 1 और 2
d) ि तो 1 और ि ही 2
Q. 113) Solution (d)
उच्च न्यायालय के न्यायाधीशों के वेति, भत्ते, नवशेषानधकार, अवकाश और पेंशि समय-समय पर सांसद द्वारा निधागररत नकए जाते
हैं। लेनकि, नवत्तीय आपात नस्थनत के अलावा उिकी नियनु क्त के बाद उन्हें उिके अलाभकारी स्वरूप में िहीं बदला जा सकता है। इस
प्कार, उच्च न्यायालय के न्यायाधीशों की सेवा की शतें उिके कायगकाल के दौराि समाि रहती हैं।

www.iasbaba.com rrs.iasbaba.com Page 99


IASbaba’s Rapid Revision (RaRe) Series Polity Day 1-120 2021
HINDI

न्यायाधीशों के वेति और भत्ते, कमगचाररयों के वेति, भत्ते और पेंशि के साथ-साथ एक उच्च न्यायालय के प्शासनिक खचग राज्य की
सांनचत निनध पर प्भाररत होते हैं। इस प्कार, वे राज्य नवधानयका द्वारा गैर-मतदाि योग्य हैं (हालाांनक इसके द्वारा उि पर चचाग की जा
सकती है)। यहाां यह ध्याि नदया जािा चानहए नक उच्च न्यायालय के न्यायाधीश की पेंशि भारत की सांनचत निनध पर प्भाररत होती है,
ि नक राज्य पर।

Q. 114) निम्िनिनित कथिों पर नवचार करें:


1. ससां द और राज्य नवधािसभाओ ां के सदस्यों के चिु ाव से सबां नां धत नववादों के सबां धां में उच्च न्यायालय का मल
ू अनधकार क्षेत्र
है।
2. िागररकों के मौनलक अनधकारों के प्वतगि के सबां धां में उच्च न्यायालयों का मल
ू और अिन्य क्षेत्रानधकार है।
3. उच्च न्यायालय का ररट क्षेत्रानधकार उच्चतम न्यायालय के ररट क्षेत्रानधकार के समवती है।
उपरोक्त में से कौि सा कथि सही हैं?
a) के वल 1 और 2
b) के वल 2
c) के वल 3
d) के वल 1 और 3
Q. 114) Solution (d)
उच्च न्यायालय का मल
ू क्षेत्रानधकार:
इसका अथग है उच्च न्यायालय की प्थम दृष्टया में नववादों को सिु िे की शनक्त, अपील के माध्यम से िहीं। यह निम्िनलनखत तक नवस्तृत
है:
• अदालत की अवमाििा के मामले।
• सांसद और राज्य नवधािसभाओ ां के सदस्यों के चिु ाव से सांबांनधत नववाद।
• राजस्व मामले या राजस्व सांग्रह में आदेनशत या नकए गए कायग के सांबधां में।
• िागररकों के मौनलक अनधकारों का प्वतगि।
• मामलों को एक अधीिस्थ अदालत से सांनवधाि की व्याख्या से सांबनां धत अपिी फाइल में स्थािाांतररत करिे का आदेश नदया
गया।
• चार उच्च न्यायालयों (यािी, कलकत्ता, बॉम्बे, मद्रास और नदपली उच्च न्यायालय) के पास उच्च मपू य के मामलों में मल

िागररक अनधकार क्षेत्र है।

www.iasbaba.com rrs.iasbaba.com Page 100


IASbaba’s Rapid Revision (RaRe) Series Polity Day 1-120 2021
HINDI

1973 से पहले, कलकत्ता, बॉम्बे और मद्रास उच्च न्यायालयों में भी मल


ू आपरानधक अनधकार क्षेत्र था। इसे आपरानधक प्निया सांनहता,
1973 द्वारा परू ी तरह से समाप्त कर नदया गया था।
उच्च न्यायालयों का ररट क्षेत्रानधकार:
• सनां वधाि का अिुच्छे द 226 एक उच्च न्यायालय को िागररकों के मौनलक अनधकारों के प्वतगि और नकसी अन्य उद्देश्य के
नलए बदां ी प्त्यक्षीकरण, परमादेश, उत्प्ेषण, प्नतषेध और अनधकार-पृच्छा सनहत ररट जारी करिे का अनधकार देता है।
• वाक्याशां 'नकसी अन्य उद्देश्य के नलए' एक सामान्य काििू ी अनधकार के प्वतगि को सदां नभगत करता है। उच्च न्यायालय नकसी
भी व्यनक्त, प्ानधकरण और सरकार को ि के वल अपिे क्षेत्रीय अनधकार क्षेत्र के भीतर बनपक अपिे क्षेत्रीय अनधकार क्षेत्र के
बाहर भी ररट जारी कर सकता है यनद उसके प्ादेनशक क्षेत्रानधकार में कारग वाई का कारण उत्पन्ि हो।
• उच्च न्यायालय का ररट क्षेत्रानधकार (अिच्ु छे द 226 के तहत) अिन्य िहीं है, बनपक सवोच्च न्यायालय के ररट क्षेत्रानधकार
(अिच्ु छे द 32 के तहत) के समवती है। इसका मतलब है, जब नकसी िागररक के मौनलक अनधकारों का उपलघां ि होता है, तो
पीनडत पक्ष के पास सीधे उच्च न्यायालय या सवोच्च न्यायालय जािे का नवकपप होता है।
• हालााँनक, उच्च न्यायालय का ररट क्षेत्रानधकार सवोच्च न्यायालय की तुलिा में व्यापक है। ऐसा इसनलए है, क्योंनक सवोच्च
न्यायालय के वल मौनलक अनधकारों के प्वतगि के नलए ररट जारी कर सकता है, ि नक नकसी अन्य उद्देश्य के नलए, अथागत यह
उस मामले तक नवस्ताररत िहीं होता है जहाां एक सामान्य काििू ी अनधकार के उपलांघि का आरोप लगाया जाता है।

Q. 115) न्यानयक समीक्षा (Judicial Review) के संदभा में निम्िनिनित कथिों पर नवचार कीनजएः
1. सवोच्च न्यायालय और उच्च न्यायालयों पर न्यानयक समीक्षा की शनक्त सांनवधाि द्वारा प्दाि की गई है।
2. न्यायपानलका पर न्यानयक समीक्षा की शनक्त सनां वधाि की मल
ू नवशेषता है।
उपरोक्त में से कौि सा कथि सही हैं?
a) के वल 1
b) के वल 2
c) दोिों 1 और 2
d) ि तो 1 और ि ही 2
Q. 115) Solution (c)
न्यानयक समीक्षा कें द्र और राज्य दोिों सरकारों के नवधायी अनधनियमों और कायगकारी आदेशों की सांवैधानिकता की जाचां करिे के नलए
न्यायपानलका की शनक्त है। जाचां करिे पर, यनद वे सांनवधाि (अनधकारातीत) का उपलांघि करते पाए जाते हैं, तो उन्हें न्यायपानलका द्वारा
अवैध, असांवैधानिक और अमान्य (ररक्त और शन्ू य) घोनषत नकया जा सकता है। ितीजति, उन्हें सरकार द्वारा लागू िहीं नकया जा सकता
है।

www.iasbaba.com rrs.iasbaba.com Page 101


IASbaba’s Rapid Revision (RaRe) Series Polity Day 1-120 2021
HINDI

न्यानयक समीक्षा का नसद्धाांत सांयक्त


ु राज्य अमेररका में उत्पन्ि और नवकनसत हुआ। यह पहली बार अमेररकी सप्ु ीम कोटग के तत्कालीि
मख्ु य न्यायाधीश जॉि माशगल द्वारा माबगरी बिाम मैनडसि (1803) के प्नसद्ध मामले में प्नतपानदत नकया गया था।
दसू री ओर, भारत में, सांनवधाि स्वयां न्यायपानलका (उच्चतम न्यायालय और उच्च न्यायालयों दोिों) को न्यानयक समीक्षा की शनक्त
प्दाि करता है। इसके अलावा, सवोच्च न्यायालय िे न्यानयक समीक्षा की शनक्त को सांनवधाि की मल ू नवशेषता या सांनवधाि की मल ू
सांरचिा के एक तत्व के रूप में घोनषत नकया है। इसनलए, न्यानयक समीक्षा की शनक्त को सांवैधानिक सशां ोधि द्वारा भी कम या बाहर िहीं
नकया जा सकता है।

Q. 116) निम्िनिनित में से कौि सा अिच्ु छे द स्पष्ट रूप से सवोच्च न्यायािय और उच्च न्यायाियों को न्यानयक समीक्षा
की शनक्त प्रदाि करता है?
1. अिुच्छे द 13
2. अिुच्छे द 133
3. अिुच्छे द 246
िीचे नदए गए कूटों में से सही उत्तर चुनिए:
a) के वल 1 और 2
b) के वल 2 और 3
c) के वल 1 और 3
d) 1, 2 और 3
Q. 116) Solution (d)
निम्िनलनखत अिच्ु छे द स्पष्ट रूप से सवोच्च न्यायालय और उच्च न्यायालयों को न्यानयक समीक्षा की शनक्त प्दाि करते हैं:
1. अिुच्छे द 13 घोनषत करता है नक सभी कािूि जो मौनलक अनधकारों के साथ असांगत हैं या उन्हें न्यिू करते हैं, वे अमान्य और
शन्ू य होंगे।
2. अिुच्छे द 32 मौनलक अनधकारों के प्वतगि के नलए सवोच्च न्यायालय जािे के अनधकार की गारांटी देता है और सवोच्च
न्यायालय को उस उद्देश्य के नलए निदेश या आदेश या ररट जारी करिे का अनधकार देता है।
3. अिुच्छे द 131 कें द्र-राज्य और अतां र-राज्यीय नववादों में सवोच्च न्यायालय के मल
ू अनधकार क्षेत्र का प्ावधाि करता है।
4. अिुच्छे द 132 सांवैधानिक मामलों में सवोच्च न्यायालय के अपीलीय क्षेत्रानधकार का प्ावधाि करता है।
5. अिुच्छे द 133 दीवािी मामलों में सवोच्च न्यायालय के अपीलीय क्षेत्रानधकार का प्ावधाि करता है।
6. अिुच्छे द 134 आपरानधक मामलों में सवोच्च न्यायालय के अपीलीय क्षेत्रानधकार का प्ावधाि करता है।
7. अिुच्छे द 134-ए उच्च न्यायालयों से सवोच्च न्यायालय में अपील के नलए प्माण पत्र से सांबांनधत है।

www.iasbaba.com rrs.iasbaba.com Page 102


IASbaba’s Rapid Revision (RaRe) Series Polity Day 1-120 2021
HINDI

8. अिुच्छे द 135 सप्ु ीम कोटग को नकसी भी पवू ग सांनवधाि कािूि के तहत सांघीय न्यायालय के अनधकार क्षेत्र और शनक्तयों का
प्योग करिे का अनधकार देता है।
9. अिुच्छे द 136 सप्ु ीम कोटग को नकसी भी अदालत या नरब्यिू ल (सैन्य न्यायानधकरण और कोटग माशगल को छोडकर) से अपील
करिे के नलए नवशेष अिमु नत देिे का अनधकार देता है।
10. अिच्ु छे द 143 राष्ट्रपनत को काििू या त्य के नकसी भी प्श्न और नकसी भी पवू ग-सनां वधाि काििू ी मामलों पर सवोच्च
न्यायालय की राय लेिे के नलए अनधकृ त करता है।
11. अिच्ु छे द 226 उच्च न्यायालयों को मौनलक अनधकारों के प्वतगि और नकसी अन्य उद्देश्य के नलए निदेश या आदेश या ररट
जारी करिे का अनधकार देता है।
12. अिच्ु छे द 227 उच्च न्यायालयों को उिके सबां नां धत क्षेत्रीय अनधकार क्षेत्र (सैन्य न्यायालयों या न्यायानधकरणों को छोडकर)
के भीतर सभी न्यायालयों और न्यायानधकरणों पर अधीक्षण की शनक्त प्दाि करता है।
13. अिुच्छे द 245 सांसद और राज्यों के नवधािमांडलों द्वारा बिाए गए कािूिों की क्षेत्रीय सीमा से सांबांनधत है।
14. अिुच्छे द 246 सांसद और राज्यों के नवधाि मांडलों (अथागत सांघ सचू ी, राज्य सचू ी और समवती सचू ी) द्वारा बिाए गए कािूिों
की नवषय वस्तु से सांबांनधत है।
15. अिुच्छे द 251 और 254 में प्ावधाि है नक कें द्रीय काििू और राज्य के कािूि के बीच सांघषग की नस्थनत में, कें द्रीय कािूि
राज्य के कािूि पर हावी होता है और राज्य का कािूि शन्ू य होगा।
16. अिुच्छे द 372 पवू ग सांनवधाि काििू ों के प्वृत्त/लागू रहिे से सांबांनधत है।

Q. 117) निम्िनिनित कथिों पर नवचार करें:


1. न्यानयक सनियता (judicial activism) की अवधारणा यक
ू े में उत्पन्ि और नवकनसत हुई।
2. जिनहत यानचका की अवधारणा न्यानयक सनियता का एक उत्पाद है।
उपरोक्त में से कौि सा कथि सही हैं?
a) के वल 1
b) के वल 2
c) दोिों 1 और 2
d) ि तो 1 और ि ही 2
Q. 117) Solution (b)

www.iasbaba.com rrs.iasbaba.com Page 103


IASbaba’s Rapid Revision (RaRe) Series Polity Day 1-120 2021
HINDI

न्यानयक सनियता िागररकों के अनधकारों की सरु क्षा और समाज में न्याय को बढ़ावा देिे में न्यायपानलका द्वारा निभाई गई सनिय भनू मका
को दशागती है। दसू रे शब्दों में, इसका तात्पयग सरकार के अन्य दो अांगों (नवधानयका और कायगपानलका) को अपिे सांवैधानिक कतगव्यों का
निवगहि करिे के नलए दबाव डालिे के नलए न्यायपानलका द्वारा निभाई गई मख ु र भनू मका से है।
न्यानयक सनियता की अवधारणा सांयक्त ु राज्य अमेररका में उत्पन्ि और नवकनसत हुई। भारत में, न्यानयक सनियता का नसद्धातां 1970
के दशक के मध्य में पेश नकया गया था। न्यायमनू तग वी.आर. कृ ष्ट्णा अय्यर, न्यायमनू तग पी.एि. भगवती, न्यायमनू तग ओ. नचन्िप्पा रे ड्डी
और न्यायमनू तग डी.ए. देसाई िे देश में न्यानयक सनियता की िींव रखी।
न्यानयक सनियता न्यानयक शनक्त का प्योग करिे का एक तरीका है जो न्यायाधीशों को प्गनतशील और िई सामानजक िीनतयों के पक्ष
में न्यानयक नमसाल के सामान्य रूप से सख्ती से पालि करिे के नलए प्ेररत करती है। यह आमतौर पर सोशल इजां ीनियररांग के नलए कॉल
करिे वाले निणगय द्वारा नचनित नकया जाता है, और कभी-कभी ये निणगय नवधायी और कायगकारी मामलों में घसु पैठ का प्नतनिनधत्व करते
हैं।
जिनहत यानचका (PIL) की अवधारणा 1960 के दशक में सांयक्त ु राज्य अमेररका में उत्पन्ि और नवकनसत हुई थी। भारत में, जिनहत
यानचका सवोच्च न्यायालय की न्यानयक सनियता भनू मका का एक उत्पाद है। इसे 1980 के दशक की शरुु आत में पेश नकया गया था।
न्यायमनू तग वी.आर. कृ ष्ट्णा अय्यर और न्यायमनू तग पी.एि.भगवती जिनहत यानचका की अवधारणा के अग्रदतू थे।
जिनहत यानचका के तहत, कोई भी जि-नहतैषी िागररक या सामानजक सांगठि नकसी भी व्यनक्त या व्यनक्तयों के समहू के अनधकारों के
प्वतगि के नलए अदालत का रुख कर सकता है, जो अपिी गरीबी या अज्ञािता या सामानजक या आनथगक रूप से वांनचत नस्थनत के कारण
खदु उपायों के नलए अदालत का दरवाजा खटखटािे में असमथग हैं। इस प्कार, एक जिनहत यानचका में, 'पयागप्त रुनच' रखिे वाला जिता
का कोई भी सदस्य अन्य व्यनक्तयों के अनधकारों को लागू करिे और एक सामान्य नशकायत के निवारण के नलए अदालत का दरवाजा
खटखटा सकता है।

Q. 118) जिनहत यानचका के रूप में निम्िनिनित में से नकस श्रेणी के अंतगात आिे वािी यानचकाओ ं पर नवचार नकया जा
सकता है?
1. नचनकत्सा सांस्थाि में प्वेश के सांबधां में यानचका
2. श्रनमकों को न्यिू तम मजदरू ी का भगु ताि ि करिा
3. मामला दजग करिे से इक
ां ार करिे पर पनु लस के नखलाफ यानचका
िीचे नदए गए कूटों में से सही उत्तर चुनिए:
a) के वल 1
b) के वल 1 और 2
c) के वल 3
d) के वल 2 और 3
Q. 118) Solution (d)

www.iasbaba.com rrs.iasbaba.com Page 104


IASbaba’s Rapid Revision (RaRe) Series Polity Day 1-120 2021
HINDI

जिनहत यानचका का दायरा (PIL):


1998 में, सप्ु ीम कोटग िे जिनहत यानचका के रूप में प्ाप्त पत्रों या यानचकाओ ां पर नवचार करिे के नलए नदशानिदेशों का एक सेट तैयार
नकया।
इि नदशानिदेशों को 1993 और 2003 में सांशोनधत नकया गया था। उिके अिुसार, के वल निम्िनलनखत श्रेनणयों के अतां गगत आिे वाले
पत्रों या यानचकाओ ां को आम तौर पर जिनहत यानचका के रूप में मािा जाएगा:
• बांधआ
ु मजदरू ी का मामला
• उपेनक्षत बच्चे
• श्रनमकों को न्यिू तम मजदरू ी का भगु ताि ि करिा और आकनस्मक श्रनमकों का शोषण और श्रम कािूिों के उपलांघि की
नशकायतें (व्यनक्तगत मामलों को छोडकर)
• जेलों से प्ताडिा, समय से पहले ररहाई और 14 साल की जेल परू ी होिे के बाद ररहाई की माांग, जेल में मौत, स्थािाांतरण,
निजी बॉन्ड पर ररहाई, मौनलक अनधकार के रूप में त्वररत सिु वाई की नशकायत करिे वाली यानचकाएां
• मामला दजग करिे से इक
ां ार करिे, पनु लस द्वारा प्तानडत करिे और पनु लस नहरासत में मौत के मामले में पनु लस के नखलाफ
यानचकाएां
• मनहलाओ ां पर अत्याचार, नवशेष रूप से दपु हि के उत्पीडि, दपु हि को जलािे, बलात्कार, हत्या, अपहरण आनद के नखलाफ
यानचकाएां।
• अिुसनू चत जानत और अिुसनू चत जिजानत और आनथगक रूप से नपछडे वगग के व्यनक्तयों से सह-ग्रामीणों या पनु लस द्वारा
ग्रामीणों के उत्पीडि या प्ताडिा की नशकायत करिे वाली यानचकाएां
• पयागवरण प्दषू ण, पाररनस्थनतक सतां ुलि की गडबडी, दवाओ,ां खाद्य अपनमश्रण, नवरासत और सांस्कृ नत के रखरखाव, प्ाचीि
वस्तएु ,ां वि और वन्य जीवि और सावगजनिक महत्व के अन्य मामलों से सबां नां धत यानचकाएां
• दगां ा पीनडतों की यानचका
• पाररवाररक पेंशि
निम्िनलनखत श्रेनणयों के अतां गगत आिे वाले मामलों पर जिनहत यानचका के रूप में नवचार िहीं नकया जाएगा:
• जमींदार-नकरायेदार मामले
• सेवा मामले और पेंशि और ग्रेच्यटु ी से सांबांनधत मामले
• नचनकत्सा और अन्य शैक्षनणक सस्ां थाि में प्वेश
• उच्च न्यायालयों और अधीिस्थ न्यायालयों में लांनबत मामलों की शीघ्र सिु वाई के नलए यानचकाएां

Q. 119) निम्िनिनित कथिों पर नवचार करें:

www.iasbaba.com rrs.iasbaba.com Page 105


IASbaba’s Rapid Revision (RaRe) Series Polity Day 1-120 2021
HINDI

1. नजला न्यायाधीश के रूप में नियक्त


ु होिे वाले व्यनक्त को दस साल के नलए वकील या अनभवक्ता होिा चानहए था।
2. नजला न्यायालयों और अन्य अधीिस्थ न्यायालयों पर नियांत्रण भारत के सवोच्च न्यायालय में निनहत है।
उपरोक्त में से कौि सा कथि सही हैं?
a) के वल 1
b) के वल 2
c) दोिों 1 और 2
d) ि तो 1 और ि ही 2
Q. 119) Solution (d)
सनां वधाि के भाग VI में अिच्ु छे द 233 से 237 अधीिस्थ न्यायालयों के सगां ठि को नवनियनमत करिे और कायगपानलका से उिकी
स्वतत्रां ता सनु िनित करिे के नलए निम्िनलनखत प्ावधाि करता है।
नजला न्यायाधीशों की नियनु क्त:
नकसी राज्य में नजला न्यायाधीशों की नियनु क्त, तैिाती और पदोन्िनत राज्य के राज्यपाल द्वारा उच्च न्यायालय के परामशग से की जाती
है।
नजला न्यायाधीश के रूप में नियक्त
ु होिे वाले व्यनक्त में निम्िनलनखत योग्यताएां होिी चानहए:
• वह पहले से ही कें द्र या राज्य सरकार की सेवा में िहीं होिा चानहए।
• उसे सात साल तक वकील या अनभवक्ता होिा चानहए था।
• उिकी नियनु क्त के नलए उच्च न्यायालय द्वारा नसफाररश की जािी चानहए।
अधीिस्थ न्यायालयों पर नियांत्रण:
नजला न्यायालयों और अन्य अधीिस्थ न्यायालयों पर नियत्रां ण, नजसमें राज्य की न्यानयक सेवा से सबां नां धत व्यनक्तयों की तैिाती, पदोन्िनत
और छुट्टी शानमल है और नजला न्यायाधीश के पद से नकसी भी पद पर हैं, उच्च न्यायालय में निनहत हैं।

Q.120) निम्िनिनित कथिों पर नवचार करें:


1. नजला न्यायाधीश के पास दीवािी और आपरानधक दोिों मामलों में मल
ू और अपीलीय क्षेत्रानधकार है।
2. मख्ु य न्यानयक मनजस्रेट के पास आजीवि कारावास और मृत्यदु डां सनहत नकसी भी सजा को लागू करिे की शनक्त है।
उपरोक्त में से कौि सा कथि सही हैं?
a) के वल 1

www.iasbaba.com rrs.iasbaba.com Page 106


IASbaba’s Rapid Revision (RaRe) Series Polity Day 1-120 2021
HINDI

b) के वल 2
c) दोिों 1 और 2
d) ि तो 1 और ि ही 2
Q. 120) Solution (a)
अधीिस्थ न्यायालय:
• अधीिस्थ न्यायपानलका की सगां ठिात्मक सरां चिा, अनधकार क्षेत्र और िामकरण राज्यों द्वारा निधागररत नकया जाता है। इसनलए,
वे एक राज्य से दसू रे राज्य में थोडे नभन्ि होते हैं।
• नजला न्यायाधीश नजले में सवोच्च न्यानयक प्ानधकरण है। उसके पास दीवािी और आपरानधक दोिों मामलों में मल
ू और
अपीलीय क्षेत्रानधकार है। दसू रे शब्दों में, नजला न्यायाधीश सत्र न्यायाधीश भी होता है।
• जब वह दीवािी मामलों को देखता है, तो उसे नजला न्यायाधीश के रूप में जािा जाता है और जब वह आपरानधक मामलों
की सिु वाई करता है, तो उसे सत्र न्यायाधीश कहा जाता है।
• नजला न्यायाधीश न्यानयक और प्शासनिक दोिों शनक्तयों का प्योग करता है। उसके पास नजले के सभी अधीिस्थ न्यायालयों
पर पयगवेक्षी शनक्तयााँ भी हैं। उिके आदेशों और निणगयों के नखलाफ उच्च न्यायालय में अपील की जाती है।
• सत्र न्यायाधीश को आजीवि कारावास और मृत्यदु डां सनहत कोई भी दडां अनधरोनपत करिे की शनक्त होती है। हालानां क, उिके
द्वारा पाररत मौत की सजा उच्च न्यायालय द्वारा पनु ष्ट के अधीि है, चाहे कोई अपील हो या िहीं।
• नजला एवां सत्र न्यायालय के िीचे दीवािी पक्ष में अधीिस्थ न्यायाधीश का न्यायालय तथा आपरानधक पक्ष में मख्ु य न्यानयक
मनजस्रेट का न्यायालय है। अधीिस्थ न्यायाधीश दीवािी मक ु दमों पर असीनमत आनथगक क्षेत्रानधकार का प्योग करता है।
• मख्ु य न्यानयक मनजस्रेट उि आपरानधक मामलों का फै सला करता है नजिके नलए सात वषग की अवनध तक कारावास की
सजा हो सकती है।
• सबसे निचले स्तर पर, दीवािी पक्ष पर, मांनु सफ का न्यायालय है और आपरानधक पक्ष पर, न्यानयक मनजस्रेट का न्यायालय
है। मांनु सफ के पास सीनमत क्षेत्रानधकार होता है और वह छोटी आनथगक नहस्सेदारी के दीवािी मामलों का फै सला करता है।
• न्यानयक मनजस्रेट आपरानधक मामलों की कोनशश करता है नजसमें तीि साल तक की कै द की सजा हो सकती है।
Q. 121) भारत के चुिाव आयोग के संबध
ं में निम्िनिनित कथिों पर नवचार करें:
1. मख्ु य चिु ाव आयक्त
ु और अन्य चिु ाव आयक्त
ु ों के बीच मतभेद के मामले में मख्ु य चिु ाव आयक्त
ु के पास निणागयक मतानधकार
होता है।
2. मख्ु य चिु ाव आयक्त
ु और अन्य चिु ाव आयक्त
ु राष्ट्रपनत के प्सादपयंत्ि अपिे पद पर बिे रहते हैं।
3. सांनवधाि िे सेवानिवृत्त होिे वाले चिु ाव आयक्त
ु ों को सरकार द्वारा आगे नकसी भी नियनु क्त से वांनचत िहीं नकया है।
उपरोक्त में से कौि सा कथि सही हैं?

www.iasbaba.com rrs.iasbaba.com Page 107


IASbaba’s Rapid Revision (RaRe) Series Polity Day 1-120 2021
HINDI

a) के वल 1 और 2
b) के वल 2 और 3
c) के वल 3
d) 1, 2 और 3
Q. 121) Solution (c)
चिु ाव आयोग देश में स्वतांत्र और निष्ट्पक्ष चिु ाव सनु िनित करिे के नलए सीधे भारत के सांनवधाि द्वारा स्थानपत एक स्थायी और स्वतांत्र
निकाय है।
मख्ु य चिु ाव आयक्तु और दो अन्य चिु ाव आयक्त
ु ों के पास समाि शनक्तयाां हैं और उन्हें समाि वेति, भत्ते और अन्य अिल
ु नब्धयाां नमलती
हैं, जो नक मख्ु य चिु ाव आयक्त
ु और/या दो के बीच मतभेद के मामले में सवोच्च न्यायालय के न्यायाधीश के समाि हैं। इस मामले पर
आयोग द्वारा बहुमत से निणगय नलया जाता है।
मख्ु य चिु ाव आयक्त
ु को कायगकाल की सरु क्षा प्दाि की जाती है। सवोच्च न्यायालय के न्यायाधीश के समाि तरीके से और उसी आधार
पर उन्हें उिके पद से हटाया िहीं जा सकता है। दसू रे शब्दों में, उसे राष्ट्रपनत द्वारा सांसद के दोिों सदिों द्वारा नवशेष बहुमत से पाररत एक
प्स्ताव के आधार पर, या तो सानबत दव्ु यगवहार या अक्षमता के आधार पर हटाया जा सकता है। इस प्कार, वह राष्ट्रपनत के प्सादपयगन्त
अपिा पद धारण िहीं करता, यद्यनप वह उसके द्वारा नियक्त ु नकया जाता है।
हालानां क सांनवधाि िे चिु ाव आयोग की स्वतत्रां ता और निष्ट्पक्षता को सरु नक्षत रखिे और सनु िनित करिे की माांग की है, लेनकि कुछ
खानमयों पर ध्याि नदया जा सकता है, जैस,े
• सनां वधाि िे चिु ाव आयोग के सदस्यों की योग्यता (काििू ी, शैनक्षक, प्शासनिक या न्यानयक) निधागररत िहीं की है।
• सनां वधाि िे चिु ाव आयोग के सदस्यों के कायगकाल को निनदगष्ट िहीं नकया है।
• सनां वधाि िे सेवानिवृत्त होिे वाले चिु ाव आयक्त
ु ों को सरकार द्वारा आगे नकसी भी नियनु क्त से वनां चत िहीं नकया है।

Q. 122) सनं वधाि का अिच्ु छे द 324 चुिाव आयोग को नकिके चुिावों के अधीक्षण, निदेशि और नियत्रं ण की शनक्त प्रदाि
करता है:
1. भारतीय सांसद
2. पचां ायतों
3. भारत के उपराष्ट्रपनत का पद
4. राज्यों के राज्यपालों का पद
िीचे नदए गए कूटों में से सही उत्तर चुनिए:
a) के वल 1

www.iasbaba.com rrs.iasbaba.com Page 108


IASbaba’s Rapid Revision (RaRe) Series Polity Day 1-120 2021
HINDI

b) के वल 1 और 3
c) के वल 1, 2 और 3
d) के वल 1, 3 और 4
Q. 122) Solution (b)
सनां वधाि के अिुच्छे द 324 में यह प्ावधाि है नक सांसद, राज्य नवधािसभाओ,ां भारत के राष्ट्रपनत के कायागलय और भारत के उपराष्ट्रपनत
के कायागलय के चिु ावों के अधीक्षण, निदेशि और नियत्रां ण की शनक्त चिु ाव आयोग में निनहत होगी।
इस प्कार, चिु ाव आयोग इस अथग में एक अनखल भारतीय निकाय है नक यह कें द्र सरकार और राज्य सरकारों दोिों के नलए समाि है।
चिु ाव आयोग का राज्यों में पचां ायतों और िगर पानलकाओ ां के चिु ावों से कोई सरोकार िहीं है। इसके नलए भारत का सांनवधाि एक
अलग राज्य चिु ाव आयोग का प्ावधाि करता है।
राज्यपाल को ि तो प्त्यक्ष रूप से जिता द्वारा चिु ा जाता है और ि ही परोक्ष रूप से नकसी नवशेष रूप से गनठत निवागचक मडां ल द्वारा
निवागनचत नकया जाता है जैसा नक राष्ट्रपनत के मामले में होता है। उन्हें राष्ट्रपनत द्वारा उिके हाथ और महु र के तहत वारांट द्वारा नियक्त

नकया जाता है।

Q. 123) निम्िनिनित कथिों पर नवचार करें:


1. भारत के सांनवधाि के अिुच्छे द 284 में नवत्त आयोग का प्ावधाि है।
2. सनां वधाि राष्ट्रपनत को नवत्त आयोग के सदस्यों की योग्यता और उिके चयि के तरीके को निधागररत करिे के नलए अनधकृ त
करता है।
3. नवत्त आयोग भारत के राष्ट्रपनत को उि नसद्धातां ों के बारे में नसफाररशें करता है जो भारत के आकनस्मक निनध से राज्यों को
सहायता अिुदाि को नियांनत्रत करिा चानहए।
उपरोक्त में से कौि सा कथि सही हैं?
a) के वल 1 और 2
b) के वल 2
c) के वल 3
d) के वल 1 और 3
Q. 123) Solution (b)
भारत के सांनवधाि का अिुच्छे द 280 एक अधग न्यानयक निकाय के रूप में एक नवत्त आयोग का प्ावधाि करता है। इसका गठि भारत
के राष्ट्रपनत द्वारा प्त्येक पााँचवें वषग या ऐसे पवू ग समय में नकया जाता है, जब वह आवश्यक समझे।
नवत्त आयोग में एक अध्यक्ष और चार अन्य सदस्य होते हैं नजन्हें राष्ट्रपनत द्वारा नियक्त
ु नकया जाता है। वे राष्ट्रपनत द्वारा अपिे आदेश में
निनदगष्ट अवनध के नलए पद धारण करते हैं। वे पिु निगयनु क्त के पात्र हैं।
www.iasbaba.com rrs.iasbaba.com Page 109
IASbaba’s Rapid Revision (RaRe) Series Polity Day 1-120 2021
HINDI

सनां वधाि सांसद को आयोग के सदस्यों की योग्यता और उिके चयि के तरीके को निधागररत करिे के नलए अनधकृ त करता है। तदिुसार,
सांसद िे आयोग के अध्यक्ष और सदस्यों की योग्यताएां निनदगष्ट की हैं। अध्यक्ष को सावगजनिक मामलों में अिुभव रखिे वाला व्यनक्त होिा
चानहए और चार अन्य सदस्यों को निम्िनलनखत में से चिु ा जािा चानहए:
1. उच्च न्यायालय का एक न्यायाधीश या एक के रूप में नियक्त
ु होिे के योग्य।
2. एक व्यनक्त नजसे सरकार के नवत्त और खातों का नवशेष ज्ञाि है।
3. एक व्यनक्त नजसे नवत्तीय मामलों और प्शासि में व्यापक अिभु व है।
4. एक व्यनक्त नजसे अथगशास्त्र का नवशेष ज्ञाि है
नवत्त आयोग को निम्िनलनखत मामलों पर भारत के राष्ट्रपनत को नसफाररशें करिे की आवश्यकता है:
1. कें द्र और राज्यों के बीच साझा नकए जािे वाले करों की शद्ध
ु आय का नवतरण, और इस तरह की आय के सबां नां धत शेयरों के
राज्यों के बीच आवांटि।
2. कें द्र द्वारा राज्यों को सहायता अिदु ाि को नियांनत्रत करिे वाले नसद्धातां (अथागत, भारत की सांनचत निनध से)।
3. राज्य नवत्त आयोग द्वारा की गई नसफाररशों के आधार पर राज्य में पचां ायतों और िगर पानलकाओ ां के सांसाधिों के अिपु नू तग के
नलए राज्य की सांनचत निनध को बढ़ािे के नलए आवश्यक उपाय।
4. सदृु ढ़ नवत्त के नहत में राष्ट्रपनत द्वारा सांदनभगत कोई अन्य मामला।

Q. 124) वस्तु एवं सेवा कर पररषद (Goods and Services Tax Council) के संदभा में निम्िनिनित कथिों पर नवचार
कीनजएः:
1. अिुच्छे द 279-ए भारत के राष्ट्रपनत को जीएसटी पररषद का गठि करिे का अनधकार देता है।
2. पररषद का प्त्येक निणगय बैठक में उपनस्थत और मतदाि करिे वाले सदस्यों के भाररत मतों के कम से कम दो-नतहाई बहुमत
से नलया जाता है।
3. जीएसटी पररषद की बैठक आयोनजत करिे के नलए कोरम पररषद के कुल सदस्यों की सख्ां या का आधा है।
उपरोक्त में से कौि सा कथि सही हैं?
a) के वल 1 और 2
b) के वल 2
c) के वल 3
d) के वल 1 और 3
Q. 124) Solution (d)

www.iasbaba.com rrs.iasbaba.com Page 110


IASbaba’s Rapid Revision (RaRe) Series Polity Day 1-120 2021
HINDI

2016 के 101वें सांशोधि अनधनियम िे देश में एक िई कर व्यवस्था (यािी वस्तु और सेवा कर - जीएसटी) की शरुु आत का मागग प्शस्त
नकया।
सश
ां ोधि िे सांनवधाि में एक िया अिुच्छे द 279-ए जोडा गया। इस अिुच्छे द िे राष्ट्रपनत को एक आदेश द्वारा जीएसटी पररषद का गठि
करिे का अनधकार नदया। तदिुसार, राष्ट्रपनत िे 2016 में आदेश जारी नकया और पररषद का गठि नकया।
पररषद के निणगय उसकी बैठकों में नलए जाते हैं। पररषद के सदस्यों की कुल सख्ां या का आधा नहस्सा बैठक आयोनजत करिे के नलए
गणपनू तग है। पररषद का प्त्येक निणगय बैठक में उपनस्थत और मतदाि करिे वाले सदस्यों के भाररत मतों के कम से कम तीि-चौथाई बहुमत
से नलया जािा है। निणगय निम्िनलनखत नसद्धातां ों के अिसु ार नलया जाता है:
1. कें द्र सरकार के वोट का भार उस बैठक में डाले गए कुल वोटों का एक नतहाई होगा।
2. सभी राज्य सरकारों के सांयक्त
ु मतों का भार उस बैठक में डाले गए कुल मतों के दो-नतहाई के बराबर होगा।

Q. 125) भाषाई अल्पसंख्यकों के निए नवशेष अनधकारी के सदं भा में, निम्िनिनित कथिों पर नवचार करें:
1. भारत के सांनवधाि के भाग XVIII में भाषाई अपपसांख्यकों के नलए नवशेष अनधकारी का प्ावधाि है।
2. वह छह साल की अवनध के नलए या 65 वषग की आयु प्ाप्त करिे तक, जो भी पहले हो, तक पद धारण करता है।
उपरोक्त में से कौि सा कथि सही हैं?
a) के वल 1
b) के वल 2
c) दोिों 1 और 2
d) ि तो 1 और ि ही 2
Q. 125) Solution (d)
मल
ू रूप से, भारत के सांनवधाि में भाषाई अपपसांख्यकों के नलए नवशेष अनधकारी के सांबांध में कोई प्ावधाि िहीं नकया गया था।
1956 के सातवें सनां वधाि सश
ां ोधि अनधनियम िे सनां वधाि के भाग XVII में एक िया अिच्ु छे द 350-बी जोडा गया।
इस अिुच्छे द में निम्िनलनखत प्ावधाि हैं:
1. भाषाई अपपसख्ां यकों के नलए एक नवशेष अनधकारी होिा चानहए। उन्हें भारत के राष्ट्रपनत द्वारा नियक्त
ु नकया जािा है।
2. यह नवशेष अनधकारी का कतगव्य होगा नक वह सनां वधाि के तहत भाषाई अपपसख्ां यकों के नलए प्दाि नकए गए सरु क्षा उपायों
से सांबांनधत सभी मामलों की जाचां करे ।
सनां वधाि भाषाई अपपसांख्यकों के नलए नवशेष अनधकारी को हटािे के नलए योग्यता, कायगकाल, वेति और भत्ते, सेवा शतों और प्निया
को निनदगष्ट िहीं करता है।

www.iasbaba.com rrs.iasbaba.com Page 111


IASbaba’s Rapid Revision (RaRe) Series Polity Day 1-120 2021
HINDI

Q. 126) भारत के नियंत्रक-महािेिापरीक्षक (CAG) के संदभा में, निम्िनिनित कथिों पर नवचार कीनजएः
1. कै ग छह साल की अवनध या 65 वषग की आयु तक, जो भी पहले हो, तक पद धारण करता है।
2. CAG का वेति उच्च न्यायालय के मख्ु य न्यायाधीश के बराबर होता है।
3. भारत के सांनवधाि को बिाए रखिा CAG का कतगव्य है।
उपरोक्त में से कौि सा कथि सही हैं?
a) के वल 1 और 2
b) के वल 2 और 3
c) के वल 1 और 3
d) 1, 2 और 3
Q. 126) Solution (c)
भारत का सांनवधाि (अिुच्छे द 148) भारत के नियांत्रक और महालेखा परीक्षक (CAG) के एक स्वतत्रां पद का प्ावधाि करता है।
वह छह वषग की अवनध या 65 वषग की आयु तक, जो भी पहले हो, तक पद धारण करता है। वह नकसी भी समय राष्ट्रपनत को त्याग पत्र
सांबोनधत कर अपिे पद से त्यागपत्र दे सकता है। उन्हें राष्ट्रपनत द्वारा उसी आधार पर और उसी तरह से सवोच्च न्यायालय के न्यायाधीश
के रूप में हटाया जा सकता है।
कै ग (CAG) की स्वतांत्रता सनु िनित करिे के नलए उिका वेति और अन्य सेवा शतें सांसद द्वारा निधागररत की जाती हैं। उिका वेति
उच्चतम न्यायालय के न्यायाधीश के बराबर है। उिकी नियनु क्त के बाद ि तो उिके वेति और ि ही अिुपनस्थनत की छुट्टी, पेंशि या
सेवानिवृनत्त की आयु के सबां धां में उिके अनधकारों में उिके अनहत में पररवतगि नकया जा सकता है।
कै ग (CAG) जिता की जेब का सांरक्षक है और कें द्र और राज्य दोिों स्तरों पर देश की सांपणू ग नवत्तीय प्णाली को नियांनत्रत करता
है। उिका कतगव्य नवत्तीय प्शासि के क्षेत्र में भारत के सांनवधाि और सांसद के काििू ों को बिाए रखिा है। वह भारत में सरकार की
लोकताांनत्रक प्णाली की एक कडी है; अन्य सवोच्च न्यायालय, चिु ाव आयोग और सांघ लोक सेवा आयोग हैं।

Q. 127) निम्िनिनित कथिों पर नवचार करें:


1. कै ग (CAG) भारत की सांनचत निनध से धि के मद्दु े पर नियत्रां ण रखता है।
2. कै ग (CAG) प्त्येक राज्य के आकनस्मक निनध और प्त्येक राज्य के सावगजनिक खाते से सभी व्यय का लेखा-जोखा करता
है।
उपरोक्त में से कौि सा कथि सही हैं?
a) के वल 1

www.iasbaba.com rrs.iasbaba.com Page 112


IASbaba’s Rapid Revision (RaRe) Series Polity Day 1-120 2021
HINDI

b) के वल 2
c) दोिों 1 और 2
d) ि तो 1 और ि ही 2
Q. 127) Solution (b)
सांसद और सांनवधाि द्वारा निधागररत कै ग (CAG) के कतगव्य और कायग हैं:
• वह भारत की सनां चत निनध, प्त्येक राज्य की समेनकत निनध और एक नवधाि सभा वाले प्त्येक कें द्र शानसत प्देश की समेनकत
निनध से सभी व्यय से सबां नां धत लेखाओ ां की लेखा परीक्षा करता है।
• वह भारत की आकनस्मकता निनध और भारत के सावगजनिक खाते के साथ-साथ प्त्येक राज्य की आकनस्मक निनध और
प्त्येक राज्य के सावगजनिक खाते से सभी व्यय का लेखा-जोखा करता है।
• वह राष्ट्रपनत को उस प्पत्र के निधागरण के सांबांध में सलाह देता है नजसमें कें द्र और राज्यों के खाते रखे जाएांगे।
• वह नकसी भी कर या शपु क की शद्ध
ु आय का पता लगाता है और प्मानणत करता है। उसका प्माणपत्र अांनतम है। 'शद्ध

आय' का अथग है कर या शपु क की आय घटाकर सांग्रह की लागत।
• वह सांसद की लोक लेखा सनमनत के मागगदशगक, नमत्र और दाशगनिक के रूप में कायग करते हैं।
कै ग (CAG) तीि ऑनडट ररपोटग राष्ट्रपनत को सौंपता है- नवनियोग खातों पर ऑनडट ररपोटग, नवत्त खातों पर ऑनडट ररपोटग और सावगजनिक
उपिमों पर ऑनडट ररपोटग। राष्ट्रपनत इि ररपोटों को सांसद के दोिों सदिों के समक्ष रखता है। इसके बाद, लोक लेखा सनमनत उिकी
जाांच करती है और अपिे निष्ट्कषों को सांसद को ररपोटग करती है।
भारत का सांनवधाि सीएजी को नियांत्रक और महालेखा परीक्षक के रूप में देखता है। हालानां क, व्यवहार में, कै ग (CAG) के वल एक
महालेखा परीक्षक की भनू मका निभा रहा है, नियांत्रक की िहीं। दसू रे शब्दों में, 'समेनकत निनध से धि के मद्दु े पर सीएजी का कोई नियांत्रण
िहीं है और कई नवभाग सीएजी से नवनशष्ट प्ानधकरण के नबिा चेक जारी करके धि निकालिे के नलए अनधकृ त हैं, जो के वल लेखा
परीक्षा चरण में सबां नां धत है जब व्यय पहले ही हो चक
ु ा है'।

Q. 128) निम्िनिनित कथिों पर नवचार करें:


1. भारत का महान्यायवादी/अटािी जिरल सनां वधाि के अिच्ु छे द 143 के तहत राष्ट्रपनत द्वारा सवोच्च न्यायालय में नकए गए
नकसी भी सांदभग में भारत सरकार का प्नतनिनधत्व करता है।
2. भारत के महान्यायवादी को सांसद के दोिों सदिों और उिकी सांयक्त
ु बैठक की कायगवाही में बोलिे, मतदाि करिे और भाग
लेिे का अनधकार है।
उपरोक्त में से कौि सा कथि सही हैं?
a) के वल 1
b) के वल 2

www.iasbaba.com rrs.iasbaba.com Page 113


IASbaba’s Rapid Revision (RaRe) Series Polity Day 1-120 2021
HINDI

c) दोिों 1 और 2
d) ि तो 1 और ि ही 2
Q. 128) Solution (a)
सनां वधाि (अिुच्छे द 76) िे भारत के नलए महान्यायवादी के कायागलय के नलए प्ावधाि नकया है। वह देश के सवोच्च कािूि अनधकारी
हैं।
भारत सरकार के मख्ु य काििू अनधकारी के रूप में, एजी के कतगव्यों में निम्िनलनखत शानमल हैं:
• ऐसे काििू ी मामलों पर भारत सरकार को सलाह देिा, जो राष्ट्रपनत द्वारा उसे भेजे जाते हैं।
• काििू ी चररत्र के ऐसे अन्य कतगव्यों का पालि करिा जो राष्ट्रपनत द्वारा उसे सौंपे जाते हैं।
• सांनवधाि या नकसी अन्य काििू द्वारा उसे प्दत्त कायों का निवगहि करिा।
राष्ट्रपनत िे महान्यायवादी/अटािी जिरल को निम्िनलनखत कतगव्य सौंपे हैं:
• भारत सरकार की ओर से सभी मामलों में सवोच्च न्यायालय में उपनस्थत होिा नजसमें भारत सरकार का सांबांध है।
• सनां वधाि के अिुच्छे द 143 के तहत राष्ट्रपनत द्वारा सवोच्च न्यायालय में नकए गए नकसी भी सांदभग में भारत सरकार का
प्नतनिनधत्व करिा।
• नकसी भी मामले में, नजसमें भारत सरकार का सांबांध है, नकसी भी उच्च न्यायालय में (जब भारत सरकार द्वारा आवश्यक हो)
उपनस्थत होिा।
अपिे आनधकाररक कतगव्यों के प्दशगि में, महान्यायवादी को भारत के क्षेत्र में सभी अदालतों में सिु वाई का अनधकार है। इसके अलावा,
उसे सांसद के दोिों सदिों या उिकी सांयक्त
ु बैठक और सांसद की नकसी भी सनमनत की कायगवाही में बोलिे और भाग लेिे का अनधकार
है, नजसमें उसे सदस्य िानमत नकया जा सकता है, लेनकि वोट देिे के अनधकार के नबिा। वह उि सभी नवशेषानधकारों और उन्मनु क्तयों
का आिदां लेता है जो एक सांसद सदस्य के नलए उपलब्ध हैं।

Q. 129) निम्िनिनित कथिों पर नवचार करें:


1. भारत के सांनवधाि के अिुच्छे द 76 में भारत के सॉनलनसटर जिरल के पद का प्ावधाि है।
2. भारत के सॉनलनसटर जिरल को वे सभी नवशेषानधकार और उन्मनु क्तयाां प्ाप्त हैं जो एक सांसद सदस्य के नलए उपलब्ध हैं।
उपरोक्त में से कौि सा कथि सही हैं?
a) के वल 1
b) के वल 2
c) दोिों 1 और 2

www.iasbaba.com rrs.iasbaba.com Page 114


IASbaba’s Rapid Revision (RaRe) Series Polity Day 1-120 2021
HINDI

d) ि तो 1 और ि ही 2
Q. 129) Solution (d)
महान्यायवादी/अटािी जिरल (एजी) के अलावा, भारत सरकार के अन्य नवनध अनधकारी हैं। वे भारत के सॉनलनसटर जिरल और भारत
के अनतररक्त सॉनलनसटर जिरल हैं।
वे एजी को उसकी आनधकाररक नजम्मेदाररयों को परू ा करिे में सहायता करते हैं।
के वल एजी का पद सांनवधाि द्वारा निनमगत है। दसू रे शब्दों में, अिुच्छे द 76 में सॉनलनसटर जिरल और अनतररक्त सॉनलनसटर जिरल का
उपलेख िहीं है।
प्धाि मत्रां ी की अध्यक्षता में कै नबिेट की नियनु क्त सनमनत द्वारा तीि साल की अवनध के नलए सॉनलनसटर जिरल की नियनु क्त की जाती
है।
भारत के सॉनलनसटर जिरल को कोई सांसदीय नवशेषानधकार प्ाप्त िहीं हैं।

Q.130) निम्िनिनित कथिों पर नवचार करें:


1. राज्य के महानधवक्ता/सानलनसटर जिरल का पद भारत के महानधवक्ता के कायागलय से मेल खाता है।
2. महानधवक्ता/सानलनसटर जिरल की नियनु क्त भारत के राष्ट्रपनत द्वारा की जाती है।
उपरोक्त में से कौि सा कथि सही हैं?
a) के वल 1
b) के वल 2
c) दोिों 1 और 2
d) ि तो 1 और ि ही 2
Q. 130) Solution (d)
सनां वधाि (अिच्ु छे द 165) िे राज्यों के नलए महानधवक्ता/सानलनसटर जिरल के पद की व्यवस्था की है। वह राज्य के सवोच्च काििू
अनधकारी हैं। इस प्कार वह भारत के महान्यायवादी से मेल खाता है।
महानधवक्ता की नियनु क्त राज्यपाल करता है। वह ऐसा व्यनक्त होिा चानहए जो उच्च न्यायालय का न्यायाधीश नियक्त
ु होिे के योग्य
हो। दसू रे शब्दों में, उसे भारत का िागररक होिा चानहए और दस साल तक न्यानयक पद पर रहिा चानहए या दस साल तक उच्च
न्यायालय का वकील होिा चानहए।
वह राज्यपाल के प्सादपयगन्त पद धारण करता है। परांपरागत रूप से, वह इस्तीफा दे देता है जब सरकार (मांनत्रपररषद) इस्तीफा दे देती है
या उसे बदल नदया जाता है, क्योंनक उसे उसकी सलाह पर नियक्त
ु नकया जाता है।

www.iasbaba.com rrs.iasbaba.com Page 115


IASbaba’s Rapid Revision (RaRe) Series Polity Day 1-120 2021
HINDI

राज्य में सरकार के मख्ु य नवनध अनधकारी के रूप में, महानधवक्ता के कतगव्यों में निम्िनलनखत शानमल हैं:
• राज्य सरकार को ऐसे कािूिी मामलों पर सलाह देिा जो राज्यपाल द्वारा उसे भेजे जाते हैं।
• काििू ी चररत्र के ऐसे अन्य कतगव्यों का पालि करिा जो राज्यपाल द्वारा उसे सौंपे जाते हैं।
• सांनवधाि या नकसी अन्य काििू द्वारा उसे प्दत्त कायों का निवगहि करिा।
Q. 131) निम्िनिनित में से कौि-सा/से संवैधानिक निकाय िहीं है/हैं?
1. राज्य लोक सेवा आयोग
2. अपपसख्ां यकों के नलए राष्ट्रीय आयोग
3. राज्य सचू िा आयोग
4. अिसु नू चत जिजानत के नलए राष्ट्रीय आयोग
िीचे नदए गए कूटों में से सही उत्तर चुनिए:
a) के वल 1 और 2
b) के वल 2 और 3
c) के वल 3 और 4
d) के वल 1 और 4
Q. 131) Solution (b)
1. राज्य लोक सेवा आयोग:
• यह एक सांवैधानिक निकाय है। भारत के सांनवधाि के भाग XIV में अिुच्छे द 315 से 323 में भारत सघां के नलए लोक सेवा
आयोग और प्त्येक राज्य के नलए एक लोक सेवा आयोग की स्थापिा का प्ावधाि है।
• सनां वधाि के अिुच्छे दों के समाि सेट (अथागत, भाग XIV में 315 से 323) लोक सेवा आयोग की सरां चिा, नियनु क्त और
सदस्यों को हटािे, शनक्त और कायों और स्वतत्रां ता से सबां नां धत हैं।
• प्त्येक राज्य में राज्य लोक सेवा आयोग राज्य सेवाओ ां में भती के नलए परीक्षा आयोनजत करिे और अिश
ु ासिात्मक मामलों
पर राज्यपाल को सलाह देिे के नलए नजम्मेदार है।

2. अपपसांख्यकों के नलए राष्ट्रीय आयोग:


कें द्र सरकार िे राष्ट्रीय अपपसांख्यक आयोग अनधनियम, 1992 के तहत राष्ट्रीय अपपसांख्यक आयोग (NCM) की स्थापिा की। इस
प्कार यह एक वैधानिक निकाय है।
आयोग के निम्िनलनखत कायग हैं::

www.iasbaba.com rrs.iasbaba.com Page 116


IASbaba’s Rapid Revision (RaRe) Series Polity Day 1-120 2021
HINDI

• सांघ और राज्यों के अधीि अपपसांख्यकों के नवकास की प्गनत का मपू याांकि करिा।


• सनां वधाि और सांसद और राज्य नवधािमांडलों द्वारा अनधनियनमत कािूिों में प्दाि नकए गए सरु क्षा उपायों के कामकाज की
निगरािी करिा।
• कें द्र सरकार या राज्य सरकारों द्वारा अपपसांख्यकों के नहतों की सांरक्षण के नलए रक्षोपायों के प्भावी कायागन्वयि के नलए
नसफाररशें करिा।
• अपपसख्ां यकों के अनधकारों और सरु क्षा उपायों से वनां चत करिे के सबां धां में नवनशष्ट नशकायतों की जाचां करें और ऐसे मामलों
को उपयक्त
ु प्ानधकाररयों के साथ उठाए।ां
• अपपसख्ां यकों के प्नत नकसी भी प्कार के भेदभाव से उत्पन्ि होिे वाली समस्याओ ां का अध्ययि करािा और उन्हें दरू करिे
के उपायों की नसफाररश करिा।
3. राज्य सचू िा आयोग:
• सचू िा का अनधकार अनधनियम 2005 राज्य स्तर पर राज्य सचू िा आयोग के गठि का प्ावधाि करता है।
• राज्य सचू िा आयोग एक उच्चानधकार प्ाप्त स्वतत्रां निकाय है जो अन्य बातों के साथ-साथ इससे की गई नशकायतों को देखता
है और अपीलों का निणगय करता है।
• यह सांबांनधत राज्य सरकार के तहत कायागलयों, नवत्तीय सांस्थािों, सावगजनिक क्षेत्र के उपिमों आनद से सांबांनधत नशकायतों
और अपीलों पर नवचार करता है।
4. राष्ट्रीय अिुसनू चत जिजानत आयोग (STs)
राष्ट्रीय अिुसनू चत जिजानत आयोग (STs) इस अथग में एक सांवैधानिक निकाय है नक यह सीधे सांनवधाि के अिुच्छे द 338-ए द्वारा
स्थानपत नकया गया है।
आयोग के कायग हैं:
• अिुसनू चत जिजानतयों के नलए सवां ैधानिक और अन्य कािूिी सरु क्षा उपायों से सांबांनधत सभी मामलों की जाचां और निगरािी
करिा और उिके कामकाज का मपू याांकि करिा;
• अिुसनू चत जिजानतयों के अनधकारों और सरु क्षा उपायों से वांनचत करिे के सांबांध में नवनशष्ट नशकायतों की जाचां करिा;
• अिुसनू चत जिजानतयों के सामानजक आनथगक नवकास की योजिा प्निया में भाग लेिा और सलाह देिा और सांघ या राज्य
के तहत उिके नवकास की प्गनत का मपू याांकि करिा;
• राष्ट्रपनत को वानषगक रूप से और ऐसे अन्य समयों पर प्स्तुत करिा जो वह उनचत समझे, उि रक्षोपायों के कायगकरण पर ररपोटग
देिा।

Q. 132) िीनत आयोग (NITI Aayog) के संदभा में निम्िनिनित कथिों पर नवचार कीनजएः

www.iasbaba.com rrs.iasbaba.com Page 117


IASbaba’s Rapid Revision (RaRe) Series Polity Day 1-120 2021
HINDI

1. यह एक सांनवधािेत्तर निकाय है।


2. राज्यों के राज्यपाल और कें द्र शानसत प्देशों के उपराज्यपाल िीनत आयोग की सचां ालि पररषद के सदस्य हैं।
3. िीनत आयोग के उपाध्यक्ष को कै नबिेट मत्रां ी का दजाग प्ाप्त है।
उपरोक्त में से कौि सा कथि सही हैं?
a) के वल 1 और 2
b) के वल 2 और 3
c) के वल 1 और 3
d) 1, 2 और 3
Q. 132) Solution (c)
िीनत आयोग भारत सरकार की प्मख
ु िीनत 'नथक
ां टैंक' है, जो नदशात्मक और िीनतगत इिपटु दोिों प्दाि करता है। भारत सरकार के नलए
रणिीनतक और दीघगकानलक िीनतयाां और कायगिम तैयार करते समय, िीनत आयोग कें द्र और राज्यों को प्ासांनगक तकिीकी सलाह भी
प्दाि करता है।
िीनत आयोग, योजिा आयोग की तरह, भारत सरकार (यािी, कें द्रीय मनां त्रमांडल) के एक कायगकारी प्स्ताव द्वारा बिाया गया था। इसनलए,
यह ि तो एक सांवैधानिक निकाय है और ि ही एक वैधानिक निकाय है। दसू रे शब्दों में, यह एक गैर-सांवैधानिक या सांनवधािेत्तर निकाय
(अथागत, सांनवधाि द्वारा निनमगत िहीं) और एक गैर-सानां वनधक निकाय (सांसद के अनधनियम द्वारा निनमगत िहीं) है।
िीनत आयोग की गवनिंग काउांनसल में सभी राज्यों के मख्ु यमत्रां ी, नवधानयकाओ ां के साथ कें द्र शानसत प्देशों के मख्ु यमांत्री (यािी, नदपली,
पडु ु चरे ी और जम्मू और कश्मीर) और अन्य कें द्र शानसत प्देशों के उपराज्यपाल शानमल हैं।
िीनत आयोग के उपाध्यक्ष की नियनु क्त भारत के प्धाि मत्रां ी द्वारा की जाती है। उन्हें कै नबिेट मत्रां ी का दजाग प्ाप्त है।

Q. 133) राष्ट्रीय मािवानधकार आयोग (NHRC) के संदभा में निम्िनिनित कथिों पर नवचार कीनजएः
1. अध्यक्ष भारत का सेवानिवृत्त मख्ु य न्यायाधीश या उच्च न्यायालय का सेवानिवृत्त मख्ु य न्यायाधीश होिा चानहए।
2. राष्ट्रीय मािवानधकार आयोग (NHRC) के अध्यक्ष और सदस्यों की नियनु क्त राष्ट्रपनत द्वारा भारत के मख्ु य न्यायाधीश की
अध्यक्षता वाली एक सनमनत की नसफाररशों पर की जाती है।
3. आयोग को उस तारीख से एक वषग की समानप्त के बाद नकसी भी मामले की जाचां करिे का अनधकार िहीं है, नजस नदि मािव
अनधकारों का उपलांघि करिे वाला कायग नकया गया था।
उपरोक्त में से कौि सा कथि सही हैं?
a) के वल 1 और 2

www.iasbaba.com rrs.iasbaba.com Page 118


IASbaba’s Rapid Revision (RaRe) Series Polity Day 1-120 2021
HINDI

b) के वल 2 और 3
c) के वल 3
d) 1, 2 और 3
Q. 133) Solution (c)
राष्ट्रीय मािवानधकार आयोग एक साांनवनधक (और सांवैधानिक िहीं) निकाय है। इसकी स्थापिा 1993 में सांसद द्वारा अनधनियनमत एक
कािूि, अथागत् मािवानधकार सांरक्षण अनधनियम, 1993 के तहत की गई थी।
नियनु क्त:
• आयोग एक बहु-सदस्यीय निकाय है नजसमें एक अध्यक्ष और पाांच सदस्य होते हैं।
• अध्यक्ष भारत का सेवानिवृत्त मख्ु य न्यायाधीश या सवोच्च न्यायालय का न्यायाधीश होिा चानहए और सदस्य सवोच्च
न्यायालय के सेवारत या सेवानिवृत्त न्यायाधीश, उच्च न्यायालय के सेवारत या सेवानिवृत्त मख्ु य न्यायाधीश और तीि व्यनक्त
होिे चानहए (नजिमें से कम से कम एक मनहला होिी चानहए) मािवानधकारों के सांबांध में ज्ञाि या व्यावहाररक अिभु व होिा
चानहए।
• इि पणू गकानलक सदस्यों के अलावा, आयोग में सात पदेि सदस्य भी होते हैं- राष्ट्रीय अपपसांख्यक आयोग के अध्यक्ष, राष्ट्रीय
अिुसनू चत जानत आयोग, राष्ट्रीय अिुसनू चत जिजानत आयोग, राष्ट्रीय मनहला आयोग, राष्ट्रीय नपछडा वगग आयोग, और
राष्ट्रीय बाल अनधकार सांरक्षण आयोग और नवकलाांग व्यनक्तयों के नलए मख्ु य आयक्त ु ।
• अध्यक्ष और सदस्यों की नियनु क्त राष्ट्रपनत द्वारा छह सदस्यीय सनमनत की नसफाररशों पर की जाती है नजसमें प्धाि मत्रां ी,
लोकसभा के अध्यक्ष, राज्यसभा के उपसभापनत,सांसद के दोिों सदिों में नवपक्ष के िेता और कें द्रीय गृह मत्रां ी शानमल होते हैं।
• इसके अलावा, सवोच्च न्यायालय के एक मौजदू ा न्यायाधीश या उच्च न्यायालय के एक मौजदू ा मख्ु य न्यायाधीश को भारत
के मख्ु य न्यायाधीश के परामशग के बाद ही नियक्त
ु नकया जा सकता है।
आयोग को उस तारीख से एक वषग की समानप्त के बाद नकसी भी मामले की जाचां करिे का अनधकार िहीं है, नजस नदि मािव अनधकारों
का उपलघां ि करिे वाला कायग नकया गया था। दसू रे शब्दों में, यह नकसी मामले को उसके घनटत होिे के एक वषग के भीतर जाचां कर
सकता है।

Q. 134) कें द्रीय सूचिा आयोग (CIC) के संबंध में निम्िनिनित में से कौि सा/से कथि सही है/हैं?
1. आयोग के पास उनचत आधार होिे पर कायागलयों, नवत्तीय सांस्थािों, सावगजनिक क्षेत्र के उपिमों से सांबनां धत नशकायतों और
अपीलों की जाचां करिे का अनधकार है।
2. मख्ु य सचू िा आयक्त
ु कें द्र सरकार द्वारा निधागररत अवनध के नलए या जब तक वे 65 वषग की आयु प्ाप्त िहीं कर लेत,े जो भी
पहले हो, पद धारण करते हैं।
3. राज्य सभा में नवपक्ष का िेता उस सनमनत का सदस्य होता है जो राष्ट्रपनत के मख्ु य सचू िा आयक्त
ु के रूप में नकसी व्यनक्त की
नियनु क्त की नसफाररश करती है।

www.iasbaba.com rrs.iasbaba.com Page 119


IASbaba’s Rapid Revision (RaRe) Series Polity Day 1-120 2021
HINDI

उपरोक्त में से कौि सा कथि सही हैं?


a) के वल 1 और 2
b) के वल 2 और 3
c) के वल 1 और 3
d) 1, 2 और 3
Q. 134) Solution (a)
कें द्रीय सचू िा आयोग की स्थापिा 2005 में कें द्र सरकार द्वारा की गई थी। इसका गठि सचू िा का अनधकार अनधनियम (2005) के
प्ावधािों के तहत एक आनधकाररक राजपत्र अनधसचू िा के माध्यम से नकया गया था।
कें द्रीय सचू िा आयोग एक उच्च अनधकार प्ाप्त स्वतत्रां निकाय है जो अन्य बातों के साथ-साथ की गई नशकायतों को देखता है और
अपीलों का निणगय करता है। यह कें द्र सरकार और कें द्र शानसत प्देशों के तहत कायागलयों, नवत्तीय सांस्थािों, सावगजनिक क्षेत्र के उपिमों
आनद से सांबांनधत नशकायतों और अपीलों पर नवचार करता है। उनचत आधार होिे पर आयोग नकसी भी मामले में जाचां का आदेश दे
सकता है (स्व-प्ेरणा शनक्त)।
आयोग में एक मख्ु य सचू िा आयक्त ु होता है और दस से अनधक सचू िा आयक्त
ु िहीं होते हैं। आयोग, जब शरू
ु में गनठत नकया गया था,
में मख्ु य सचू िा आयक्त
ु सनहत पाचां आयक्त
ु थे।
उन्हें राष्ट्रपनत द्वारा एक सनमनत की नसफाररश पर नियक्त ु नकया जाता है नजसमें अध्यक्ष के रूप में प्धाि मत्रां ी, लोकसभा में नवपक्ष के िेता
और प्धाि मांत्री द्वारा िानमत कें द्रीय कै नबिेट मत्रां ी शानमल होते हैं।
मख्ु य सचू िा आयक्त
ु और एक सचू िा आयक्त ु कें द्र सरकार द्वारा निधागररत अवनध के नलए या जब तक वे 65 वषग की आयु प्ाप्त िहीं कर
लेत,े जो भी पहले हो, पद धारण करें गे। वे पिु निगयनु क्त के पात्र िहीं हैं।

Q. 135) कें द्रीय सतका ता आयोग (CVC) के सदं भा में निम्िनिनित कथिों पर नवचार कीनजएः
1. यह ि तो सांवैधानिक निकाय है और ि ही साांनवनधक निकाय है।
2. यह उि नशकायतों को देखता है नजन्हें भ्रष्टाचार निवारण अनधनियम, 1988 के तहत अपराध घोनषत नकया गया है।
3. कें द्रीय सतकग ता आयक्त
ु पाांच वषग की अवनध के नलए या पैंसठ वषग की आयु प्ाप्त करिे तक, जो भी पहले हो, तक पद धारण
करता है।
उपरोक्त में से कौि सा कथि सही हैं?
a) के वल 1 और 2
b) के वल 2
c) के वल 3

www.iasbaba.com rrs.iasbaba.com Page 120


IASbaba’s Rapid Revision (RaRe) Series Polity Day 1-120 2021
HINDI

d) के वल 1 और 3
Q. 135) Solution (b)
कें द्रीय सतकग ता आयोग (CVC) कें द्र सरकार में भ्रष्टाचार को रोकिे के नलए मख्ु य एजेंसी है। इसकी स्थापिा 1964 में कें द्र सरकार के
एक कायगकारी प्स्ताव द्वारा की गई थी।
इसकी स्थापिा की नसफाररश भ्रष्टाचार निवारण पर सथां ािम सनमनत द्वारा की गई थी।
मल
ू रूप से सीवीसी ि तो एक सांवैधानिक निकाय था और ि ही एक साांनवनधक निकाय। बाद में, 2003 में, सांसद िे कें द्रीय सतकग ता
आयोग को वैधानिक दजाग प्दाि करिे वाला एक अनधनियम पाररत नकया।
कें द्रीय सतकग ता आयोग (CVC) का कायग कें द्र सरकार द्वारा नकए गए एक सदां भग पर पछू ताछ या जाचां या निरीक्षण का सचां ालि करिा
है नजसमें यह आरोप लगाया गया है नक एक लोक सेवक कें द्र सरकार या उसके अनधकाररयों का कमगचारी है। भ्रष्टाचार निवारण अनधनियम,
1988 के तहत अपराध नकया गया है।
कें द्रीय सतकग ता आयोग एक बहु-सदस्यीय निकाय है नजसमें एक कें द्रीय सतकग ता आयक्त
ु (अध्यक्ष) होता है और दो से अनधक सतकग ता
आयक्त ु िहीं होते हैं।
उन्हें राष्ट्रपनत द्वारा अपिे हाथ के तहत वारांट द्वारा नियक्त
ु नकया जाता है और तीि सदस्यीय सनमनत की नसफाररश पर प्धाि मांत्री, कें द्रीय
गृह मांत्री और लोकसभा में नवपक्ष के िेता के रूप में प्धाि मांत्री की नसफाररश पर महु र लगाई जाती है।
वे चार साल की अवनध के नलए या पैंसठ वषग की आयु प्ाप्त करिे तक, जो भी पहले हो, तक पद धारण करते हैं। अपिे कायगकाल के
बाद, वे कें द्र या राज्य सरकार के तहत आगामी नियनु क्त के नलए पात्र िहीं हैं।

Q. 136) कें द्रीय जांच ब्यूरो (CBI) के संबध


ं में निम्िनिनित में से कौि सा/से कथि सही है/हैं?
1. यह गृह मत्रां ालय के तहत काम करता है।
2. सीबीआई भ्रष्टाचार के अपराध, आनथगक अपराध, आतक
ां वाद और गभां ीर और सगां नठत अपराध की जाचां करती है।
3. यह नदपली नवशेष पनु लस स्थापिा अनधनियम, 1946 से अपिी शनक्तयााँ प्ाप्त करता है।
िीचे नदए गए कूटों में से सही उत्तर चुनिए:
a) के वल 1 और 2
b) के वल 2
c) के वल 3
d) 1, 2 और 3
Q. 136) Solution (c)

www.iasbaba.com rrs.iasbaba.com Page 121


IASbaba’s Rapid Revision (RaRe) Series Polity Day 1-120 2021
HINDI

कें द्रीय जाचां ब्यरू ो (CBI) की स्थापिा 1963 में गृह मत्रां ालय के एक प्स्ताव द्वारा की गई थी। बाद में, इसे कानमगक मांत्रालय में स्थािातां ररत
कर नदया गया और अब इसे एक सल ां ग्ि कायागलय का दजाग प्ाप्त है।
भ्रष्टाचार की रोकथाम पर सांथािम सनमनत (1962-1964) द्वारा सीबीआई की स्थापिा की नसफाररश की गई थी।
सीबीआई एक सानां वनधक निकाय िहीं है। यह नदपली नवशेष पनु लस स्थापिा अनधनियम, 1946 से अपिी शनक्तयााँ प्ाप्त करता है।
सीबीआई कें द्र सरकार की प्मख ु जाांच एजेंसी है। यह भ्रष्टाचार की रोकथाम और प्शासि में सत्यनिष्ठा बिाए रखिे में महत्वपणू ग भनू मका
निभाता है। यह कें द्रीय सतकग ता आयोग और लोकपाल को भी सहायता प्दाि करता है।
सीबीआई आतक
ां वाद के अलावा भ्रष्टाचार, आनथगक अपराध और गभां ीर और सगां नठत अपराध के अपराध की जाचां करती है।
एिआईए (NIA) का गठि 2008 में मांबु ई आतांकवादी हमले के बाद मख्ु य रूप से आतांकवादी हमलों की घटिाओ,ां आतांकवाद के
नवत्तपोषण और अन्य आतांकवाद से सांबांनधत अपराधों की जाचां के नलए नकया गया है।

Q. 137) िोकपाि और िोकायुक्त अनधनियम, 2013 के संदभा में, निम्िनिनित में से कौि सा/से कथि सही है/हैं?
1. लोकपाल के अनधकार क्षेत्र में भारत के प्धाि मत्रां ी शानमल हैं।
2. लोकपाल के पास नकसी भी लोक सेवक के नखलाफ कारग वाई करिे की स्वत: सज्ञां ाि की शनक्त है।
3. सरकार द्वारा सहायता प्ाप्त सांस्थािों को लोकपाल के दायरे से बाहर रखा गया है।
िीचे नदए गए कूटों में से सही उत्तर चुनिए:
a) के वल 1 और 2
b) के वल 2
c) के वल 3
d) के वल 1 और 3
Q. 137) Solution (d)
लोकपाल और लोकायक्त
ु अनधनियम, 2013 की मख्ु य नवशेषताएां इस प्कार हैं:
• यह कें द्र में लोकपाल की सांस्था और राज्य के स्तर पर लोकायक्त ु की स्थापिा करिा चाहता है और इस प्कार कें द्र और राज्यों
दोिों में राष्ट्र के नलए एक समाि सतकग ता और भ्रष्टाचार नवरोधी रोड मैप प्दाि करिा चाहता है। लोकपाल के अनधकार क्षेत्र
में प्धाि मांत्री, मांत्री, सांसद सदस्य और समहू ए, बी, सी और डी के अनधकारी और कें द्र सरकार के अनधकारी शानमल हैं।
• लोकपाल में एक अध्यक्ष होगा नजसमें अनधकतम 8 सदस्य होंगे नजिमें से 50% न्यानयक सदस्य होंगे।
• लोकपाल के 50% सदस्य एससी, एसटी, ओबीसी, अपपसांख्यक और मनहलाओ ां में से होंगे।

www.iasbaba.com rrs.iasbaba.com Page 122


IASbaba’s Rapid Revision (RaRe) Series Polity Day 1-120 2021
HINDI

• इसमें भ्रष्ट तरीकों से अनजगत लोक सेवकों की सांपनत्त की कुकी और जब्ती के प्ावधाि शानमल हैं, भले ही अनभयोजि लनां बत
हो।
• यह स्पष्ट समय-सीमा निधागररत करता है। प्ारांनभक जाचां के नलए, यह तीि महीिे के नलए तीि महीिे के नलए बढ़ाया जा
सकता है। जाचां के नलए यह छह महीिे का होता है नजसे एक बार में छह महीिे के नलए बढ़ाया जा सकता है। परीक्षण के
नलए, यह एक वषग है नजसे एक वषग तक बढ़ाया जा सकता है और इसे प्ाप्त करिे के नलए, नवशेष अदालतें स्थानपत की जािी
हैं।
• सरकार द्वारा पणू ग या आनां शक रूप से नवत्तपोनषत सस्ां थाि लोकपाल के अनधकार क्षेत्र में हैं, लेनकि सरकार द्वारा सहायता प्ाप्त
सस्ां थािों को बाहर रखा गया है।
• यह ईमािदार और न्यायसगां त लोक सेवकों को पयागप्त सरु क्षा प्दाि करता है।
• लोकपाल को सरकार या सक्षम प्ानधकारी के स्थाि पर लोक सेवकों के नखलाफ मक
ु दमा चलािे की मजां रू ी देिे की शनक्त
प्दाि की गई।
लोकपाल और लोकायक्त
ु अनधनियम, 2013 की कनमयाां निम्िनलनखत हैं:
• लोकपाल नकसी भी लोक सेवक के नखलाफ स्वप्ेरणा से कायगवाही िहीं कर सकता।
• सार के बजाय नशकायत के रूप पर जोर।
• लोक सेवकों के नखलाफ झठू ी और तुच्छ नशकायतों के नलए भारी सजा लोकपाल को नशकायत दजग करिे से रोक सकती है।
• बेिामी नशकायतों की अिुमनत िहीं है - के वल सादे कागज पर नशकायत िहीं कर सकते हैं और इसे सहायक दस्तावेजों के
साथ एक बॉक्स में छोड सकते हैं।
• लोक सेवक को कािूिी सहायता, नजसके नखलाफ नशकायत दजग की गई है।
• नशकायत दजग करिे के नलए 7 वषग की सीमा अवनध।
• पीएम के नखलाफ नशकायतों (non-transparent procedure) से निपटिे के नलए बहुत ही गैर-पारदशी प्निया

Q. 138) राष्ट्रीय आपदा प्रबध


ं ि प्रानधकरण (NDMA) के सदं भा में, निम्िनिनित कथिों पर नवचार करें:
1. प्धािमत्रां ी एिडीएमए (NDMA) के पदेि अध्यक्ष होते हैं
2. यह कें द्रीय पृ्वी नवज्ञाि मांत्रालय के प्शासनिक नियांत्रण में काम करता है।
3. यह आपदा से प्भानवत व्यनक्तयों को राहत के न्यिू तम मािक प्दाि करिे के नलए नदशा-निदेशों की नसफाररश करता है।
उपरोक्त में से कौि सा कथि सही हैं?
a) के वल 1 और 2
b) के वल 1 और 3

www.iasbaba.com rrs.iasbaba.com Page 123


IASbaba’s Rapid Revision (RaRe) Series Polity Day 1-120 2021
HINDI

c) के वल 2 और 3
d) 1, 2 और 3
Q. 138) Solution (b)
राष्ट्रीय आपदा प्बांधि प्ानधकरण (NDMA) देश में आपदा प्बांधि के नलए शीषग निकाय है। यह कें द्रीय गृह मांत्रालय के प्शासनिक
नियांत्रण में काम करता है।
एिडीएमए की स्थापिा इस दृनष्टकोण के साथ की गई थी: 'एक समग्र, सनिय, प्ौद्योनगकी सचां ानलत और सतत नवकास रणिीनत द्वारा
एक सरु नक्षत और आपदा प्नतरोधी भारत का निमागण करिा नजसमें सभी नहतधारक शानमल हों और रोकथाम, तैयारी और शमि की
सस्ां कृ नत को बढ़ावा दें।
एिडीएमए में एक अध्यक्ष और अन्य सदस्य होते हैं, जो िौ से अनधक िहीं होते हैं। प्धािमांत्री एिडीएमए के पदेि अध्यक्ष होते हैं। अन्य
सदस्यों को एिडीएमए के अध्यक्ष द्वारा िानमत नकया जाता है। एिडीएमए के अध्यक्ष सदस्यों में से एक को एिडीएमए के उपाध्यक्ष के
रूप में िानमत करते हैं। उपाध्यक्ष को कै नबिेट मांत्री का दजाग प्ाप्त होता है जबनक अन्य सदस्यों को राज्य मांत्री का दजाग प्ाप्त होता है।
एिडीएमए निम्िनलनखत कायग करता है:
• यह आपदा से प्भानवत व्यनक्तयों को राहत के न्यिू तम मािक प्दाि करिे के नलए नदशा-निदेशों की नसफाररश करता है।
• यह अिुशांसा करता है, गभां ीर पररमाण की आपदाओ ां के मामलों में, ऋणों की अदायगी में राहत या ऐसी आपदाओ ां से प्भानवत
व्यनक्तयों को ररयायती शतों पर िए ऋण प्दाि करिा।
• यह राष्ट्रीय आपदा प्नतनिया बल (NDMA) के सामान्य अधीक्षण, निदेशि और नियांत्रण का प्योग करता है।
• इस बल का गठि नकसी खतरिाक आपदा की नस्थनत या आपदा के नलए नवशेषज्ञ प्नतनिया के उद्देश्य से नकया गया है।
• यह सांबांनधत नवभाग या प्ानधकरण को नकसी भी खतरिाक आपदा की नस्थनत या आपदा में बचाव या राहत के नलए प्ावधािों
या सामनग्रयों की आपातकालीि खरीद करिे के नलए अनधकृ त करता है। ऐसे मामले में, निनवदाओ ां को आमनां त्रत करिे की
आवश्यकता वाली मािक प्निया को माफ कर नदया गया मािा जाता है।
• यह अपिी गनतनवनधयों पर एक वानषगक ररपोटग तैयार करता है और इसे कें द्र सरकार को प्स्ततु करता है। कें द्र सरकार इसे ससां द
के दोिों सदिों के सामिे रखवाती है।

Q. 139) निम्िनिनित कथिों पर नवचार करें:


1. अिुच्छे द 323 ए के तहत नरब्यिू ल/ न्यायानधकरण सांसद और राज्य नवधािसभाओ ां द्वारा उिकी नवधायी क्षमता के भीतर आिे
वाले मामलों के सांबांध में स्थानपत नकए जा सकते हैं।
2. अिुच्छे द 323 बी के वल लोक सेवा मामलों के नलए न्यायानधकरणों की स्थापिा का प्ावधाि करता है।
उपरोक्त में से कौि सा कथि सही हैं?
a) के वल 1

www.iasbaba.com rrs.iasbaba.com Page 124


IASbaba’s Rapid Revision (RaRe) Series Polity Day 1-120 2021
HINDI

b) के वल 2
c) दोिों 1 और 2
d) ि तो 1 और ि ही 2
Q. 139) Solution (d)
मल
ू सांनवधाि में न्यायानधकरणों के सांबांध में प्ावधाि िहीं थे। 1976 के 42वें सांशोधि अनधनियम िे सांनवधाि में एक िया भाग XIV-A
जोडा।
यह भाग 'अनधकरण' के रूप में है और इसमें के वल दो अिच्ु छे द शानमल हैं-अिच्ु छे द 323ए प्शासनिक नरब्यिू ल से सबां नां धत है और
अिच्ु छे द 323 बी अन्य मामलों के नलए नरब्यिू ल से सबां नां धत है।
अिुच्छे द 323 ए और 323 बी निम्िनलनखत तीि पहलओ
ु ां में नभन्ि हैं:
• जबनक अिच्ु छे द 323 ए के वल सावगजनिक सेवा मामलों के नलए अनधकरण की स्थापिा पर नवचार करता है, अिच्ु छे द 323
बी कुछ अन्य मामलों के नलए अनधकरण की स्थापिा पर नवचार करता है।
• जबनक अिुच्छे द 323 ए के तहत अनधकरण के वल सांसद द्वारा स्थानपत नकए जा सकते हैं, अिुच्छे द 323 बी के तहत
अनधकरण सांसद और राज्य नवधािसभाओ ां दोिों द्वारा उिकी नवधायी क्षमता के भीतर आिे वाले मामलों के सांबांध में स्थानपत
नकए जा सकते हैं।
• अिुच्छे द 323 ए के तहत, कें द्र के नलए के वल एक और प्त्येक राज्य या दो या अनधक राज्यों के नलए एक अनधकरण स्थानपत
नकया जा सकता है। अनधकरण के पदािुिम का कोई सवाल ही िहीं है, जबनक अिुच्छे द 323 बी के तहत अनधकरण का
एक पदािुिम बिाया जा सकता है।

Q.140) सहकारी संघवाद (cooperative federalism) के िक्ष्य को साकार करिे और भारत में सुशासि को सक्षम करिे
के निए निम्िनिनित में से नकस संस्थाि का गठि नकया गया है?
a) लोकपाल और लोकायक्त

b) चिु ाव आयोग
c) कें द्रीय सतकग ता आयोग
d) िीनत आयोग
Q. 140) Solution (d)
िीनत आयोग का गठि सहकारी सघां वाद के महत्वपणू ग लक्ष्य को साकार करिे और भारत में सश ु ासि को सक्षम करिे के नलए, एक मजबूत
राष्ट्र बिािे वाले मजबूत राज्यों के निमागण के नलए नकया गया है। वास्तव में सांघीय राज्य में, कई उद्देश्यों को प्ाप्त नकया जािा चानहए जो
परू े देश में राजिीनतक प्भाव डाल सकते हैं। नकसी भी सांघीय सरकार के नलए राज्य सरकारों के सनिय सहयोग के नबिा राष्ट्रीय उद्देश्यों
को प्ाप्त करिा असांभव है। इसनलए, यह महत्वपणू ग है नक कें द्र और राज्य सरकारें नमलकर काम करें ।

www.iasbaba.com rrs.iasbaba.com Page 125


IASbaba’s Rapid Revision (RaRe) Series Polity Day 1-120 2021
HINDI

सहकारी सांघवाद की दो प्मख


ु नवशेषताएां या पहलू हैं:
• कें द्र और राज्यों द्वारा राष्ट्रीय नवकास एजेंडा पर सांयक्त
ु रूप से ध्याि कें नद्रत करिा; तथा
• कें द्रीय मांत्रालयों के साथ राज्य के दृनष्टकोण की वकालत।
इसे ध्याि में रखते हुए, िीनत आयोग को राज्यों की सनिय भागीदारी के साथ राष्ट्रीय नवकास प्ाथनमकताओ,ां क्षेत्रों और रणिीनतयों की
एक साझा दृनष्ट नवकनसत करिे का कायग सौंपा गया है। इि प्ाथनमकताओ ां को राष्ट्रीय उद्देश्यों को प्नतनबनां बत करिा चानहए और राज्यों
को निरांतर आधार पर सांरनचत समथगि के माध्यम से सहकारी सांघवाद को बढ़ावा देिा चानहए। िीनत आयोग को राज्यों को ग्राम स्तर पर
नवश्वसिीय योजिाएाँ तैयार करिे के नलए तत्रां नवकनसत करिे में मदद करिी चानहए और सरकार के उच्च स्तरों पर इन्हें उत्तरोत्तर एकनत्रत
करिा चानहए। इसका उद्देश्य उस चरण से प्गनत करिा है जब कें द्र िे वास्तव में सांघीय सरकार के नलए नवकास िीनतयों का फै सला नकया,
नजसमें राज्य योजिा प्निया में समाि नहतधारक हैं।
Q. 141) निम्िनिनित कथिों पर नवचार करें:
1. भारत के सांनवधाि का भाग XX कुछ राज्यों के नलए अस्थायी, सांिमणकालीि और नवशेष प्ावधाि प्दाि करता है।
2. अिुच्छे द 371-ए के अिुसार, सांसद राज्य की नवधाि सभा की सहमनत के नबिा असम में भनू म और उसके सांसाधिों के
स्वानमत्व और हस्तातां रण में हस्तक्षेप िहीं कर सकती है।
उपरोक्त में से कौि सा कथि सही हैं?
a) के वल 1
b) के वल 2
c) दोिों 1 और 2
d) ि तो 1 और ि ही 2
Q. 141) Solution (d)
भारत के सनां वधाि का भाग XXI एक देश और राज्यों के सघां के रूप में भारत के सनां वधाि से सबां नां धत काििू ों का सक
ां लि है नजससे
इसका गठि हुआ है। सनां वधाि के इस भाग में कुछ राज्यों के नलए अस्थायी, सि ां मणकालीि और नवशेष प्ावधािों पर अिच्ु छे द शानमल
हैं। भाग XXI भारतीय सघां के अन्य राज्यों के सबां धां में नवशेष प्ावधािों को पररभानषत करता है।
भारतीय सांनवधाि का अिुच्छे द 371-ए िागालैंड राज्य के नलए नवशेष प्ावधाि करता है।
निम्िनलनखत मामलों से सबां नां धत ससां द के अनधनियम िागालैंड पर तब तक लागू िहीं होंगे जब तक नक राज्य नवधाि सभा ऐसा निणगय
िहीं लेती:
• िागाओ ां की धानमगक या सामानजक प्थाएां;
• िागा प्थागत कािूि और प्निया;
• िागा प्थागत कािूि के अिुसार निणगयों को शानमल करते हुए दीवािी और आपरानधक न्याय का प्शासि; तथा

www.iasbaba.com rrs.iasbaba.com Page 126


IASbaba’s Rapid Revision (RaRe) Series Polity Day 1-120 2021
HINDI

• भनू म और उसके सांसाधिों का स्वानमत्व और हस्तातां रण।


राज्यपाल को यह सनु िनित करिा होता है नक कें द्र सरकार द्वारा नकसी नवनशष्ट उद्देश्य के नलए प्दाि नकया गया धि उस उद्देश्य से सांबांनधत
अिुदाि की मागां में शानमल है, ि नक राज्य नवधािसभा में पेश की गई नकसी अन्य माांग में।
अिुच्छे द में कहा गया है नक जब तक शत्रतु ापणू ग िागाओ ां के कारण आांतररक अशाांनत जारी रहेगी, तब तक िागालैंड के राज्यपाल के
पास राज्य में कािूि और व्यवस्था के नलए नवशेष नजम्मेदारी होगी। इस नजम्मेदारी के निवगहि में राज्यपाल, मांनत्रपररषद से परामशग करिे
के बाद, अपिे व्यनक्तगत निणगय का प्योग करता है और उसका निणगय अांनतम होता है।

Q. 142) भारतीय सनं वधाि का अिच्ु छे द 371-एफ नकस राज्य के सबं ध


ं में नवशेष प्रावधाि प्रदाि करता है:
a) आांध्र प्देश
b) तेलांगािा
c) नसनक्कम
d) मनणपरु
Q. 142) Solution (c)
1975 के 36वें सांनवधाि सांशोधि अनधनियम िे नसनक्कम को भारतीय सांघ का एक पणू ग राज्य बिा नदया। इसमें नसनक्कम के सांबांध में
नवशेष प्ावधािों वाला एक िया अिुच्छे द 371-एफ शानमल था। ये इस प्कार हैं:
1. नसनक्कम नवधाि सभा में कम से कम 30 सदस्य होिे चानहए।
2. लोकसभा में नसनक्कम को एक सीट आवांनटत की जाती है और नसनक्कम एक सांसदीय निवागचि क्षेत्र बिाता है।
3. नसनक्कम की आबादी के नवनभन्ि वगों के अनधकारों और नहतों की रक्षा के उद्देश्य से, ससां द को निम्िनलनखत के नलए प्ावधाि
करिे का अनधकार है:
1. नसनक्कम नवधाि सभा में सीटों की सख्ां या जो ऐसे वगों से सबां नां धत उम्मीदवारों द्वारा भरी जा सकती है; तथा
2. नवधािसभा निवागचि क्षेत्रों का पररसीमि जहाां से ऐसे वगों के उम्मीदवार अके ले नवधािसभा के चिु ाव के नलए खडे
हो सकते हैं।
4. नसनक्कम की आबादी के नवनभन्ि वगों की सामानजक और आनथगक उन्िनत सनु िनित करिे के नलए शानां त और एक समाि
व्यवस्था के नलए राज्यपाल की नवशेष नजम्मेदारी होगी। इस उत्तरदानयत्व के निवगहि में राज्यपाल अपिे नववेक से राष्ट्रपनत
द्वारा जारी निदेशों के अधीि कायग करे गा।
5. राष्ट्रपनत नसनक्कम में भारतीय सांघ के राज्य में लागू नकसी भी कािूि को (प्नतबांधों या सांशोधिों के साथ) बढ़ा सकते हैं।

Q. 143) निम्िनिनित कथिों पर नवचार करें:


1. अिुच्छे द 371-जी में प्ावधाि है नक नमजोरम नवधाि सभा में कम से कम 20 सदस्य होिे चानहए।
2. अिुच्छे द 371-एच में प्ावधाि है नक अरुणाचल प्देश नवधािसभा में कम से कम 30 सदस्य होिे चानहए।

www.iasbaba.com rrs.iasbaba.com Page 127


IASbaba’s Rapid Revision (RaRe) Series Polity Day 1-120 2021
HINDI

उपरोक्त में से कौि सा कथि सही हैं?


a) के वल 1
b) के वल 2
c) दोिों 1 और 2
d) ि तो 1 और ि ही 2
Q. 143) Solution (b)
अिच्ु छे द 371-जी नमजोरम के नलए निम्िनलनखत नवशेष प्ावधािों को निनदगष्ट करता है:
1. निम्िनलनखत मामलों से सांबांनधत ससां द के अनधनियम नमजोरम पर तब तक लागू िहीं होंगे जब तक नक राज्य नवधाि सभा ऐसा
निणगय ि ले:
1. नमजो की धानमगक या सामानजक प्थाएां;
2. नमजो प्थागत कािूि और प्निया;
3. दीवािी और फौजदारी न्याय का प्शासि नजसमें नमजो प्थागत कािूि के अिुसार निणगय शानमल हैं; और (iv)
भनू म का स्वानमत्व और हस्ताांतरण।
2. नमजोरम नवधाि सभा में कम से कम 40 सदस्य होिे चानहए।
अिुच्छे द 371-एच के तहत अरुणाचल प्देश के नलए निम्िनलनखत नवशेष प्ावधाि नकए गए हैं:
1. अरुणाचल प्देश के राज्यपाल के पास राज्य में काििू और व्यवस्था के नलए नवशेष नजम्मेदारी होगी। इस नजम्मेदारी के निवगहि
में राज्यपाल, मांनत्रपररषद से परामशग करिे के बाद, अपिे व्यनक्तगत निणगय का प्योग करता है और उसका निणगय अांनतम होता
है। राष्ट्रपनत के ऐसा निदेश देिे पर राज्यपाल का यह नवशेष उत्तरदानयत्व समाप्त हो जाएगा।
2. अरुणाचल प्देश नवधाि सभा में कम से कम 30 सदस्य होिे चानहए।

Q. 144) निम्िनिनित कथिों पर नवचार करें:


1. भारतीय सनां वधाि का भाग VIII कें द्र शानसत प्देशों से सबां नां धत है।
2. एक कें द्र शानसत प्देश का प्शासक राष्ट्रपनत का एजेंट होता है, ि नक राज्य का मनु खया।
3. ससां द सभी कें द्र शानसत प्देशों के नलए राज्य सचू ी सनहत तीि सनू चयों में से नकसी भी नवषय पर काििू बिा सकती है।
उपरोक्त में से कौि सा कथि सही हैं?
a) के वल 1 और 2
b) के वल 2 और 3
c) के वल 1 और 3

www.iasbaba.com rrs.iasbaba.com Page 128


IASbaba’s Rapid Revision (RaRe) Series Polity Day 1-120 2021
HINDI

d) 1, 2 और 3
Q. 144) Solution (d)
सनां वधाि के भाग आठ में अिुच्छे द 239 से 241 कें द्र शानसत प्देशों से सांबांनधत है। भले ही सभी कें द्र शानसत प्देश एक श्रेणी के हैं,
लेनकि उिकी प्शासनिक व्यवस्था में एकरूपता िहीं है।
प्त्येक कें द्र शानसत प्देश का प्शासि राष्ट्रपनत द्वारा उसके द्वारा नियक्त
ु प्शासक के माध्यम से कायग करता है। एक कें द्र शानसत प्देश
का प्शासक राष्ट्रपनत का एजेंट होता है ि नक राज्यपाल की तरह राज्य का मनु खया।
राष्ट्रपनत एक प्शासक के पद को निनदगष्ट कर सकता है; यह उपराज्यपाल या मख्ु य आयक्त ु या प्शासक हो सकता है। वतगमाि में, यह
नदपली, पडु ु चेरी, अांडमाि और निकोबार द्वीप समहू , जम्मू और कश्मीर और लद्दाख के मामले में उपराज्यपाल और चांडीगढ़, दादरा और
िगर हवेली, दमि और दीव और लक्षद्वीप के मामले में प्शासक है।
राष्ट्रपनत नकसी राज्य के राज्यपाल को निकटवती कें द्र शानसत प्देश का प्शासक भी नियक्त
ु कर सकता है। उस क्षमता में, राज्यपाल
को अपिे मांनत्रपररषद से स्वतांत्र रूप से कायग करिा होता है।
कें द्र शानसत प्देशों के नलए सांसद तीि सनू चयों (राज्य सचू ी सनहत) के नकसी भी नवषय पर कािूि बिा सकती है। सांसद की यह शनक्त
पडु ु चरे ी, नदपली और जम्मू और कश्मीर तक भी फै ली हुई है, नजिकी अपिी स्थािीय नवधानयकाएाँ हैं। इसका अथग यह है नक राज्य सचू ी
के नवषयों पर कें द्र शानसत प्देशों के नलए सांसद की नवधायी शनक्त उिके नलए स्थािीय नवधानयका स्थानपत करिे के बाद भी अप्भानवत
रहती है।

Q. 145) निम्िनिनित कथिों पर नवचार करें:


1. जम्मू और कश्मीर की नवधाि सभा सावगजनिक व्यवस्था, पनु लस और भनू म को छोडकर राज्य सचू ी के नकसी भी नवषय पर
काििू बिा सकती है।
2. राष्ट्रपनत कें द्र शानसत प्देश पडु ु चेरी की शानां त, प्गनत और अच्छी सरकार के नलए नियम तभी बिा सकते हैं जब नवधािसभा
निलनां बत या नवघनटत हो।
उपरोक्त में से कौि सा कथि सही हैं?
a) के वल 1
b) के वल 2
c) दोिों 1 और 2
d) ि तो 1 और ि ही 2
Q. 145) Solution (b)
जम्मू और कश्मीर की नवधाि सभा राज्य सचू ी (सावगजनिक व्यवस्था और पनु लस को छोडकर) और समवती सचू ी के नकसी भी नवषय
पर काििू बिा सकती है।

www.iasbaba.com rrs.iasbaba.com Page 129


IASbaba’s Rapid Revision (RaRe) Series Polity Day 1-120 2021
HINDI

राष्ट्रपनत कें द्र शानसत प्देश पडु ु चरे ी की शाांनत, प्गनत और अच्छी सरकार के नलए नियम तभी बिा सकते हैं जब नवधािसभा निलांनबत
या नवघनटत हो।
राष्ट्रपनत द्वारा बिाए गए नवनियम में सांसद के अनधनियम के समाि ही बल और प्भाव होता है और इि कें द्र शानसत प्देशों के सांबांध में
सांसद के नकसी भी अनधनियम को निरस्त या सांशोनधत भी कर सकता है।

Q. 146) निम्िनिनित कथिों पर नवचार करें:


1. कें द्र शानसत प्देश नदपली और पडु ु चेरी का अपिा उच्च न्यायालय है।
2. अांडमाि और निकोबार द्वीप समहू को मद्रास उच्च न्यायालय के अनधकार क्षेत्र में रखा गया है।
उपरोक्त में से कौि सा कथि सही हैं?
a) के वल 1
b) के वल 2
c) दोिों 1 और 2
d) ि तो 1 और ि ही 2
Q. 146) Solution (d)
ससां द एक कें द्र शानसत प्देश के नलए एक उच्च न्यायालय स्थानपत कर सकती है या इसे आसन्ि राज्य के उच्च न्यायालय के अनधकार
क्षेत्र में रख सकती है।
नदपली का अपिा एक उच्च न्यायालय है (1966 से)। बॉम्बे हाईकोटग को कें द्र शानसत प्देश दादरा और िगर हवेली और दमि और
दीव पर अनधकार क्षेत्र नमला है।
अडां माि और िोकोबार द्वीप समहू , चडां ीगढ़, लक्षद्वीप और पडु ु चेरी को िमशः कोलकाता, पजां ाब और हररयाणा, के रल और मद्रास
उच्च न्यायालयों के अधीि रखा गया है।
जम्मू और कश्मीर उच्च न्यायालय दो कें द्र शानसत प्देशों जम्मू और कश्मीर और लद्दाख के नलए सामान्य उच्च न्यायालय है।
सश ां ोधि अनधनियम के माध्यम से लाए गए अिच्ु छे द 230 के अिसु ार, ससां द काििू द्वारा नकसी भी कें द्र शानसत प्देश से उच्च न्यायालय
के अनधकार क्षेत्र को बढ़ा सकती है, या उच्च न्यायालय के अनधकार क्षेत्र को बाहर कर सकती है।
कें द्र शानसत प्देशों के सांबांध में इस शनक्त को सांनवधाि की सातवीं अिुसचू ी के तहत सांघ सचू ी में प्नवनष्ट 79 के रूप में शानमल करके
कें द्र के नलए अिन्य बिा नदया गया था। दसू री ओर, सांशोधि, अिुच्छे द 241 के माध्यम से, सांसद को कें द्र शानसत प्देश के नलए एक
उच्च न्यायालय का गठि करिे या ऐसे क्षेत्र में नकसी भी अदालत को उच्च न्यायालय घोनषत करिे की शनक्त प्दाि करता है। कें द्र शानसत
प्देश में स्थानपत इस तरह के उच्च न्यायालय की सांवैधानिक नस्थनत राज्य के उच्च न्यायालय के समाि होती है।

www.iasbaba.com rrs.iasbaba.com Page 130


IASbaba’s Rapid Revision (RaRe) Series Polity Day 1-120 2021
HINDI

Q. 147) निम्िनिनित में से कौि सा संवैधानिक प्रावधाि अिुसूनचत क्षेत्रों और जिजातीय क्षेत्रों के रूप में िानमत कुछ क्षेत्रों
के निए प्रशासि का प्रावधाि करता है?
1. भाग XI
2. भाग X
3. पाचां वी अिुसचू ी
िीचे नदए गए कूटों में से सही उत्तर चुनिए:
a) के वल 1 और 2
b) के वल 2 और 3
c) के वल 1 और 3
d) 1, 2 और 3
Q. 147) Solution (c)
सनां वधाि के भाग X में अिुच्छे द 244 में 'अिुसनू चत क्षेत्रों' और 'जिजातीय क्षेत्रों' के रूप में िानमत कुछ क्षेत्रों के नलए प्शासि की एक
नवशेष प्णाली की पररकपपिा की गई है।
सनां वधाि की पाचां वीं अिुसचू ी असम, मेघालय, नत्रपरु ा और नमजोरम के चार राज्यों को छोडकर नकसी भी राज्य में अिुसनू चत क्षेत्रों और
अिुसनू चत जिजानतयों के प्शासि और नियांत्रण से सांबनां धत है।
दसू री ओर, सांनवधाि की छठी अिुसचू ी, चार उत्तर-पवू ी राज्यों असम, मेघालय, नत्रपरु ा और नमजोरम में जिजातीय क्षेत्रों के प्शासि से
सबां नां धत है।

Q. 148) भारतीय संनवधाि की पांचवीं अिुसूची के संदभा में, निम्िनिनित कथिों पर नवचार कीनजएः
1. राष्ट्रपनत सबां नां धत राज्य के राज्यपाल के परामशग से नकसी क्षेत्र को अिसु नू चत क्षेत्र घोनषत कर सकता है।
2. सनां वधाि की पाचां वीं अिसु चू ी के अिसु ार अिसु नू चत क्षेत्रों वाले राज्यों में ही एक जिजानत सलाहकार पररषद की स्थापिा की
जा सकती है।
उपरोक्त में से कौि सा कथि सही हैं?
a) के वल 1
b) के वल 2
c) दोिों 1 और 2
d) ि तो 1 और ि ही 2

www.iasbaba.com rrs.iasbaba.com Page 131


IASbaba’s Rapid Revision (RaRe) Series Polity Day 1-120 2021
HINDI

Q. 148) Solution (a)


पााँचवीं अिुसचू ी में निनहत प्शासि की नवनभन्ि नवशेषताएां इस प्कार हैं:
• राष्ट्रपनत को नकसी क्षेत्र को अिुसनू चत क्षेत्र घोनषत करिे का अनधकार है। वह अपिे क्षेत्र को बढ़ा या घटा भी सकता है, उसकी
सीमा रे खाओ ां में पररवतगि कर सकता है, ऐसे पदिाम को रद्द कर सकता है या सांबांनधत राज्य के राज्यपाल के परामशग से नकसी
क्षेत्र पर इस तरह के पिु : पदिाम के नलए िए आदेश दे सकता है।
• अिसु नू चत क्षेत्रों वाले प्त्येक राज्य को अिसु नू चत जिजानतयों के कपयाण और उन्िनत पर सलाह देिे के नलए एक जिजानत
सलाहकार पररषद की स्थापिा करिी होती है। इसमें 20 सदस्य होते हैं, नजिमें से तीि-चौथाई राज्य नवधाि सभा में अिसु नू चत
जिजानतयों के प्नतनिनध होते हैं। इसी तरह की पररषद अिसु नू चत जिजानतयों वाले राज्य में भी स्थानपत की जा सकती है,
लेनकि अिसु नू चत क्षेत्रों में िहीं, यनद राष्ट्रपनत ऐसा निदेश देते हैं।
• नकसी राज्य की कायगकारी शनक्त उसके अिसु नू चत क्षेत्रों तक फै ली हुई है। लेनकि ऐसे क्षेत्रों के सबां धां में राज्यपाल की नवशेष
नजम्मेदारी होती है। उसे ऐसे क्षेत्रों के प्शासि के सांबांध में, वानषगक रूप से या जब भी राष्ट्रपनत की आवश्यकता हो, राष्ट्रपनत
को एक ररपोटग प्स्ततु करिी होती है। कें द्र की कायगकारी शनक्त ऐसे क्षेत्रों के प्शासि के सांबांध में राज्यों को निदेश देिे तक फै ली
हुई है।
• राज्यपाल को यह निदेश देिे का अनधकार है नक सांसद या राज्य नवधानयका का कोई नवशेष अनधनियम अिुसनू चत क्षेत्र पर
लागू ि हो या निनदगष्ट सांशोधिों और अपवादों के साथ लागू ि हो। वह जिजानत सलाहकार पररषद से परामशग करके नकसी
अिुसनू चत क्षेत्र की शानां त और अच्छी सरकार के नलए नियम भी बिा सकता है। इस तरह के नवनियम अिुसनू चत जिजानतयों
के सदस्यों द्वारा या उिके बीच भनू म के हस्ताांतरण को सीनमत या प्नतबांनधत कर सकते हैं, अिुसनू चत जिजानतयों के सदस्यों
को भनू म के आवांटि को नवनियनमत कर सकते हैं और अिुसनू चत जिजानतयों के सांबांध में धि-उधार के व्यवसाय को नवनियनमत
कर सकते हैं।

Q. 149) भारत के संनवधाि की छठी अिुसूची के संदभा में, निम्िनिनित कथिों पर नवचार कीनजएः
1. असम, मेघालय, नत्रपरु ा और नमजोरम राज्यों में जिजातीय क्षेत्र सबां नां धत राज्य के कायगपानलका प्ानधकार से बाहर हैं।
2. राष्ट्रपनत को असम, मेघालय, नत्रपरु ा और नमजोरम राज्यों में आनदवासी क्षेत्रों को स्वायत्त नजलों के रूप में गनठत करिे का
अनधकार है।
3. प्त्येक स्वायत्त नजले में एक नजला पररषद होती है नजसमें 30 सदस्य होते हैं, नजिमें से चार राज्यपाल द्वारा मिोिीत होते हैं
और शेष 26 वयस्क मतानधकार के आधार पर चिु े जाते हैं।
उपरोक्त में से कौि सा कथि सही हैं?
a) के वल 1
b) के वल 1 और 2
c) के वल 3
d) 2 और 3

www.iasbaba.com rrs.iasbaba.com Page 132


IASbaba’s Rapid Revision (RaRe) Series Polity Day 1-120 2021
HINDI

Q. 149) Solution (c)


छठी अिुसचू ी के तहत सांनवधाि में चार उत्तर-पवू ी राज्यों असम, मेघालय, नत्रपरु ा और नमजोरम में आनदवासी क्षेत्रों के प्शासि के नलए
नवशेष प्ावधाि हैं।
चार राज्यों असम, मेघालय, नत्रपरु ा और नमजोरम में आनदवासी क्षेत्रों को स्वायत्त नजलों के रूप में गनठत नकया गया है। लेनकि, वे
सबां नां धत राज्य के कायगकारी प्ानधकरण के बाहर िहीं आते हैं।
राज्यपाल को स्वायत्त नजलों को व्यवनस्थत और पिु गगनठत करिे का अनधकार है। इस प्कार, वह उिके क्षेत्रों को बढ़ा या घटा सकता है
या उिके िाम बदल सकता है या उिकी सीमाओ ां को पररभानषत कर सकता है और इसी तरह।
यनद एक स्वायत्त नजले में नवनभन्ि जिजानतयााँ हैं, तो राज्यपाल नजले को कई स्वायत्त क्षेत्रों में नवभानजत कर सकता है।
प्त्येक स्वायत्त नजले में एक नजला पररषद होती है नजसमें 30 सदस्य होते हैं, नजिमें से 4 राज्यपाल द्वारा मिोिीत होते हैं और शेष 26
वयस्क मतानधकार के आधार पर चिु े जाते हैं। निवागनचत सदस्य 5 साल की अवनध के नलए पद धारण करते हैं (जब तक नक पररषद
पहले भगां िहीं हो जाती) और मिोिीत सदस्य राज्यपाल के प्सादपयंत पद धारण करते हैं।

Q.150) निम्िनिनित कथिों पर नवचार करें:


1. 1991 के 68वें सांनवधाि सांशोधि अनधनियम िे कें द्र शानसत प्देश नदपली को एक नवशेष दजाग प्दाि नकया।
2. कें द्र शानसत प्देश नदपली में मांनत्रपररषद की सांख्या नवधािसभा की कुल सांख्या का बीस प्नतशत तय की जाती है।
उपरोक्त में से कौि सा कथि सही हैं?
a) के वल 1
b) के वल 2
c) दोिों 1 और 2
d) ि तो 1 और ि ही 2
Q. 150) Solution (d)
1991 के 69वें सनां वधाि सश ां ोधि अनधनियम िे कें द्र शानसत प्देश नदपली को एक नवशेष दजाग प्दाि नकया, और इसे राष्ट्रीय राजधािी
क्षेत्र नदपली में बदल नदया और नदपली के प्शासक को लेनफ्टिेंट (लेनफ्टिेंट) गविगर के रूप में िानमत नकया।
इसिे नदपली के नलए एक नवधाि सभा और एक मनां त्रपररषद का गठि नकया। पहले, नदपली में एक महािगर पररषद और एक कायगकारी
पररषद थी।
नवधािसभा की ताकत 70 सदस्यों पर तय की जाती है, जो सीधे लोगों द्वारा चिु े जाते हैं। चिु ाव भारत के चिु ाव आयोग द्वारा आयोनजत
नकए जाते हैं।

www.iasbaba.com rrs.iasbaba.com Page 133


IASbaba’s Rapid Revision (RaRe) Series Polity Day 1-120 2021
HINDI

नवधािसभा राज्य सचू ी और समवती सचू ी के सभी मामलों पर राज्य सचू ी के तीि मामलों, यािी सावगजनिक व्यवस्था, पनु लस और भनू म
को छोडकर कािूि बिा सकती है। लेनकि, सांसद के काििू नवधािसभा द्वारा बिाए गए कािूिों पर प्बल या अनभभावी होते हैं।
मांनत्रपररषद की सांख्या नवधािसभा की कुल सांख्या की दस प्नतशत तय की जाती है, यािी सात-एक मख्ु यमांत्री और छह अन्य मांत्री।
मख्ु यमत्रां ी की नियनु क्त राष्ट्रपनत द्वारा की जाती है (उपराज्यपाल द्वारा िहीं)। अन्य मनां त्रयों की नियनु क्त राष्ट्रपनत द्वारा मख्ु यमांत्री की सलाह
पर की जाती है। मत्रां ी राष्ट्रपनत के प्सादपयंत पद धारण करते हैं। मनां त्रपररषद सामनू हक रूप से सभा के प्नत उत्तरदायी होती है।
Q. 151) बिवंत राय मेहता सनमनत द्वारा पंचायती राज संस्थाओ ं के संबंध में निम्िनिनित में से कौि सी नसफाररशें दी गई
हैं/हैं?
1. ग्राम पचां ायत, पचां ायत सनमनत और नजला पररषद के प्नतनिनधयों का प्त्यक्ष चिु ाव।
2. पचां ायत सनमनत को एक कायगकारी निकाय होिा चानहए।
3. नजला पररषद को कायगकारी निकाय के बजाय सलाहकारी और पयगवक्ष
े ी निकाय होिा चानहए।
िीचे नदए गए कूटों में से सही उत्तर चुनिए:
a) के वल 1 और 2
b) के वल 2 और 3
c) के वल 1 और 3
d) 1, 2 और 3
Q. 151) Solution (b)
जिवरी 1957 में, भारत सरकार िे सामदु ानयक नवकास कायगिम (1952) और राष्ट्रीय नवस्तार सेवा (1953) के कामकाज की जाचां
करिे और उिके बेहतर कामकाज के उपायों का सझु ाव देिे के नलए एक सनमनत नियक्त ु की। इस सनमनत के अध्यक्ष बलवतां राय जी
मेहता थे। सनमनत िे िवांबर 1957 में अपिी ररपोटग प्स्ततु की और 'लोकताांनत्रक नवकें द्रीकरण' की योजिा की स्थापिा की नसफाररश की,
नजसे अांततः पांचायती राज के रूप में जािा जािे लगा। इसके द्वारा की गई नवनशष्ट नसफाररशें हैं:
1. नत्रस्तरीय पचां ायती राज व्यवस्था की स्थापिा - ग्राम स्तर पर ग्राम पचां ायत, प्खडां स्तर पर पचां ायत सनमनत और नजला स्तर पर
नजला पररषद। इि स्तरों को अप्त्यक्ष चिु ावों के माध्यम से व्यवनस्थत रूप से जोडा जािा चानहए।
2. प्त्यक्ष निवागनचत प्नतनिनधयों के साथ ग्राम पचां ायत का गठि नकया जािा चानहए, जबनक अप्त्यक्ष रूप से निवागनचत सदस्यों
के साथ पचां ायत सनमनत और नजला पररषद का गठि नकया जािा चानहए।
3. सभी नियोजि और नवकास गनतनवनधयों को इि निकायों को सौंपा जािा चानहए।
4. पचां ायत सनमनत कायगकारी निकाय होिी चानहए जबनक नजला पररषद सलाहकार, समन्वय और पयगवेक्षी निकाय होिी चानहए।
5. नजला कलेक्टर को नजला पररषद का अध्यक्ष होिा चानहए।
6. इि लोकतानां त्रक निकायों को सत्ता और नजम्मेदारी का वास्तनवक हस्ताांतरण होिा चानहए।

www.iasbaba.com rrs.iasbaba.com Page 134


IASbaba’s Rapid Revision (RaRe) Series Polity Day 1-120 2021
HINDI

7. इि निकायों को अपिे कायों का निवगहि करिे और अपिी नजम्मेदाररयों को परू ा करिे में सक्षम बिािे के नलए पयागप्त सांसाधिों
को स्थािातां ररत नकया जािा चानहए।
8. भनवष्ट्य में प्ानधकरण के और हस्ताांतरण को प्भानवत करिे के नलए एक प्णाली नवकनसत की जािी चानहए।
सनमनत की इि नसफाररशों को राष्ट्रीय नवकास पररषद िे जिवरी 1958 में स्वीकार कर नलया। पररषद िे एक भी कठोर पैटिग या स्वरूप
पर जोर िहीं नदया और स्थािीय पररनस्थनतयों के अिक ु ू ल अपिे स्वयां के पैटिग नवकनसत करिे के नलए इसे राज्यों पर छोड नदया। लेनकि
बनु ियादी नसद्धातां और व्यापक बनु ियादी नसद्धातां परू े देश में एक जैसे होिे चानहए। राजस्थाि पचां ायती राज स्थानपत करिे वाला पहला
राज्य था।

Q. 152) निम्िनिनित कथिों पर नवचार करें:


1. भारतीय सनां वधाि का अिुच्छे द 243-ओ पचां ायतों के 29 कायागत्मक मदों से सांबांनधत है।
2. 73वें सांनवधाि सांशोधि अनधनियम, 1992 िे भारतीय सांनवधाि के अिुच्छे द 44 को व्यावहाररक रूप नदया है।
उपरोक्त में से कौि सा कथि सही हैं?
a) के वल 1
b) के वल 2
c) दोिों 1 और 2
d) ि तो 1 और ि ही 2
Q. 152) Solution (d)
73वें सांनवधाि सांशोधि अनधनियम, 1992 िे भारत के सांनवधाि में एक िया भाग-IX जोडा। यह भाग 'पांचायतों' के रूप में हकदार है
और इसमें अिुच्छे द 243 से 243 ओ के प्ावधाि शानमल हैं। इसके अलावा, अनधनियम िे सांनवधाि में एक िई ग्यारहवीं अिुसचू ी भी
जोडी है। इस अिुसचू ी में पांचायतों की 29 कायागत्मक मदें शानमल हैं। यह अिुच्छे द 243-जी से सांबांनधत है।
अनधनियम िे सनां वधाि के अिच्ु छे द 40 को एक व्यावहाररक आकार नदया है जो कहता है नक, "राज्य ग्राम पचां ायतों को सगां नठत करिे
के नलए कदम उठाएगा और उन्हें ऐसी शनक्तयाां और अनधकार प्दाि करे गा जो उन्हें स्वशासि की इकाइयों के रूप में कायग करिे में सक्षम
बिािे के नलए आवश्यक हो सकते हैं।"
यह लेख राज्य के िीनत निदेशक तत्वों का एक नहस्सा है। यह अनधनियम पचां ायती राज सांस्थाओ ां को सांवैधानिक दजाग प्दाि करता
है। इसिे उन्हें सांनवधाि के न्यायोनचत भाग के दायरे में ला नदया है।
दसू रे शब्दों में, राज्य सरकारें अनधनियम के प्ावधािों के अिुसार िई पांचायती राज प्णाली को अपिािे के नलए सांवैधानिक दानयत्व के
अधीि हैं। ितीजति, ि तो पचां ायतों का गठि और ि ही नियनमत अांतराल पर चिु ाव करािा राज्य सरकार की इच्छा पर निभगर करता है।

Q. 153) 73वें संनवधाि संशोधि अनधनियम, 1992 की नवशेषताओ ं के संदभा में निम्िनिनित कथिों पर नवचार कीनजएः

www.iasbaba.com rrs.iasbaba.com Page 135


IASbaba’s Rapid Revision (RaRe) Series Polity Day 1-120 2021
HINDI

1. यनद कोई व्यनक्त 25 वषग से कम आयु का है तो उसे पचां ायत का सदस्य होिे के नलए अयोग्य घोनषत कर नदया जाएगा।
2. ग्राम, मध्यवती और नजला स्तर पर पांचायतों के सभी सदस्य सीधे जिता द्वारा चिु े जाएांगे।
उपरोक्त में से कौि सा कथि सही हैं?
a) के वल 1
b) के वल 2
c) दोिों 1 और 2
d) ि तो 1 और ि ही 2
Q. 153) Solution (b)
73वें सनां वधाि सश
ां ोधि अनधनियम, 1992 के प्ावधािों के अिसु ार एक व्यनक्त को पचां ायत के सदस्य के रूप में चिु े जािे या सदस्य होिे
के नलए अयोग्य घोनषत नकया जाएगा यनद वह इस प्कार अयोग्य है:
a) सबां ांनधत राज्य की नवधानयका के चिु ाव के उद्देश्य से उस समय लागू नकसी भी कािूि के तहत, या
b) राज्य नवधानयका द्वारा बिाए गए नकसी भी कािूि के तहत।
तथानप, कोई भी व्यनक्त इस आधार पर अयोग्य िहीं होगा नक उसकी आयु 25 वषग से कम है यनद उसिे 21 वषग की आयु प्ाप्त कर ली
है। इसके अलावा, अयोग्यता के सभी प्श्नों को राज्य नवधािमांडल द्वारा निधागररत प्ानधकरण के पास भेजा जाएगा।
73वें सांनवधाि सांशोधि अनधनियम, 1992 के प्ावधािों के अिुसार, ग्राम, मध्यवती और नजला स्तर पर पचां ायतों के सभी सदस्य सीधे
जिता द्वारा चिु े जाएांगे। इसके अलावा, मध्यवती और नजला स्तर पर पांचायतों के अध्यक्ष का चिु ाव अप्त्यक्ष रूप से उिके निवागनचत
सदस्यों द्वारा और उिमें से नकया जाएगा।
हालााँनक, ग्राम स्तर पर पचां ायत के अध्यक्ष का चिु ाव राज्य नवधािमडां ल द्वारा निधागररत तरीके से नकया जाएगा। प्त्यक्ष या अप्त्यक्ष
रूप से निवागनचत पच ां ायत के अध्यक्ष और पच ां ायत के अन्य सदस्यों को पचां ायतों की बैठकों में मत देिे का अनधकार होगा।

Q. 154) भारतीय संनवधाि की ग्यारहवीं अिुसूची के अिुसार, निम्िनिनित में से कौि सी वस्तु पंचायतों के दायरे में आती
है?
1. पररवार कपयाण
2. पस्ु तकालयें
3. उच्च व्यावसानयक नशक्षा
4. सावगजनिक नवतरण प्णाली
िीचे नदए गए कूटों में से सही उत्तर चुनिए:
a) के वल 1, 2 और 3

www.iasbaba.com rrs.iasbaba.com Page 136


IASbaba’s Rapid Revision (RaRe) Series Polity Day 1-120 2021
HINDI

b) के वल 2, 3 और 4
c) के वल 1, 2 और 4
d) के वल 1, 3 और 4
Q. 154) Solution (c)
ग्यारहवीं अिुसचू ी: इसमें पांचायतों के दायरे में रखे गए निम्िनलनखत 29 कायागत्मक मदें शानमल हैं:
1. कृ नष, कृ नष नवस्तार सनहत
2. भनू म सधु ार, भनू म सधु ारों का कायागन्वयि, भनू म चकबदां ी और मृदा सरां क्षण
3. लघु नसच
ां ाई, जल प्बधां ि और वाटरशेड नवकास
4. पशपु ालि, डेयरी और मगु ी पालि
5. मछली पालि
6. सामानजक वानिकी और कृ नष वानिकी
7. लघु विोपज
8. खाद्य प्सांस्करण उद्योगों सनहत लघु उद्योग
9. खादी, ग्राम एवां कुटीर उद्योग
10. ग्रामीण आवास
11. पीिे का पािी
12. ईधि
ां और चारा
13. सडकें , पनु लया, पल
ु , घाट, जलमागग और सचां ार के अन्य साधि
14. नबजली के नवतरण सनहत ग्रामीण नवद्यतु ीकरण
15. गैर-पारांपररक ऊजाग स्रोत
16. गरीबी उन्मल
ू ि कायगिम
17. प्ाथनमक और माध्यनमक नवद्यालयों सनहत नशक्षा
18. तकिीकी प्नशक्षण और व्यावसानयक नशक्षा
19. वयस्क और अिौपचाररक नशक्षा
20. पस्ु तकालयें

www.iasbaba.com rrs.iasbaba.com Page 137


IASbaba’s Rapid Revision (RaRe) Series Polity Day 1-120 2021
HINDI

21. साांस्कृ नतक गनतनवनधयाां


22. बाजार और मेले
23. अस्पतालों, प्ाथनमक स्वास््य कें द्रों और औषधालयों सनहत स्वास््य और स्वच्छता
24. पररवार कपयाण
25. मनहला बाल नवकास
26. नवकलागां और मािनसक रूप से मदां लोगों के कपयाण सनहत सामानजक कपयाण
27. कमजोर वगों और नवशेष रूप से अिसु नू चत जानतयों और अिसु नू चत जिजानतयों का कपयाण
28. सावगजनिक नवतरण प्णाली
29. सामदु ानयक सपां नत्त का रखरखाव।

Q. 155) निम्िनिनित में से कौि सा/से भारत के संनवधाि के 73वें संनवधाि सश


ं ोधि अनधनियम, 1992 के अनिवाया
प्रावधाि हैं/हैं?
1. पांचायती राज के तीिों स्तरों पर पचां ायतों में मनहलाओ ां के नलए एक नतहाई सीटों का आरक्षण।
2. पचां ायती राज के तीि स्तरों पर पचां ायतों में नपछडे वगों के नलए सीटों का आरक्षण।
3. पचां ायतों को स्वशासि की सांस्थाओ ां के रूप में कायग करिे में सक्षम बिािे के नलए शनक्तयााँ और अनधकार प्दाि करिा।
िीचे नदए गए कूटों में से सही उत्तर चुनिए:
a) के वल 1
b) के वल 1 और 2
c) के वल 3
d) 1, 2 और 3
Q. 155) Solution (a)
73वें सनां वधाि सश
ां ोधि अनधनियम, 1992 के प्ावधािों को दो श्रेनणयों में बाटां ा जा सकता है- अनिवायग और स्वैनच्छक।अनधनियम के
अनिवायग (अनिवायग या बाध्यकर) प्ावधािों को िई पच ां ायती राज व्यवस्था बिािे वाले राज्य काििू ों में शानमल करिा होगा। दसू री ओर,
स्वैनच्छक प्ावधािों को राज्यों के नववेक पर शानमल नकया जा सकता है। इस प्कार अनधनियम के स्वैनच्छक प्ावधाि िई पचां ायती राज
प्णाली को अपिाते समय राज्यों के भौगोनलक, राजिीनतक-प्शासनिक और अन्य जैसे स्थािीय कारकों को ध्याि में रखिे के अनधकार
को सनु िनित करते हैं।
अनिवायग प्ावधाि:

www.iasbaba.com rrs.iasbaba.com Page 138


IASbaba’s Rapid Revision (RaRe) Series Polity Day 1-120 2021
HINDI

1. नकसी गााँव या गााँवों के समहू में ग्राम सभा का सांगठि।


2. ग्राम, मध्यवती और नजला स्तर पर पांचायतों की स्थापिा।
3. ग्राम, मध्यवती और नजला स्तर पर पच
ां ायतों की सभी सीटों पर सीधे चिु ाव।
4. मध्यवती और नजला स्तर पर पच
ां ायतों के अध्यक्ष पद के नलए अप्त्यक्ष चिु ाव।
5. प्त्यक्ष या अप्त्यक्ष रूप से चिु े गए पचां ायत के अध्यक्ष और अन्य सदस्यों के मतदाि के अनधकार।
6. पचां ायतों के चिु ाव लडिे के नलए न्यिू तम आयु 21 वषग होगी।
7. तीिों स्तरों पर पच
ां ायतों में अिसु नू चत जानत और अिसु नू चत जिजानत के नलए सीटों (सदस्य और अध्यक्ष दोिों) का आरक्षण।
8. तीिों स्तरों पर पचां ायतों में मनहलाओ ां के नलए एक नतहाई सीटों (सदस्य और अध्यक्ष दोिों) का आरक्षण।
9. सभी स्तरों पर पचां ायतों के नलए पाचां साल का कायगकाल तय करिा और नकसी भी पचां ायत के अनधिमण की नस्थनत में छह
महीिे के भीतर िए नसरे से चिु ाव करािा।
10. पचां ायतों के चिु ाव करािे के नलए राज्य चिु ाव आयोग की स्थापिा।
11. पचां ायतों की नवत्तीय नस्थनत की समीक्षा के नलए प्त्येक पाांच वषग के बाद एक राज्य नवत्त आयोग का गठि।
स्वैनच्छक प्ावधाि:
1. ग्राम सभा को ग्राम स्तर पर शनक्तयााँ और कायग प्दाि करिा।
2. ग्राम पचां ायत के अध्यक्ष के चिु ाव के तरीके का निधागरण।
3. नजला पांचायतों में मध्यवती पचां ायतों या राज्य में मध्यवती पचां ायत ि होिे की नस्थनत में ग्राम पांचायतों के अध्यक्षों को
प्नतनिनधत्व देिा।
4. नजला पच
ां ायतों में मध्यवती पच
ां ायतों के अध्यक्षों को प्नतनिनधत्व देिा।
5. ससां द के सदस्यों (दोिों सदिों) और राज्य नवधानयका (दोिों सदिों) को उिके निवागचि क्षेत्रों के भीतर आिे वाले नवनभन्ि
स्तरों पर पच
ां ायतों में प्नतनिनधत्व देिा।
6. नकसी भी स्तर पर पच
ां ायतों में नपछडे वगों के नलए सीटों (सदस्य और अध्यक्ष दोिों) का आरक्षण प्दाि करिा।
7. पचां ायतों को स्वशासि की सस्ां थाओ ां के रूप में कायग करिे में सक्षम बिािे के नलए शनक्तयााँ और अनधकार प्दाि करिा (सक्ष
ां पे
में, उन्हें स्वायत्त निकाय बिािा)।
8. आनथगक नवकास और सामानजक न्याय के नलए योजिाएाँ तैयार करिे के नलए पचां ायतों को शनक्तयों और नजम्मेदाररयों का
हस्ताांतरण; और सांनवधाि की ग्यारहवीं अिुसचू ी में सचू ीबद्ध 29 कायों में से कुछ या सभी को करिे के नलए।
9. पचां ायतों को नवत्तीय शनक्तयााँ प्दाि करिा, अथागत उन्हें कर, शपु क, टोल और फीस लगािे, एकत्र करिे और उनचत करिे के
नलए अनधकृ त करिा।
10. एक पांचायत को राज्य सरकार द्वारा लगाए और एकत्र नकए गए करों, शपु कों, टोलों और फीसों को सौंपिा।
www.iasbaba.com rrs.iasbaba.com Page 139
IASbaba’s Rapid Revision (RaRe) Series Polity Day 1-120 2021
HINDI

11. राज्य की सांनचत निनध से पांचायतों को सहायता अिुदाि देिा।


12. पचां ायतों के सभी धि को जमा करिे के नलए निनधयों के गठि का प्ावधाि।

Q. 156) निम्िनिनित कथिों पर नवचार करें:


1. भारत के नियांत्रक-महालेखापरीक्षक पचां ायतों द्वारा खातों के रखरखाव और ऐसे खातों की लेखा परीक्षा के सांबांध में प्ावधाि
करते हैं।
2. राज्य चिु ाव आयक्त
ु की नियनु क्त भारत के राष्ट्रपनत द्वारा पांचायतों के सभी चिु ावों के अधीक्षण, निदेशि, नियांत्रण और सांचालि
के नलए की जाती है।
उपरोक्त में से कौि सा कथि सही हैं?
a) के वल 1
b) के वल 2
c) दोिों 1 और 2
d) ि तो 1 और ि ही 2
Q. 156) Solution (d)
पांचायतों के खातों की लेखापरीक्षा: राज्य नवधािमांडल पचां ायतों द्वारा खातों के रखरखाव और ऐसे खातों की लेखा परीक्षा के सांबांध में
प्ावधाि कर सकता है।
राज्य चिु ाव आयोग:
• मतदाता सचू ी तैयार करिे और पचां ायतों के सभी चिु ावों के सचां ालि का अधीक्षण, निदेशि और नियत्रां ण राज्य चिु ाव आयोग
में निनहत होगा। इसमें एक राज्य चिु ाव आयक्त
ु होता है नजसे राज्यपाल द्वारा नियक्त
ु नकया जाता है।
• उिकी सेवा की शतें और पद का कायगकाल भी राज्यपाल द्वारा निधागररत नकया जाएगा। राज्य उच्च न्यायालय के न्यायाधीश
को हटािे के नलए निधागररत तरीके और आधारों को छोडकर उसे कायागलय से िहीं हटाया जाएगा। उिकी नियनु क्त के बाद
उिकी सेवा की शतों में उिके नलए अलाभकारी पररवतगि िहीं नकया जाएगा।
• राज्य नवधानयका पचां ायतों के चिु ाव से सबां नां धत सभी मामलों के सबां धां में प्ावधाि कर सकती है।

Q. 157) 74वें संनवधाि संशोधि अनधनियम, 1992 की नवशेषताओ ं के संदभा में निम्िनिनित कथिों पर नवचार कीनजएः
1. यह अनधनियम प्त्येक िगरपानलका में अिुसनू चत जानतयों और अिुसनू चत जिजानतयों के नलए सीटों की कुल सांख्या के कम
से कम एक नतहाई आरक्षण का प्ावधाि करता है।

www.iasbaba.com rrs.iasbaba.com Page 140


IASbaba’s Rapid Revision (RaRe) Series Polity Day 1-120 2021
HINDI

2. नवघटि होिे की नस्थनत में िगर पानलका के गठि के नलए िए चिु ाव उसके नवघटि की तारीख से छह महीिे की अवनध की
समानप्त से पहले परू े नकए जािे चानहए।
उपरोक्त में से कौि सा कथि सही हैं?
a) के वल 1
b) के वल 2
c) दोिों 1 और 2
d) ि तो 1 और ि ही 2
Q. 157) Solution (b)
74वें सनां वधाि सश
ां ोधि अनधनियम, 1992 िे भारत के सनां वधाि में एक िया भाग IX-A जोडा। यह नहस्सा 'िगर पानलकाओ'ां के रूप
में हकदार है और इसमें अिच्ु छे द 243-पी से 243-जेडजी के प्ावधाि शानमल हैं। इसके अलावा, अनधनियम िे सनां वधाि में एक िई
बारहवीं अिुसचू ी भी जोडी है। इस अिुसचू ी में िगर पानलकाओ ां के अठारह कायागत्मक मदें शानमल हैं। यह अिुच्छे द 243-डब्पयू से
सांबांनधत है।
इस अनधनियम िे िगर पानलकाओ ां को सांवैधानिक दजाग नदया। इसिे उन्हें सांनवधाि के न्यायोनचत भाग के दायरे में ला नदया है। दसू रे
शब्दों में, राज्य सरकारें अनधनियम के प्ावधािों के अिुसार िगर पानलकाओ ां की िई प्णाली को अपिािे के नलए सांवैधानिक दानयत्व
के अधीि हैं।
सीटों का आरक्षण: अनधनियम प्त्येक िगरपानलका में अिुसनू चत जानतयों और अिुसनू चत जिजानतयों के नलए उिकी आबादी के
अिुपात में िगरपानलका क्षेत्र में कुल आबादी के नलए सीटों के आरक्षण का प्ावधाि करता है। इसके अलावा, यह मनहलाओ ां के नलए
सीटों की कुल सांख्या (एससी और एसटी से सांबांनधत मनहलाओ ां के नलए आरनक्षत सीटों की सांख्या सनहत) के कम से कम एक नतहाई
आरक्षण का प्ावधाि करता है।
राज्य नवधानयका अिसु नू चत जानत, अिसु नू चत जिजानत और मनहलाओ ां के नलए िगर पानलकाओ ां में अध्यक्षों के पदों के आरक्षण के
तरीके का प्ावधाि कर सकती है। यह नपछडे वगों के पक्ष में नकसी भी िगर पानलका या िगर पानलकाओ ां में अध्यक्षों के कायागलयों में
सीटों के आरक्षण के नलए कोई प्ावधाि कर सकता है।
िगर पानलकाओ ां की अवनध: अनधनियम प्त्येक िगरपानलका के नलए पाचां साल के कायगकाल का प्ावधाि करता है। हालानां क, इसका
कायगकाल परू ा होिे से पहले इसे भगां नकया जा सकता है। इसके अलावा, िगरपानलका के गठि के नलए िए चिु ाव (ए) पाचां साल की
अवनध की समानप्त से पहले परू ा नकया जाएगा; या (बी) नवघटि के मामले में, इसके नवघटि की तारीख से छह महीिे की अवनध की
समानप्त से पहले।
लेनकि, जहाां शेष अवनध (नजसके नलए भांग की गई िगरपानलका जारी रहेगी) छह महीिे से कम है, ऐसी अवनध के नलए िई िगरपानलका
के गठि के नलए कोई चिु ाव करािा आवश्यक िहीं होगा।
इसके अलावा, एक िगरपानलका अपिी अवनध की समानप्त से पहले एक िगर पानलका के नवघटि पर गनठत के वल उस शेष अवनध के
नलए जारी रहेगी, नजसके नलए भांग की गई िगरपानलका जारी रहेगी यनद इसे भगां िहीं नकया गया था। दसू रे शब्दों में, समय से पहले

www.iasbaba.com rrs.iasbaba.com Page 141


IASbaba’s Rapid Revision (RaRe) Series Polity Day 1-120 2021
HINDI

नवघटि के बाद पिु गगनठत िगरपानलका पाचां साल की परू ी अवनध का लाभ िहीं लेती है, लेनकि शेष अवनध के नलए ही कायगरत् में रहती
है।
अनधनियम नवघटि के सांबांध में दो और प्ावधाि भी करता है:
a. एक िगरपानलका को उसके नवघटि से पहले सिु वाई का उनचत अवसर नदया जािा चानहए; तथा
b. वतगमाि में लागू नकसी भी काििू में कोई भी सश
ां ोधि पाचां साल की अवनध की समानप्त से पहले िगरपानलका के नवघटि का
कारण िहीं बिेगा।

Q. 158) निम्िनिनित कथिों पर नवचार करें:


1. भारतीय सनां वधाि का भाग XVIII राजभाषा से सांबांनधत है।
2. सांसद की राजभाषा सनमनत में राज्यसभा से 20 और लोकसभा से 10 सदस्य शानमल हैं।
उपरोक्त में से कौि सा कथि सही हैं?
a) के वल 1
b) के वल 2
c) दोिों 1 और 2
d) ि तो 1 और ि ही 2
Q. 158) Solution (d)
सनां वधाि का भाग XVII अिुच्छे द 343 से 351 में आनधकाररक भाषा से सांबांनधत है। इसके प्ावधािों को चार प्मख
ु ों में नवभानजत नकया
गया है-सघां की भाषा, क्षेत्रीय भाषाएां, न्यायपानलका की भाषा और काििू ों के ग्रथां और नवशेष निदेश।
राजभाषा अनधनियम (1963) िे सांघ के आनधकाररक उद्देश्य के नलए नहदां ी के उपयोग में हुई प्गनत की समीक्षा करिे के नलए सांसद की
राजभाषा सनमनत की स्थापिा का प्ावधाि नकया। अनधनियम के तहत, इस सनमनत का गठि अनधनियम के लागू होिे के दस साल बाद
(यािी, 26 जिवरी, 1965) नकया जािा था।
तदिसु ार, इस सनमनत का गठि 1976 में नकया गया था। इस सनमनत में ससां द के 30 सदस्य, लोकसभा से 20 और राज्य सभा से 10
सदस्य शानमल हैं।
सनमनत सांघ के आनधकाररक प्योजिों के नलए नहदां ी के प्योग में हुई प्गनत की समीक्षा करती है और उस पर नसफाररशें करते हुए राष्ट्रपनत
को एक ररपोटग प्स्तुत करती है और राष्ट्रपनत ररपोटग को सांसद के प्त्येक सदि के समक्ष रखवाएगां े और इसे सभी राज्यों के सरकारों को
भेजेंगे।

Q. 159) निम्िनिनित में से नकसे शास्त्रीय भाषा का दजाा नदया गया है?

www.iasbaba.com rrs.iasbaba.com Page 142


IASbaba’s Rapid Revision (RaRe) Series Polity Day 1-120 2021
HINDI

1. नहदां ी
2. सांस्कृ त
3. उनडया
4. उदगू
िीचे नदए गए कूटों में से सही उत्तर चुनिए:
a) के वल 1, 2 और 3
b) के वल 2 और 3
c) के वल 2, 3 और 4
d) के वल 3 और 4
Q. 159) Solution (b)
2004 में, भारत सरकार िे "शास्त्रीय भाषा" िामक भाषाओ ां की िई श्रेणी बिािे का निणगय नलया। अब तक, छह भाषाओ ां को शास्त्रीय
भाषा का दजाग नदया गया है:
• तानमल
• सांस्कृ त
• तेलगु ू
• कन्िड
• मलयालम
• उनडया
एक बार जब नकसी भाषा को शास्त्रीय घोनषत कर नदया जाता है, तो उसे उस भाषा के अध्ययि के नलए उत्कृ ष्टता कें द्र स्थानपत करिे के
नलए नवत्तीय सहायता नमलती है और प्नतनष्ठत नवद्वािों के नलए दो प्मख ु परु स्कारों का मागग भी खल
ु जाता है। इसके अलावा,
नवश्वनवद्यालय अिदु ाि आयोग से अिरु ोध नकया जा सकता है नक - कम से कम कें द्रीय नवश्वनवद्यालयों में - भाषा में प्नतनष्ठत नवद्वािों के
नलए शास्त्रीय भाषाओ ां के नलए एक निनित सख्ां या में पेशेवर पदों का निमागण करें ।

Q.160) निम्िनिनित में से कौि सी भाषा भारतीय संनवधाि की आठवीं अिुसूची में निनदाष्ट है?
1. डोगरी
2. िेपाली
3. अवधी

www.iasbaba.com rrs.iasbaba.com Page 143


IASbaba’s Rapid Revision (RaRe) Series Polity Day 1-120 2021
HINDI

िीचे नदए गए कूटों में से सही उत्तर चुनिए:


a) के वल 1 और 2
b) के वल 2 और 3
c) के वल 1 और 3
d) 1, 2 और 3
Q.160) Solution (a)
सनां वधाि की आठवीं अिसु चू ी 22 भाषाओ ां (मल ू रूप से 14 भाषाओ)ां को निनदगष्ट करती है। ये हैं असनमया, बगां ाली, बोडो, डोगरी
(डोंगरी), गजु राती, नहदां ी, कन्िड, कश्मीरी, कोंकणी, मैनथली (मैनथली), मलयालम, मनणपरु ी, मराठी, िेपाली, उनडया, पजां ाबी, सस्ां कृ त,
सथां ाली, नसधां ी, तनमल, तेलगु ु और उद।गू नसधां ी को 1967 के 21वें सश
ां ोधि अनधनियम द्वारा जोडा गया था; 1992 के 71वें सश ां ोधि
अनधनियम द्वारा कोंकणी, मनणपरु ी और िेपाली को जोडा गया; और बोडो, डोंगरी, मैनथली और सथां ाली को 92वें सश ां ोधि अनधनियम
2003 द्वारा जोडा गया था।
सांनवधाि के प्ावधािों के सांदभग में, आठवीं अिुसचू ी में उपरोक्त क्षेत्रीय भाषाओ ां के नवनिदेशि के पीछे दो उद्देश्य हैं:
1. इि भाषाओ ां के सदस्यों को राजभाषा आयोग में प्नतनिनधत्व नदया जािा है; तथा
2. इि भाषाओ ां के रूप, शैली और अनभव्यनक्त का उपयोग नहदां ी भाषा के सांवधगि के नलए नकया जािा है।

www.iasbaba.com rrs.iasbaba.com Page 144

You might also like